★東大入試作問者になったつもりのスレ★ 第十六問

このエントリーをはてなブックマークに追加
1132人目の素数さん
理系で数学が得意な高校生が25〜50分で
解ける問題を考えてうぷするスレ。
これ以外の難易度の問題はスレ違いとなります。
関連スレへどうぞ

過去ログは>>2以降
2132人目の素数さん:2008/08/31(日) 02:07:37
 過去ログ
★東大入試作問者になったつもりのスレ★
http://science.2ch.net/test/read.cgi/math/1000592003/
★東大京大入試作問者になったつもりのスレ★
http://science.2ch.net/test/read.cgi/math/1046165076/
★東大入試作問者になったつもりのスレ★ 第三問
http://science3.2ch.net/test/read.cgi/math/1069171672/
★東大入試作問者になったつもりのスレ★ 第4問
http://science3.2ch.net/test/read.cgi/math/1099493043/
★東大入試作問者になったつもりのスレ★ 第五問
http://science4.2ch.net/test/read.cgi/math/1116752400/
★東大入試作問者になったつもりのスレ★ 第六問
http://science4.2ch.net/test/read.cgi/math/1134000000/
★東大入試作問者になったつもりのスレ★ 第七問
http://science4.2ch.net/test/read.cgi/math/1148569109/
★東大入試作問者になったつもりのスレ★ 第八問
http://science6.2ch.net/test/read.cgi/math/1166904000/
★東大入試作問者になったつもりのスレ★ 第九問
http://science6.2ch.net/test/read.cgi/math/1182629190/
★東大入試作問者になったつもりのスレ★ 第十問
http://science6.2ch.net/test/read.cgi/math/1188545067/
★東大入試作問者になったつもりのスレ★ 第十一問
http://science6.2ch.net/test/read.cgi/math/1190854032/
★東大入試作問者になったつもりのスレ★ 第十二問
http://science6.2ch.net/test/read.cgi/math/1194120000/
★東大入試作問者になったつもりのスレ★ 第十三問
http://science6.2ch.net/test/read.cgi/math/1199706844/
★東大入試作問者になったつもりのスレ★ 第十四問
http://science6.2ch.net/test/read.cgi/math/1204606214/
★東大入試作問者になったつもりのスレ★ 第十五問
http://science6.2ch.net/test/read.cgi/math/1212563635/
3132人目の素数さん:2008/08/31(日) 02:26:23
4132人目の素数さん:2008/08/31(日) 02:49:01
自然対数の底をe、および自然数nについて
e/(2n+2)≦e-(1+1/n)^n≦e/(2n+1)
が成り立つことを示せ。

このスレで似たような問題出てたなかったっけ?
図書館で刷った大数の宿題で見つけたのだけど。
5132人目の素数さん:2008/08/31(日) 02:50:09
このスレじゃなくて過去スレの間違い。
6132人目の素数さん:2008/08/31(日) 03:55:03
>>5
不等式スレじゃろ。
7132人目の素数さん:2008/08/31(日) 11:50:10
さっき道歩いてたら>>1がキモい顔して近寄ってきたでまじムッカついてボコボコにしたら
鼻血出て眼鏡割れて前歯折れて「あがああ!まえば!おれた」て叫んでうざかったで
口に牛肉ねじこんだら「んごごごごご」とか呻いてたでオメガ便器に顔突っ込んだら
鼻血まみれで「ばぶう」とか言ってて超絶笑えたでとどめにしねバーカとか罵声あびしたったwwwww
wwwwwwwwwwwwww>>1悲惨wwwwwwwwwwwwwwwwwwwwwwwwwwwwwww
8132人目の素数さん:2008/08/31(日) 14:15:33
放物線P:y=ax^2 と 円C:x^2 + y^2 = a^2 (ただしa>0)
が囲む面積をS(a)とする
S(a)の最大値を求めよ
9132人目の素数さん:2008/08/31(日) 14:26:06
[前スレ.956]

3次元空間内で次の不等式で表される多面体の体積を求めよ
 |x+y+z| + |-x+y+z| + |x-y+z| + |x+y-z| ≦ 4,
10132人目の素数さん:2008/08/31(日) 14:29:02
>>9

(略解)
場合分けすると、左辺は
 4|x|, 4|y|, 4|z|, 2|x+y+z|, 2|-x+y+z|, 2|x-y+z|, 2|x+y-z|,
となるので、14面体である。
6面が正方形、8面が正3角形である。(立方8面体と言うらしい.)
頂点は12個。 (x,y,z) = (±1,±1,0)、 (±1,0,±1)、 (0,±1,±1).
稜は24本で、長さは√2.

・正方形は、面積S=2、高さh=1, 体積 (1/3)Sh=2/3,
・正3角形は、面積S=(√3)/2、高さh=2/(√3), 体積 (1/3)Sh=1/3,  (*)
よって、体積V = (2/3)*6 + (1/3) *8 = 20/3.

*) 正3角形の中心は (±2/3,±2/3,±2/3). よって高さh=2/(√3).
11132人目の素数さん:2008/08/31(日) 15:15:07
>>6
そうか、ありがとう。
数オリスレで話題になったことがあった富永(理V→同大医学部卒→オウム)について調べようと、この年の大数をコピーしてきたけど、本当に宿題正解者の常連だった。

逆行列をもつ任意の2次の正方行列Aについてつぎの命題が真であることを証明せよ。
命題:任意の角θ(0<θ<π)についてp↑、q↑のなす角がθで、Ap↑とAq↑のなす角もθとなるような0↑でないp↑、q↑が存在する。
(86' 6月)

は富永の解答レポートが掲載されてた。暇な時間で適当に書き上げてtexで答えうpしときます。
12132人目の素数さん:2008/08/31(日) 17:17:49
>>522
じゃあ質問
逆行列をもつ任意の2次の正方行列Aについてつぎの命題が真であることを証明せよ。
命題:任意の角θ(0<θ<π)についてp↑、q↑のなす角がθで、Ap↑とAq↑のなす角もθとなるような0↑でないp↑、q↑が存在する。
証明教えてくれ
13132人目の素数さん:2008/08/31(日) 17:46:12
えらい長距離パスやなw
14132人目の素数さん:2008/08/31(日) 18:06:30
>>11
存在するわけねーじゃん
p↑とq↑のなす角は一意なんだから、任意のθになるわけがない
15132人目の素数さん:2008/08/31(日) 19:22:05
>>14
バカ?
16132人目の素数さん:2008/08/31(日) 19:47:09
>>15
は?どこが間違いか指摘してみろクズ
17132人目の素数さん:2008/08/31(日) 21:02:00
∀θと∃p↑、q↑ such that 〜 を ∃p↑、q↑∀θ↑ such that と読んでいるようだ。
18132人目の素数さん:2008/08/31(日) 21:40:36
>>11はかなり大雑把に書くと
任意のθについて〜が成り立つようなベクトルp,qが必ず存在することを証明しろ
ってことだよな?
とくにおかしい点は無いと思うが
19132人目の素数さん:2008/09/01(月) 02:52:03
x進法で表された方程式x^3-3x^2-x+3=0を
x-2進法で表せ。
20132人目の素数さん:2008/09/01(月) 04:07:20
題意がよく分からん
もしかして、ただの組み立て除法?
21132人目の素数さん:2008/09/01(月) 04:11:33
t=x-2とおいて与式をtの式にすればいいのか?
22132人目の素数さん:2008/09/01(月) 07:41:08
x進法なんて出さないだろ
23132人目の素数さん:2008/09/01(月) 08:52:45
>>12
成分計算すりゃいいんじゃないの?
24132人目の素数さん:2008/09/01(月) 08:57:55
>>23
エレガントに解いてお
25132人目の素数さん:2008/09/01(月) 09:38:09
実数列b[0],b[1],b[2],…はb[n+2]−2b[n+1]+b[n]≦0 (n=0,1,2,3,…)を満たすとする。
また、実数列a[1],a[2],a[3],…はΣ[k=1〜n]a[k]≧b[n] (n=1,2,3,…)を満たすとする。
このとき、次が成り立つ。
Σ[k=1〜n]a[k]^2≧Σ[k=1〜n](b[k]−b[k−1])^2 (n=1,2,3,…)
また、この不等式において、a[n]=b[n]−b[n−1] (n=1,2,3,…)のときのみ等号が成り立つ。
26132人目の素数さん:2008/09/01(月) 10:11:27
>>25
ほうほう
それで?
27132人目の素数さん:2008/09/01(月) 10:26:30
0=b[0]≦b[1]≦b[2]≦… も必要だった(´・ω・`)
28132人目の素数さん:2008/09/01(月) 10:27:55
>>27
出題してんの?
29132人目の素数さん:2008/09/01(月) 11:04:36
解く気がない奴にとっては、出題しようがしまいが、
そこに書かれているのは「定理」であり、逆に、
解く気がある奴にとっては、出題しようがしまいが、
そこに書かれているのは「問題」である。

出題か否かを問うのはナンセンス。
30132人目の素数さん:2008/09/01(月) 16:11:00
なんだ結局>>14はただのゆとりだったのか
3111:2008/09/01(月) 22:05:04
32132人目の素数さん:2008/09/01(月) 22:15:22
>>10
 「立方8面体」は、立方体(稜の長さ2)から、頂点と3稜の中点を結んだ4面体(体積1/6)を除いたもの。
 V = (2^3) - (1/6)*8 = 20/3.
337743:2008/09/01(月) 23:06:20
>>32
明解な解答ですね。
対称性を利用して切断面を考えて積分でもできませんか?
34132人目の素数さん:2008/09/05(金) 03:52:31
|{(-3)^n-(-1)^n}/n!|≦13/3

を示せ
35132人目の素数さん:2008/09/05(金) 16:19:21
a[n]=|{(-3)^n-(-1)^n}/n!|=|{(-1)^n(3^n-1)}/n!|=(3^n-1)/n!
a[1]=2,a[2]=4,a[3]=13/3
n≧4でa[n]≦3^n/n!≦3^n/(3^(n-4)4!)=3^4/4!=27/8<13/3
36132人目の素数さん:2008/09/05(金) 18:34:34
また糞問かよ
37132人目の素数さん:2008/09/06(土) 01:03:07
前スレにまだ解かれてない面白そうな問題あったからそれでも解いて待ってなよ
38132人目の素数さん:2008/09/10(水) 01:50:21
M 個の石の山と N 個の石の山がある。ただし M ≦ N である。
二人で交互に一度ずつ石を取っていき、最後の石を取ったほうが負けとなる。
片方の山から石を取るか、或いは両方の山から同数ずつ石を取れる。

a = (-1 + √5)/2, [x]を実数 x の整数部分として、

(M, N) = (0,1),(2,2), ([na], [na] + n)のときに後手必勝、その他のとき先手必勝となることを示せ

http://science6.2ch.net/test/read.cgi/math/1209732803/538-548
39132人目の素数さん:2008/09/12(金) 00:50:04
Fn(x)=納k=1,n] x^(k-1) とする。F5^(n-1)(x)≠0のとき、
F5^n(x)=0の解をそれぞれ2・5^(n-1)乗したものの総和を求めよ。
ただし重解の有無についての証明は無視してよく、N重解はN個の解として扱うものとする。


はじめ、持ち点を1とする。n個中1個が当たりのくじ引きを引き、当たりなら持ち点を倍にして戻し、
はずれなら何もせず戻すという動作をn回繰り返し、試行後の持ち点の期待値をXとする。
また、上記のようにして、当たりの時にa倍していったときの期待値をX'とする。
n→∞としたとき、X'がXの倍以上になるための最小の自然数aを求めよ。


An=(2008^x)/{(k^a)x+k^b}^(k^c) とし、F(x)=A1*A2*A3*・・・*An とする。
lim[n→∞] F'(0)/F(0)・n^m =α が0<α<log2008 を満たすための、
整数a,b,cの関係式と実数mの値、またその時のαを求めよ。


正直小問つけたほうがいい気がするけど、その前に問題として成り立ってるかどうか怪しいのもあるから、
まぁまずお前らが解いてくれ。んで難易度調整とかしてみてくれ。
40132人目の素数さん:2008/09/12(金) 23:50:46
最後の問題、An=(2008^x)/{(k^a)x+k^b}^{-(k^c)} だった
41132人目の素数さん:2008/09/16(火) 12:29:47
与えられた実数係数の整式f(x)について∫[0→1]f(x)dx=2、∫[0→1]xf(x)dx=3になるとする。
そのとき∫[0→1](f(x)-ax-b)^2dxの値を最小にする実数aおよびbの値を求めよ。
42132人目の素数さん:2008/09/16(火) 22:36:01
>>8
PとCの交点をA,Bとする。
 A(√(b/a), b) B(-√(b/a), b)
ここに b = {√(1+4a^4) -1}/(2a),
線分OAとPで囲まれた部分の面積は b^(3/2) / (6√a),
線分OBとPで囲まれた部分の面積は b^(3/2) / (6√a),
扇形OABの面積は (1/2)(a^2)(∠AOB) = (a^2)arccos(b/a),
 S(a) = b^(3/2) / (3√a) + (a^2)arccos(b/a),
aが大きくなるとき発散の予感
43132人目の素数さん:2008/09/16(火) 22:53:49
>>41
 I(a,b) = ∫[0→1] {f(x)-ax-b}^2 dx,
とおくと、
 ∂I/∂a = -2∫[0→1] {xf(x) -ax^2 -bx} dx -2{∫[0→1] xf(x)dx -a/3 -b/2},
 ∂I/∂b = -2∫[0→1] {f(x) -ax -b} dx -2{∫[0→1] f(x)dx -a/2 -b},
I(a,b) が最小になるのは ∂I/∂a =∂I/∂b = 0 のとき。
 a=24, b=-10.

http://ja.wikipedia.org/wiki/最小二乗法
http://mathworld.wolfram.com/LeastSquaresFitting.html
44132人目の素数さん:2008/09/25(木) 16:10:15
2^a*3^b*5^c*7^dが2つの整数の平方の和になるときの
整数a,b,c,dの必要十分条件を求めよ
45132人目の素数さん:2008/09/25(木) 16:24:23
辺の長さaの立方体ABCD-EFGHにおいて、辺ABを軸に立方体を回転させた体積V1、
辺ADを軸に立方体を回転させた体積V2、辺AEを軸に立方体を回転させた体積V3の
共通部分の体積をaを用いて表せ。
46132人目の素数さん:2008/09/25(木) 19:18:31
pが素数,x1,…,xnが整数のとき
(x1+…+xn)^pをpで割った余りは
x1^p+…+xn^pをpで割った余りと等しいことを示せ
47132人目の素数さん:2008/09/25(木) 19:41:50
>>45
a^3になったけど違うよな
48132人目の素数さん:2008/09/25(木) 19:55:03
>>46

〔多項定理〕
 (x1+…+xn)^p = Σ[r1+r2+…+rn=p] {p!/[r1!*r2!*・・・・・*rn!]} (x1^r1) (x2^r2) ・・・・ (xn^rn),

ところで、ri>0, rj>0 (i≠j) ならば {p!/[r1!*r2!*・・・・・rn!]} は分母にpを含まないから、pの倍数。

∴ Σ[i=1,n] xi^p が残る。
49132人目の素数さん:2008/09/25(木) 20:52:08
a[1]=2,
a[n+1]はa[n]の各桁の10乗の和
とする。このとき同じ数字がでることを証明せよ。
50132人目の素数さん:2008/09/25(木) 21:13:23
んなこたーない
51132人目の素数さん:2008/09/25(木) 21:16:04
2008項の自然数からなる等差数列で各桁の和も等差数列であるものは存在するか?
52132人目の素数さん:2008/09/25(木) 21:33:13
1,1,1,1,…,1,1 (2008個)
53132人目の素数さん:2008/09/25(木) 21:43:20
公差>0じゃないと問題にならないな
54132人目の素数さん:2008/09/25(木) 22:16:48
>>49
a[n]は有界な整数列なので、鳩ノ巣原理より題意が従う
55132人目の素数さん:2008/09/25(木) 22:27:14
>>47正解
56132人目の素数さん:2008/09/25(木) 23:03:08
>>54
a[n]って有界とは限らないでしょ
ある数より小さいa[n]が無限個あることはわかるけど
57132人目の素数さん:2008/09/25(木) 23:44:06
>>39誰か解かない?
58132人目の素数さん:2008/09/26(金) 00:04:07
>>56
各位の数の10乗 < 10^10であることから、数列の作り方より
a[1] < 10 : 1桁
a[2] < 1 * 10^10 = 10^10 : 11桁以下
a[3] < 11 * 10^10 = 10^12 : 13桁以下
a[4] < 13 * 10^10 < 10^12 : 13桁以下
となり、全ての項が13桁以下であることが分かる

自明だと思って説明を入れなかったスマン
59132人目の素数さん:2008/09/26(金) 00:20:19
賢いな〜
俺の証明はこれを見たらウンコみたいなもんだわ
60132人目の素数さん:2008/09/26(金) 00:22:33
>>59
別証明が思いつかないから教えてくれ
>>49の問題じゃなかったらスマン
61132人目の素数さん:2008/09/26(金) 03:41:44
>>49
a[25374] = a[28338] = 19871647813
62132人目の素数さん:2008/09/26(金) 05:00:35
>>49は東大模試の改変だな。2乗を10乗に変えただけ。
63132人目の素数さん:2008/09/26(金) 18:51:09
郵便切手の問題でも出しておけば受験生パワーで誰か解いてくれそうだな
64132人目の素数さん:2008/09/26(金) 21:49:41
>>60
定義から
a[n+1]≦9^10*(1+loga[n])
であり
x>10^11だと
9^10*(1+logx)<x
であるからa[n]>10^11のとき
a[n+1]<a[n]
これより
少なくともa[n]<10^11までは減少数列になる
よって
a[n]<10^11となるnは無限個あるので
a[m]=a[n]となることがある
65132人目の素数さん:2008/09/27(土) 22:34:37
正二十面体のそれぞれの面に1,2,3のいずれかを1つずつ配置していく。

ある面とその面ととなりあう3つの面の数の積が奇数になる配置の仕方は何通りか。
また和が奇数になる配置の仕方は何通りか。
ただし使わない数があってもよいとし、回転して他のものと同じになる配置は考えない。
66132人目の素数さん:2008/09/28(日) 00:58:59
>>65
ある面ってなんだよ
67132人目の素数さん:2008/09/28(日) 08:19:16
>>66
普段はまじめなサラリーマンなんだけど、女装して近所の公園で野糞する趣味を持っているとか。
68132人目の素数さん:2008/09/28(日) 17:14:40
>>67
女装して近所の公園で糞証明する趣味? (>>59 みたいに)
69132人目の素数さん:2008/09/29(月) 04:31:49
a[1],…,a[n]を正の実数としたとき
(a[1]^a[1])*…*(a[n]^a[n])≧(a[1]*…*a[n])^(a[1]+…+a[n])
が成り立つことを証明せよ
70132人目の素数さん:2008/09/29(月) 04:34:38
↑間違えた

a[1],…,a[n]を正の実数としたとき
{(a[1]^a[1])*…*(a[n]^a[n])}^n≧(a[1]*…*a[n])^(a[1]+…+a[n])
が成り立つことを証明せよ
71132人目の素数さん:2008/09/29(月) 06:40:47
>>70の不等式って成り立つ?

n=2,a_1=1/2,a_2=3/2 の時、成り立たないような気がする。
72132人目の素数さん:2008/09/29(月) 10:31:29
>>70

y=log(x) は単調増加ゆえ, Σ同順序積 ≧ Σ乱順序積 より
 n{a[1]・log(a[1]) + a[2]・log(a[2]) + ・・・・ + a[n]・log(a[n])}
 ≧ (a[1] + a[2] + ・・・・ + a[n])・{log(a[1]) + log(a[2]) + ・・・・ + log(a[n])},
両辺の真数をとる。
73132人目の素数さん:2008/09/29(月) 21:01:27
>>71
成り立ってるわ
74132人目の素数さん:2008/09/29(月) 21:58:50

              ,-─‐ 、
             / iiii i ヽ、、
            /ゞ、i!llllliii川//ヽ、
           /ミ〃      〃彡ヽ
          lミミ        彡彡}
          lミミ,r‐-、 ,,r─、 彡彡ll|
          iミミ ィェx   ,rェt  彳彡!
           ',   .:       9}"
            !   ::,、,、    l_丿
            ',  _,_   /、
            rゝ  =   ノi!ヽト、
          -{;ヽ` ー─ " /;/: : \
        /: : : |;;;\     /;;;;/: : :/: :\
      /: : : : : :│;;;;;;\/;;;;;;;;/: : :/: : : : :\


           成 田 テ ル (74)
75132人目の素数さん:2008/09/29(月) 22:50:20
a,bを実数とする
x^4+ax^3+bx^2+ax+1=0
が実数解を持つとき
a^2+b^2の最小値を求めよ
76132人目の素数さん:2008/09/30(火) 00:39:55
a^2+b^2≧2
77132人目の素数さん:2008/09/30(火) 03:03:27
それはない
78132人目の素数さん:2008/09/30(火) 05:45:20
相反方程式→二次方程式→領域→糞問糸冬了
79132人目の素数さん:2008/09/30(火) 06:38:47
nを自然数とし2^nの最上位の位の数をa[n]とする.
このとき(a[k],a[k+1],…,a[k+10],a[k+11])は何種類あるでしょう.
80132人目の素数さん:2008/09/30(火) 19:32:14
めんどいけど数えたところ52種類だった
81132人目の素数さん:2008/09/30(火) 20:57:48
数学の問題ってどうやって作るんですか?
82132人目の素数さん:2008/09/30(火) 21:31:57
天才は突然思いつく
83132人目の素数さん:2008/10/02(木) 00:36:11
a^2+b^2=4/5
84132人目の素数さん:2008/10/02(木) 00:54:22
一つの面が4マス(2×2)のルービックキューブは何通りあるでしょう。ただし回転して重なるのは同一とする。
85132人目の素数さん:2008/10/02(木) 01:04:30
8!×(3^8)/(24*3)=3674160
86132人目の素数さん:2008/10/02(木) 08:29:59
「ルービックキューブは何通りあるでしょう」って訊かれてもなあ。
大きさの違い、色づかいの違い、材質の違い等、何を差異とするかによるからなあ。
87132人目の素数さん:2008/10/02(木) 09:09:55
すべて異なるものとみなす。
88132人目の素数さん:2008/10/02(木) 20:55:21
大学入試の組合せの問題だとよく
>>85みたいな答えを書く人居るけど、こういうの採点に困るよね。
塾とかだとほぼ 0 点になることが多いし、たぶん実際の入試でもそうだと思う。

組合せの問題って日本語能力のテスト的な側面があるから良いよね。
89132人目の素数さん:2008/10/02(木) 22:22:36
ネイピア数(自然対数の底)の小数第1位の数字が7であることを証明せよ。
90132人目の素数さん:2008/10/02(木) 22:40:07
>>88
大学入試の採点をしたことがあるんかい……
本職も交じってるんだな
91132人目の素数さん:2008/10/02(木) 23:29:00
>>81

・既存問題の改良、拡張
・自分が疑問に思うことをそのまま問題にする
・適当な数学分野から題材をとってきて問題を作る
92132人目の素数さん:2008/10/03(金) 01:40:02
良問作った時ってガッツポーズするの?
93132人目の素数さん:2008/10/03(金) 02:06:40
むしろこのスレに投稿して誰かに解いてもらえたらガッツポーズ。
94132人目の素数さん:2008/10/03(金) 02:07:29
(b[n])^2+1がa[n](a[n]+1)の倍数となるような
自然数からなる単調増加数列a[n],b[n]が存在することを示せ
95132人目の素数さん:2008/10/03(金) 02:31:09
a,b,c,dを自然数とする
このとき(a^3+b^3)/(c^3+d^3)がすべての有理数を表すことができることを示せ
96132人目の素数さん:2008/10/03(金) 02:31:59
訂正
有理数⇒正の有理数
97132人目の素数さん:2008/10/03(金) 11:00:27
>>95
ある正の有理数pをとり、p=n/mとする。
(p/2)^(1/3)<q<(2p)^(1/3)となるような有理数qが存在する。
q=y/xとおくと、
2my^3>nx^3、2nx^3>my^3

a=nx^3*y+my^4
b=2nx^3*y-my^4
c=mxy^3+nx^4
d=2mxy^3-nx^4
とすると、a,b,c,dはいずれも自然数。

代入して計算すると
(a^3+b^3)/(c^3+d^3) = n/m = p
98132人目の素数さん:2008/10/03(金) 11:08:01
ちなみに、
>>95でa,b,c,dが自然数ではなく整数ならば、
>>97のqに関するくだりは不要で、x,yを全部なくしてしまえばおk
99132人目の素数さん:2008/10/04(土) 10:33:16
>>97どっから思いついたか説明して!突然思いつくとか卑怯だし!
100132人目の素数さん:2008/10/04(土) 10:43:48
8!3^8/3*2=44089920
101132人目の素数さん:2008/10/04(土) 10:57:56
a,b,c,d
(a^3+b^3)/(c^3+d^3)
q=k/s (k,s)=1
c=sj,d=st
a=kj,b=kt

102132人目の素数さん:2008/10/04(土) 12:00:15
競馬板に書いたのですが、誰も解いてくれなくて悲しかったので
(ちょうど大学入試レベルでもありますし)ここに書きます。
nは2以上の整数です。
平面内に2n個の点があって、どの3点も同一直線上にないとする。
ここから、2点を選んで線分を何本か引く。(最大n(2n-1)本引けます)
n^2+1(nの二乗+1)本以上の線分を引けば、ある3点が存在して
その3点が互いに線分で結ばれていることを示して下さい。

解けた人はスプリンターズステークスの予想でもついでに書いといてください。
103132人目の素数さん:2008/10/04(土) 12:27:11
競馬板に書いて、どうして解いてくれると思ったかが疑問だなwww
104132人目の素数さん:2008/10/04(土) 14:20:19
>>99
卑怯って言われちまったw

えーと、最初は
(m+α)^3+(m-α)^3=2m(m^2+3α^2)
(n+β)^3+(n-β)^3=2n(n^2+3β^2)
という形を思いついたので、
m^2+3α^2=n^2+3β^2となるような整数の組α,βを作ることを考えたが、
3が邪魔だったので、
(3m+α)^3+(3m-α)^3=18m(3m^2+α^2)
(3n+β)^3+(3n-β)^3=18n(3n^2+β^2)
とおきなおし、
3(n^2-m^2)=α^2-β^2
3(n-m)(n+m)=(α-β)(α+β)
から、仮に
α+β=3(n-m),α-β=n+m
と置くと、
α=2n-m,β=n-2mであり、
a=3n+β=4n-2m,b=3n-β=2n+2m
c=3m+α=2n+2m,d=3m-α=-2n+4m
とすれば、(a^3+b^3)/(c^3+d^3)=n/mとなることがわかった。
(実際には、a,b,c,dは半分にしても可)

ただし、このままでは4n-2mと-2n+4mが自然数となるには1/2<n/m<2の条件が
必要なので、n/m=(y^3*x^3*n)/(x^3*y^3*m)と考えて、
1/n<(x^3*n)/(y^3*m)<2となるようにし、
(x^3*n)/(y^3*m)=(a^3+b^3)/(c^3+d^3)なら
(y^3*x^3*n)/(x^3*y^3*m)=((ya)^3+(yb)^3)/((xc)^3+(xd)^3)になると考えた。
105132人目の素数さん:2008/10/04(土) 14:23:07
>>104の修正
誤:1/n<(x^3*n)/(y^3*m)<2となるようにし、
正:1/2<(x^3*n)/(y^3*m)<2となるようにし、
106132人目の素数さん:2008/10/04(土) 14:50:05
>>104なるほどなるほど!納得した!ありがとう!
107132人目の素数さん:2008/10/04(土) 15:46:07
>>102
大学入試レベルの解答は思い付かないが、グラフ理論を使えば出来た。
2n個の頂点をもち条件(A)を満たす無向グラフG=(V,E)で、|E|が最大のものを求める。
(A)どの3点をとっても、辺で結ばれない2点が存在する
Gは2-連結(Gは連結で、Gから頂点を1つ取り除いても連結)としてよい。
なぜならGが非連結なら別の連結成分の間に辺を加えることにより、
Gから頂点vを除いたグラフが非連結なら、連結成分のどれかはvに隣接しない頂点を持つか、
さもなくばGはE={(v,w)|w∈V\{v}}なる放射状のグラフ(|E|=n-1)だからである。
Gが2-連結ならば各頂点を1回ずつ通る閉路が存在するので、この閉路に沿って
頂点にV={v[1],...,v[2n]}, (v[i],v[i+1])∈E, v[2n+i]=v[i]となるよう番号を付ける。
各頂点v[i]について、(v[i],v[j])∈Eと(v[i],v[j+1])∈Eは同時には成り立たないから、
v[i]の次数(v[i]に接続する辺の数)は高々nであるから|E|≦n^2となる。
なお、各v[i]がv[i+1],v[i+3],...,v[i+2n-3],v[i+2n-1]と接続するようなグラフを
任意のnに対して作ることができて(正2n角形を描いてみよ)、このときE=n^2を達成できる。
108132人目の素数さん:2008/10/04(土) 15:47:46
×Gが非連結なら別の連結成分の間に辺を加えることにより、
○Gが非連結なら別の連結成分の間に辺を加えることが可能であり、
×さもなくばGはE={(v,w)|w∈V\{v}}なる放射状のグラフ(|E|=n-1)だからである。
○さもなくばGはE={(v,w)|w∈V\{v}}なる放射状のグラフ(|E|=2n-1)だからである。
109132人目の素数さん:2008/10/04(土) 19:52:04
 
110132人目の素数さん:2008/10/04(土) 21:11:56
ある3点が存在してその3点が互いに線分で結ばれている

線がm本->点が2m個
線がn^2+1ー>点が2n^2+2>2n

ピジョンホール
111132人目の素数さん:2008/10/04(土) 21:46:14
点と線を考える,点の色は白か黒

操作1:点に線を足してその新しい端を白点にし、もとの点のいろを逆転(例:白ー>黒)する。
操作2:線の中間に白点をたし、その両端の点の色を逆転させる。

G1を単独の白点とする

G1: 白

1、白ー白ー白

2、
  白   
  |
白ー白ー白
  |
  白

3、

  白   
  |
白ー白ー白   
  |
白ー白ー白
  |
  白
  |
  白

はG1から操作1、2を有限回やってできることを示しなさい。
112132人目の素数さん:2008/10/04(土) 21:47:13
3、

  白   
  |
白ー白ー白   
  |
白ー白ー白
  |
  白
113132人目の素数さん:2008/10/04(土) 21:53:36
4、



から

n個の白の直線を作るとき、nはどんな数か。

白ー白ー・・・・ー白

5、

白からできるグラフのオイラー数を計算しなさい。
114132人目の素数さん:2008/10/04(土) 22:48:36
>>107-108

すっげーカッケー答っすね。もうビンビンです。
一応僕が用意していた答です。n=2の時はまあできたとします。
nの時成り立っているとします。(数学的帰納法を使います)
さて、頂点が2n+2コある時ですが、少なくとも(n+1)^2+1本線分を引くわけですから
当然ある二点が存在してその二点は線分で結ばれています。
わかりやすいように、その2点をa1,a2とでもおいて、
残りの2n個の点をb1,b2…,b(2n)とおくことにします。
115132人目の素数さん:2008/10/04(土) 23:05:30
つづきです
b(i)どおしでn^2+1本線分を引くと仮定により線分で結ばれた3点が存在さますので
b(i)どおしでは多くてもn^2本しか線分を引いていないとします。
a1とa2は線分で結んでいますので少なくとも残り(n+1)^2+1-n^2-1=2n+1本線分を引かないといけません。
これはa(i)とb(i)を結ぶ線分ですので、あるb(i)が存在して
a1とa2とb(i)は線分で結ばれてしまいます。

明日はキセキ産駒がワンツースリーを決めて
アグネスタキオンファンを黙らせて欲しいですね(^o^)/
116132人目の素数さん:2008/10/04(土) 23:05:33
6、K色のボールをAjk個(k=色のインデックス)壺に入れて、N回引く、毎回引いたボールは同じ色の
追加のボール1個といっしょにすぐ壺に戻す。
このとき、Nを無限にしたとき、壺のなかの各色のボールの数の分布を計算しなさい。
117132人目の素数さん:2008/10/04(土) 23:31:49
>>116

割合は始めと変わらないという答えであってますか?
118132人目の素数さん:2008/10/05(日) 09:06:32
#
IRA: Interactive Real Analysis
Interactive Real Analysis is an online, interactive textbook for Real Analysis or Advanced Calculus in one real variable. It deals with sets, sequences, ...
web01.shu.edu/projects/reals/ - 3k - Cached - Similar pages
#
119132人目の素数さん:2008/10/06(月) 00:17:02
>>116ポリアの壷?
120132人目の素数さん:2008/10/06(月) 21:20:35
Suppose that f is an integrable function over a set E, and take any ε > 0. Show that

* There exists a simple function s such that

∫ E | f - s | dx < ε

* There exists a step function s such that

∫ E | f - s | dx <ε

* There exists a continuous function s such that

∫ E | f - s | dx <ε

121132人目の素数さん:2008/10/06(月) 21:22:51
# If possible, find the Riemann and Lebesgue integrals of the constant function f(x) = 1
over the Cantor middle-third set.
# Show that the restriction of a bounded continuous function to a measurable set is Lebesgue
integrable.
122132人目の素数さん:2008/10/06(月) 21:23:45
* Is the function f(x) = x Lebesgue integrable over [0, 1]? If so, find the integral.
* Is the function f(x) = x2 Lebesgue integrable over the rational numbers inside [0, 2]? If so, find the integral.
* Is the Dirichlet function restricted to [0, 1] Lebesgue integrable? If so, find the integral.
* Is every bounded function Lebesgue integrable?
123132人目の素数さん:2008/10/06(月) 22:15:09
http://changi.2ch.net/test/read.cgi/voiceactor/1221808073/13
13 :名無しさん@お腹いっぱい。:2008/09/19(金) 18:11:05 ID:sWdchyr40
Fラン私大工学部での微積分の授業のテストらしいが、金朋はこういうの解けるのだろうか?
・101次方程式 51 x^{101} - 2323 x^{100} - 45 x + 1035 = 0が区間[45^{1/100},46]の中に少なくとも一つ実数解を持つことを、Rolleの定理を使って証明せよ。
・不定積分 \int (x^{30} + x^{20} + x^{10}) (2x^{20} + 3x^{10} + 6)^{1/10} dx を求めよ。
・定積分 \int^{2}_{0} dx (2x+1)/ \sqrt{x^2+4} を求めよ。
124132人目の素数さん:2008/10/06(月) 22:44:50
>>123

(中)
 (被積分函数) = (x^29 + x^19 + x^9)*(2*x^30 + 3*x^20 + 6*x^10)^(1/10) = (1/60)f '(x)*f(x)^(1/10),
これを積分すると
 (1/66)*f(x)^(11/10) + c,

(下)
 ∫ x/√(x^2 +4) dx = √(x^2 +4) -2,
 ∫ 1/√(x^2 +4) dx = log(x+√(x^2 +4)) - log(2),
∴ 4(√2 -1) + log(1+√2),
125132人目の素数さん:2008/10/06(月) 23:33:53
http://web01.shu.edu/projects/reals/integ/index.html

ぬこでもわかるルベグ積分 
126132人目の素数さん:2008/10/07(火) 01:14:47
>>123
まあ工学部っても学科によって全然違うし。
127132人目の素数さん:2008/10/08(水) 12:08:05
逆行列をもつ2次正方行列Aにより表される平面上の1次変換f を考える。
このとき、長方形Dで、Dのfによる像がDと相似になるものが存在することを示せ。
128132人目の素数さん:2008/10/08(水) 21:08:05
>>127
>>31により,直交する2つの単位ベクトル↑p,↑qで,A↑pとA↑qも直交するものが存在する。
A↑p, A↑q それぞれの長さをa, b(>0)とおき,k=√(a/b) とおく。
D={ ↑p + t↑q | 0≦t≦k } とおくと,Dは2辺の長さが 1 と k の長方形である。
このとき,f(D)={ A↑p + t A↑q | 0≦t≦k } も長方形であり,2辺の長さは a と bk である。
a : bk = a : √(ab) = √(a/b) : 1 = k : 1 であるので,f(D) は D と相似である。■
129132人目の素数さん:2008/10/09(木) 19:31:20
188:Zeus(ゼウス)[]
2008/10/09(木) 08:33:37 ID:AQ7gcWuF0
>>186
あほ!!
そりゃあ、おれが、別のスレッドに書いた
解答だ。
君自身で、独創的な問題を作れるのかと
きいているのだ。

190:Zeus(ゼウス)[]
2008/10/09(木) 09:01:29 ID:AQ7gcWuF0
>>189
中学生の脳みそで解く問題だぞ。
難しいに決まっているだろうが。

191:Zeus(ゼウス)[]
2008/10/09(木) 09:05:38 ID:AQ7gcWuF0
高校生用には、こういう問題を
用意してある。

「2球面の交わりによってできる円に関する問題を作り、解け」


「連立方程式と線形性に関する論証問題を作り、数式を使わず論証せよ」

★★★★★茨城の高校★★★★★ part18
http://namidame.2ch.net/test/read.cgi/ojyuken/1219237794/
130132人目の素数さん:2008/10/09(木) 19:37:09
>>129
ワラタ
131132人目の素数さん:2008/10/10(金) 03:34:18
どこが面白いのか分からなくて悲しい。
132132人目の素数さん:2008/10/12(日) 00:12:44
フジタキスレに俺の書き込み張った奴でてこいやwww
133132人目の素数さん:2008/10/12(日) 01:11:58
晒し者にされたのか?
134132人目の素数さん:2008/10/25(土) 02:59:40
>>132
さっさとアフリカいけやw
135132人目の素数さん:2008/10/25(土) 05:11:58
(1) Σ[n=1,∞) 1/(n^2 −a^2)
(2) Σ[n=1,∞) 1/(n^4 −a^4)
(3) Σ[n=1,∞) 1/(n^2 +a^2)

の極限値を求めたいのですが、どうしたら良いでしょうか? a≠整数 です。
136132人目の素数さん:2008/10/25(土) 10:00:00
>>135
解析概論。
137135:2008/10/26(日) 14:01:17
(4) Σ[n=1,∞) 1/(n^2 −a^2) * (-1)^(n-1),
(5) Σ[n=1,∞) 1/(n^2 + a^2) * (-1)^(n-1),

の極限値を求めたいのですが、どうしたら良いでしょうか? a≠整数 です。

>>136
高木:「解析概論」改訂第3版, 岩波書店(1962) 第5章,§64 ?
138132人目の素数さん:2008/10/30(木) 21:39:17
>>89と並んで如何にも東大がやりそうな問題

log_{10} 2の小数第3位が1であることを証明せよ。
139138:2008/10/30(木) 21:59:57
そういえば1968年にもう一回り簡単な問題、
0.3<log_{10} 2 < 0.302を示せというのがありました。
http://www.j3e.info/ojyuken/math/php_q.php?name=tokyo&year=1968&num=1
140132人目の素数さん:2008/10/30(木) 22:34:01
3 次方程式x3 . nx + 1 = 0 の解がすべて無理数となるような整数n を求
めよ.
141132人目の素数さん:2008/10/30(木) 22:34:48
3 次方程式x^3-nx + 1 = 0 の解がすべて無理数となるような整数n を求
めよ.
142132人目の素数さん:2008/10/30(木) 22:59:29
x^3-nx + 1 = 0
(x-a)(x-b)(x-c)=x^3-(a+b+c)x^2+(ab+bc+ca)x-abc=0
c=-a-b
ab-(a+b)^2=n
ab(a+b)=1
ab-(ab)^-2=n

143132人目の素数さん:2008/10/30(木) 23:01:11
ab-(a+b)^2=-n
ab(a+b)=1
ab-(ab)^-2=-n
144132人目の素数さん:2008/10/30(木) 23:28:26
f(x)=xe^{-x}のとき、f(0.99), f(1.00), f(1.01)の大小を調べよ。
145132人目の素数さん:2008/10/31(金) 00:19:03
毎度のごとくe^x>1+x+x^2/2を示してe^0.02を評価すれば終わり。
146132人目の素数さん:2008/10/31(金) 01:34:38
exp(x)>x+1だけで十分です
147132人目の素数さん:2008/10/31(金) 01:42:38
Your solution doesn't make sense at all.
148132人目の素数さん:2008/10/31(金) 01:43:31
ごめんなさい十分じゃありませんでしたごめんなさい
149132人目の素数さん:2008/10/31(金) 06:29:48
>>144
これは大昔の大学への数学の「模試」の転載ですか?
150132人目の素数さん:2008/10/31(金) 08:29:48
いいえ, 埼玉工大の入試問題です。
151132人目の素数さん:2008/10/31(金) 20:49:51
1/7<a/b<1/6でa,bは整数で最小のaのときa+bをもとめなさいって、
a,bがマイナスなら解梨じゃないの?

152132人目の素数さん:2008/10/31(金) 20:59:38
>>143
nが整数であることの証明は?
153132人目の素数さん:2008/11/01(土) 17:30:47
n両編成の電車の車両をそれぞれ赤青黄のいずれかの色で塗ってゆく。
赤の車両が隣り合わないような塗り方は何通りあるか。
154132人目の素数さん:2008/11/01(土) 17:39:31
赤い車両が連続しない塗り方ね。
155132人目の素数さん:2008/11/01(土) 18:17:20
((1+√3)^(n+2)-(1-√3)^(n+2)) / (4√3)通り
156132人目の素数さん:2008/11/01(土) 21:08:52
>>155正解。
157132人目の素数さん:2008/11/01(土) 21:12:36
3項間漸化式か?
158132人目の素数さん:2008/11/01(土) 21:22:00
京大の過去問の改変?
159132人目の素数さん:2008/11/01(土) 22:20:08
>>158そうです。このように改変した方がちょっと難しいと思います。
160132人目の素数さん:2008/11/01(土) 23:59:07
>>153
数セミのパクリ
161132人目の素数さん:2008/11/02(日) 07:35:14
>>160
>>153は俺だが、数セミなんか参考にしてません。
あくまでも京大の2色の問題を3色に改変してみただけです。隣接3項間漸化式を立てられるかどうかの問題。
コツさえ知っていれば誰でも簡単に立てられるんだけどね。
162132人目の素数さん:2008/11/02(日) 08:04:39
では京大が数セミを下敷きにしたとか。
163132人目の素数さん:2008/11/02(日) 09:50:57
いいサイトみつけた
htttp://www.surprise002.co.nr
ソフトの確認もできたし、低価格でいいよ
ソフトの確認はネットカフェで一度インストール試したら
いいと思う、正常なら自分のPCに
164132人目の素数さん:2008/11/02(日) 17:03:58
たぶん最初に考えたのは日本人じゃなくて外国の人で、
それを数セミの出題者の先生と京大の先生が二人とも
元ネタにした、とかそんな感じだと思うぞ。

というか数セミの出題者は京大関係者じゃないんだよね?
165132人目の素数さん:2008/11/02(日) 18:51:57
こんな単純な設定の問題、パクるも何もないだろww
166132人目の素数さん:2008/11/03(月) 20:17:28
aを 0<a<1 であるような有理数とするとき、自然数n≧3に対して
(1-a^n)^(1/n) が無理数であることを示せ
167132人目の素数さん:2008/11/03(月) 21:08:31
>166
 (1-a^n)^(1/n) =b とおくと
 a^n + b^n =1,
 a∈Q, 0<a<1, n∈N, n≧3,
bが有理数ならば、フェルマーの最終定理(A.Wiles)と矛盾する。
168132人目の素数さん:2008/11/04(火) 12:19:28
数セミの出題者は安田亨だった
169132人目の素数さん:2008/11/04(火) 13:46:26
長さNメートルの紐の端と端を結んでできる輪の面積で一番大きい
面積はどれくらいか?
170132人目の素数さん:2008/11/04(火) 14:54:22
>>168
ああ、じゃあ数セミのは京大の改変だね。
171132人目の素数さん:2008/11/05(水) 20:01:52
>>169 N^2/4π
172132人目の素数さん:2008/11/05(水) 21:52:49
どこのクズだこんなひどい問題だしてるのは
173132人目の素数さん:2008/11/09(日) 16:06:23
>>169
結び目の大きさは考慮しなくていいのか?
あと、紐はどれぐらい曲げられるの? 硬い紐だと意外と曲がらないよ
174132人目の素数さん:2008/11/09(日) 17:55:53
数列{a_n}は、次の漸化式で与えられる。
a_(n+3) = -a_(n+2) + 2a_(n+1) + 8a_n
a_1 = a_2 = a_3 = 1
この時a_nのすべての項は平方数であることを証明せよ。
175132人目の素数さん:2008/11/09(日) 17:57:10
またパクリ問かよ
176132人目の素数さん:2008/11/09(日) 21:26:23
>>174

数列{b_n}を
 b_(n+2) = b_(n+1) -2*b_n,
 b_1 = b_2 = 1, b_3 =-1,
で定義すると b_n は明らかに整数で、a_n = (b_n)^2.

注) b_n = (2/√7)・2^(n/2)・sin(nβ), ここに β = arcsin(√(7/8)).
177132人目の素数さん:2008/11/10(月) 03:10:45
>>176
おお!どうなってのか教えて!
178132人目の素数さん:2008/11/11(火) 11:50:53
パクリだなこれ
何回も見たことある

新作問題キボンヌ
179132人目の素数さん:2008/11/11(火) 14:45:56
残念ながら無理
180132人目の素数さん:2008/11/12(水) 01:06:14
>>178
誰も解いてくれないから>>39解いてくれ
181132人目の素数さん:2008/11/12(水) 01:23:09
>>180
やってみるお
182132人目の素数さん:2008/11/14(金) 08:49:56
写像aは整数から整数への写像であり、
・ a(1)=1
・ a(n+2)=a(n+1)+a(n)
・ 1≦i<jならばa(i)<a(j)
・ 任意の整数mに対して、ある整数nが存在し、a(n)はmの倍数
を満たす。
このとき、写像aを求めよ
183132人目の素数さん:2008/11/15(土) 00:00:52
誰も解いてくれないから>>89>>138解いてくれ
184132人目の素数さん:2008/11/15(土) 02:22:52
頑張って数値計算するだけだからなあ。
185132人目の素数さん:2008/11/15(土) 19:00:17
円周率πが3.15未満であることを示せ
186132人目の素数さん:2008/11/16(日) 02:26:23
糞つまらん問題ばっかだな最近
187132人目の素数さん:2008/11/16(日) 11:34:23
>>182解いて
188132人目の素数さん:2008/11/16(日) 15:29:22
>>182
N→Nじゃなくて
Z→Zでいいの?
189182:2008/11/17(月) 00:18:36
>>188
うん
190132人目の素数さん:2008/11/17(月) 00:43:08
0や負の数に大して定義されてないんじゃない?
191132人目の素数さん:2008/11/17(月) 01:04:46
a[n]がn>0で定義されてれば、漸化式から0≧nに対してa[n]が求まるだろ
192132人目の素数さん:2008/11/17(月) 21:20:00
a(−1)=0。
193132人目の素数さん:2008/11/17(月) 22:52:58
>>192
それの証明が重要なんじゃないのか?
194132人目の素数さん:2008/11/17(月) 22:55:36
nを10進数表記したとき、奇数桁目に出てくる奇数の個数をa(n)とする。
例) a(111)=2、a(232)=0、a(1234)=0、a(2345)=2
納k=1,n] a(2^k)/2^k (n→∞)
の極限値を求めよ
195132人目の素数さん:2008/11/17(月) 23:00:36
>>193
フィボナッチ数列を一個ずらした数列を考えれば良い
最後の性質は、フィボナッチの場合を示せばよい(良く知られた証明)

全く以って入試に不向きな問題だな
196132人目の素数さん:2008/11/17(月) 23:02:49
>>195
いやだから、フィボナッチ数列しかないことを証明しろってことなんじゃないの?
197132人目の素数さん:2008/11/17(月) 23:04:17
×フィボナッチ数列しか
○フィボナッチ数列をずらした数列しか

a(1)=1で、a(2)=2ならフィボナッチで、条件を満たすけど
a(1)=1、a(2)=3ならリュカで条件を満たさない。条件をみたすaが、a(2)=2のみに限ることを言わないと証明じゃないのでは……
198132人目の素数さん:2008/11/17(月) 23:08:10
>>196
具体的に写像aを求めろって問題だろ
一意性を示せとはどこにも書いてないと思うが
199132人目の素数さん:2008/11/17(月) 23:10:43
求めろとしか書いてないんだから、「全部」求めろって意味だと思ってたんだが……
んで、あくまでもおれの予想としてフィボナッチをずらしたものしかなさそうなので、メインは一意性の証明かなぁと。
単に求めろだから、やっぱ「全部」じゃね?
200132人目の素数さん:2008/11/18(火) 00:30:00
m=0。
201132人目の素数さん:2008/11/18(火) 00:45:30
任意のm
202132人目の素数さん:2008/11/18(火) 06:17:14
任意の正の整数nに対して不等式

|sin1|+|sin2|+|sin3|+・・・+|sin(2n)|> 4n/5

を証明せよ。
ただしπ=3,1...sin1=0.84...,cos1=0,54...sin2=0.90...cos2=-0.41...
は証明なしで使ってもよいものとする。
203修正:2008/11/18(火) 13:11:13
写像aは整数から整数への写像であり、
・ a(1)=1
・ a(n+2)=a(n+1)+a(n)
・ 1≦i<jならばa(i)<a(j)
・ 任意の正整数mに対して、ある整数nが存在し、a(n)はmの倍数
を満たす。
このとき、写像aを全て求めよ
204132人目の素数さん:2008/11/18(火) 13:56:58
1より大きい実数a_[2]を求めよでいいじゃん
205132人目の素数さん:2008/11/18(火) 13:58:25
実数じゃなくて整数、の間違い
206132人目の素数さん:2008/11/18(火) 22:40:20
通常の1から6までの目のサイコロをn回振る。
n回目までの出た目の和が素数である確率を求めよ。
207132人目の素数さん:2008/11/18(火) 23:06:04
nを飛ばすの忘れてるぞ
208132人目の素数さん:2008/11/19(水) 00:00:34
久しぶりに来たけどあんま賑わってないね
とりあえず>>39>>182あたりが解かれてないのか
209132人目の素数さん:2008/11/19(水) 00:03:56
>>202
http://science6.2ch.net/test/read.cgi/math/1179000000/653
不等式スレ3

の式に a=1, m=2n を代入すると、
 (左辺) > n + (1/4) - 1/(4sin(1)) > n -0.0471
210132人目の素数さん:2008/11/19(水) 00:07:03
>>208
自作問題が解かれてないからって気を落とすなよ

お前が作ってないなら、糞問なんだからスルーしてやれよ
211132人目の素数さん:2008/11/19(水) 00:20:37
>>194
212132人目の素数さん:2008/11/19(水) 00:21:09
>>182
a(2)≧3のとき条件を満たさないことを示す
まずa(k)とa(k+1)は互いに素なのはユークリッドの互除法的に明らか
次にa(2)=tとするとa(k)+(t^2-2t)a(k+1)は常にt^2-t-1の倍数となる
したがって、a(k)がt^2-t-1の倍数だとa(k+1)もt^2-t-1になるがこれは矛盾
a(2)=2はフィボナッチの性質から解ける感じ
ということでa(2)=2となるときだけ

>>39はスルーされて当然だったな
一個目:-1
煮込め:X=e,X'=e^(a-1)よりa=3
三個目:対数微分とか使わせるにしてもあまりに糞問
213132人目の素数さん:2008/11/19(水) 01:22:19
>>212
多分39の一個目違う
214132人目の素数さん:2008/11/19(水) 08:02:11
a,bを2^a+3^bが平方数となるような正の整数とする。

(1)a,bはともに奇数であるか、ともに偶数であることを示せ。
(2)(a,b)としてありうるものをすべて求めよ。
215132人目の素数さん:2008/11/19(水) 17:07:25
>次にa(2)=tとするとa(k)+(t^2-2t)a(k+1)は常にt^2-t-1の倍数となる
どうやって気づいたのかkwsk
216212:2008/11/19(水) 18:04:06
>>215
実はその部分は敢えてどう解いたのかばれないような表記にしてました
実際の思考の順序は
・おそらくa(2)≧3だとある数を法としたときに0を含まない循環にできるはず
・漸化式の形から、pa(k)+qa(k+1)が常に何かを法として不変になるはず
・その「何か」と上の「ある数」を自分で作ればよいはず
・a(1)=1,a(2)=tのとき、p=t,q=-1とすればta(1)-a(2)=0で何かよさそう。このときta(2)-a(3)=t^2-t-1
・t^2-t-1が上の「何か」になるはず、実験して確認・あとはp=1,q>0になるように工作
みたいな感じでした
217132人目の素数さん:2008/11/19(水) 18:38:54
>>216
勉強になります
218132人目の素数さん:2008/11/20(木) 21:46:10
>>185
解きました。キーワードは「pi_315」
http://www1.axfc.net/uploader/He/so/160906
219132人目の素数さん:2008/11/20(木) 21:58:51
>>216
なるほどぉ!
220132人目の素数さん:2008/11/20(木) 22:35:35
>>206の問題、解けた人いる??
221132人目の素数さん:2008/11/20(木) 22:45:21
>>214
(a,b)=(4,2).
222132人目の素数さん:2008/11/20(木) 22:46:22
g:N→Nがg(1)=1、g(mn)=g(m)g(n)を任意の正整数m,nに対して満たすとき、完全な関数と呼ぶ。
F(n)=納k=1,n]f(k) が完全な関数になる完全な関数fをすべて求めよ
223132人目の素数さん:2008/11/20(木) 23:04:17
>>216
t^2−t−1=0はフィボナッチ数列の特性方程式だから、
こう考えた方が本質的な気がする。↓

a(n+2)=a(n+1)+a(n)の特性方程式はx^2−x−1=0だから、
a(1)=1,a(2)=tとするとき、m:=t^2−t−1を法とすれば
a(n)≡t^n (mod m) とキレイに解ける。もしa(n)がmの倍数
であるようなnが存在するならば、t^n≡0 (mod m)となるが、
gcd(t,m)=1であるから、m≧2のときは矛盾が起きる。
m≧2 ⇔ t≧3だから、結局、t≧3のときは矛盾が起きる。

一般化するならこうなるか。
a(n+k)=c(k−1)a(n+k−1)+…+c(1)a(n+1)+c(0)a(n) (各c(i)は整数でc(0)=±1)
a(0)=1,a(1)=t,a(2)=t^2,…,a(k−1)=t^(k−1)
という漸化式を考える。tが十分大きな自然数ならば、
(tごとに)ある正整数mが存在して、a(n)とmは互いに素(n=0,1,2,…)である。
224132人目の素数さん:2008/11/21(金) 00:11:23
三角形Tの内部(周を含む)を、動点Pが次のルールで動く。

1) Pは頂点を以外のある内点からスタートし、三角形の辺にぶつかるまでまっすぐ進む。
2) 辺にぶつかったら跳ね返る。入射角と反射角は等しいものとする。
3) 1,2を繰り返して、途中で三角形の頂点にぶつかる軌道は考えないものとする。
4) 1,2を繰り返して、途中から軌道が周期的になるもののみを考える。


三角形Tの周の長さを 1 とし、上記の軌道からなる集合をL(T)とする。
また、Tを固定したとき、L(T)に含まれる軌道の長さの最小値をm(T)とおく。
Tを変化させたとき、m(T)の最大値は存在するか?
225132人目の素数さん:2008/11/21(金) 00:52:11
>>222
F(n)=n,f(n)=1 (n=1,2,3,…)となることを示す。
与えられた条件からn=1,2,…,9のときを地道に調べることで、
n=1,2,…,9のときはF(n)=n,f(n)=1が成り立つことが言える。
以下、k≧10として、n<kのときF(n)=n,f(n)=1が成り立つすると、
n=kのときは以下の議論によってF(k)=k,f(k)=1となるので、
数学的帰納法より成立。

kが素数のとき:k≧10だからkは奇素数であり、よってk+1は合成数である。
k+1=Πpi^ei と素因数分解すると、各iに対してpi<kとなることに注意して
F(k+1)=ΠF(pi)^ei=Πpi^ei=k+1となる。一方、F(k+1)=Σ[j=1〜k+1]f(j)
であり、j<kのときは帰納法の仮定からf(j)=1なのでF(k+1)=k−1+f(k)+f(k+1)
となる。また、f(k+1)=Πf(pi)^ei=Π1^ei=1となるから、結局F(k+1)=k+f(k)
であり、これにF(k+1)=k+1を代入してf(k)=1となる。このとき
F(k)=Σ[j=1〜k]f(j)=kとなる。よってF(k)=k,f(k)=1となる。

kが合成数のとき:k=Πpi^ei と素因数分解して、F(k)の値を上と同様にして計算する。
上の議論よりも簡単にF(k)=k,f(k)=1が出る。
226132人目の素数さん:2008/11/21(金) 04:28:01
0を入れないならF(2^n)=2^nだから全部1。
227132人目の素数さん:2008/11/21(金) 05:04:37
g(1)=1っていらんだろ
228132人目の素数さん:2008/11/21(金) 08:06:29
Nに0を含める流儀だと必要かな
229132人目の素数さん:2008/11/21(金) 09:31:58
>>218
力技すぎワロタ
230132人目の素数さん:2008/11/21(金) 18:40:33
>>218
なんという力技www
231132人目の素数さん:2008/11/21(金) 18:47:50
実戦的ないい解答だ
232132人目の素数さん:2008/11/21(金) 18:49:44
まぁ、明らかに185の問題は例の問題パクって適当に作ったものだろうから、
そういう回答になっちまうのもしかたないのかもな
233232:2008/11/21(金) 18:51:21
適当に、というよりは思いつきで、のほうがしっくりくるか
234132人目の素数さん:2008/11/21(金) 19:02:30
次の2つの条件をともに満たす、定数でない整数係数多項式f(x)をすべて求めよ。

(1)f(x)のすべての係数の絶対値は1である。
(2)方程式,f(x)=0の解はすべて実数である。
235232:2008/11/21(金) 19:27:29
別の方法で>>185解いてみた
創価相乗より
cosθ+cosθ+(1/cosθ)^2≧3 (0≦θ<π/2)
両辺0からπ/12までθで積分して、
2sin(π/12)+tan(π/12)>3*(π/12)
sin(π/12)=(√6-√2)/4、tan(π/12)=2-√3 を代入して整理
π<2√6-4√3-2√2+8<2*2.45-4*1.732-2*1.414+8=3.144<3.15
(∵√6<2.45、√3>1.732、√2>1.414)

218よりは計算量少なめ
236132人目の素数さん:2008/11/21(金) 19:46:06
>>234
x±1、x^2-1
237132人目の素数さん:2008/11/21(金) 19:59:59
>>236
>x^2-1
条件(1)を満たしていない
238132人目の素数さん:2008/11/21(金) 20:54:31
pを3で割った時に1余る素数とする。このとき
a^2+ab+b^2=p
となる整数a,bが存在することを示せ
239132人目の素数さん:2008/11/21(金) 21:54:37
>>235
 2sinθ + tnaθ > 3θ,   0<θ<π/2 .
を Snell の式とか言うらしいよ。

http://science6.2ch.net/test/read.cgi/math/1179000000//591 , 565
不等式スレ3
240132人目の素数さん:2008/11/21(金) 22:12:14
>>235
平方根の近似値使わなくてもできる

>>235 から
4(2sin(π/12) + tan(π/12)) > π  …(1)

また、
22/7 - 4(2sin(π/12) + tan(π/12))
= (2/7) (-17 + 7√2 + 14√3 - 7√6)
= (2/7) (√2-1)^3 (2-√3)^2 (3√6-7)
= (2/7) (√2-1)^3 (2-√3)^2 (√54-√49)
> 0
∴ 22/7 > 4(2sin(π/12) + tan(π/12))  …(2)

3.15 > 22/7  …(3)
は明らか

(1)(2)(3) より π<3.15

ところで、いきなり相加相乗使ってるけど、
そのやりかた有名なの?
241132人目の素数さん:2008/11/22(土) 08:23:27
>>240
なるほど・・・
22/7が割と正確な円周率の近似であることをうまく利用するんですね

この方法は>>239にあるとおり、光の屈折に関するsnellの法則で有名なWillebrord Snellが
円周率の値を評価するときに使った方法です
3sinθ/(2+cosθ)<θ<(2sinθ+tanθ)/3の右側ですね

242132人目の素数さん:2008/11/22(土) 19:48:19
↑の略証
 1 - {(1-cos(x))/(2+cos(x))}^2 < 1 < {cos(x) + cos(x) + 1/(cos(x)^2)}/3,
を [0,θ] で積分する・・・・
243132人目の素数さん:2008/11/22(土) 22:53:55
>>185
これって、2008年度の京大乙6番のように三角関数表が与えられたら、こういう解答も成り立つ。
正45角形ならばS_45 = 45 tan 4°だが、0.0699 < tan 4° < 0.0700より、
\pi < S_45 < 45*0.0700=3.15より、円周率は3.15未満。
0.0697 < sin 4°< 0.0698より、\pi > s_90 = 45*sin 4°>45*0.0697=3.1365>3.1
http://hiw.oo.kawai-juku.ac.jp/nyushi/honshi/08/k01-23p/3.html
但し、三角関数表自体は切り捨てか切り上げか明言していないので、使うべき角度には注意が必要。
角度が小さ過ぎても、円周率には躙り寄れるが今度は誤差が大きいので問題あり。
244132人目の素数さん:2008/11/22(土) 23:39:37
なんだかんだで185は人気だなww
245132人目の素数さん:2008/11/23(日) 00:13:39
一辺が1の正四面体OABCにおいてOA、OB、OC上に点P、Q、Rが
四面体OPQRの体積が正四面体OABCの1/3になるように動く。
このとき三角形PQRの周および内部が通過する領域の体積を求めよ。
246132人目の素数さん:2008/11/23(日) 00:28:56
google入社試験のやつだろ。
247132人目の素数さん:2008/11/23(日) 04:33:12
>>241-242
サンクス
不等式スレも見てるのに見落としてた…
248132人目の素数さん:2008/11/23(日) 15:11:27
正の無理数αに対し、二つの数列
2α、4α、8α、16α、32α…(2^nα)
6α、12α、24α、48α、96α…(3*2^nα)
を考える。

このとき、いかなるαを考えたとしても、
これらの数の中に必ず、小数部分が1/4より大きくなるものがあることを示せ。

また、上記命題において1/4が最良であることも示せ
249132人目の素数さん:2008/11/23(日) 16:21:04
>>248
2進法で考えて、α の小数点以下(n+1)桁目が 1 だったら、
2^n*α は小数点以下1桁目が 1 だから、小数部分は 1/2 以上
だから、1/4 が最良ってのは変じゃないか?
250248:2008/11/23(日) 16:30:45
すまん、ちょっと考えなおしてくる
251249:2008/11/23(日) 19:14:14
1/4 を 3/4 にすれば問題が成立してると思う
252132人目の素数さん:2008/11/24(月) 01:07:35
f:N×N→Nが次の3条件を満たすとき、fをすべて求めよ
f(x,x)=x
f(x,y)=f(y,x)
(x+y)f(x,y)=yf(x,x+y)
253132人目の素数さん:2008/11/24(月) 01:17:20
>>248
αの2進小数表示を考える。
小数部分の数字の中に 1 が連続している場所が在れば
その部分 11 が小数点のすぐ右に来たときに2^n・αの
小数部分が3/4より大きく)になる。
また連続した場所が無ければ、小数点の右に 010が来たときに
3・2^nαの小数部分が3/4より大きくなる。
2進小数で
0.00.........(0がn桁).........00100.........(0がn+1桁).........00100.........(0がn+2桁).........00100(0が三桁)
というような数を考えると、nが大きいとき、>>248の系列の最大値は3/4に充分近い。
254132人目の素数さん:2008/11/25(火) 05:23:29
m,nは正の整数とする。x,yの方程式
mx^2-ny^2=1
が解をもつとき、この方程式は無数に多くの解を持つことを証明せよ。
255132人目の素数さん:2008/11/25(火) 07:00:36
どう見ても双曲線
(x,y)=((√m)/cost,tant/(√n))
256254:2008/11/25(火) 07:02:16
すまない、「整数解」が抜けていた。正しくは

m,nは正の整数とする。x,yの方程式
mx^2-ny^2=1
が整数解をもつとき、この方程式は無数に多くの整数解を持つことを証明せよ。
257132人目の素数さん:2008/11/25(火) 20:44:24
m=n=1の時点で有限個なんだが
258132人目の素数さん:2008/11/25(火) 20:49:06
ワラタ
259132人目の素数さん:2008/11/25(火) 21:06:14
連続で有界な定数でない実関数f,gが任意のx,yに対して
f(x+y)=f(x)f(y)-g(x)g(y)
g(x+y)=g(x)f(y)+f(x)g(y)
を満たすとき、f,gを求めよ。
260132人目の素数さん:2008/11/25(火) 21:13:48
Fラン用?
f(x)=cosx
g(x)=sinx
261259修正:2008/11/25(火) 21:39:00
連続で有界な定数でない実関数f,gが任意のx,yに対して
f(x+y)=f(x)f(y)-g(x)g(y)
g(x+y)=g(x)f(y)+f(x)g(y)
を満たすとき、f,gを『すべて』求めよ。


いや、っていうか何の条件もなかったら普通「すべて」だよね……
そう思ってた、俺がバカなのか……orz
だから、唯一性の証明がだなぁry
262132人目の素数さん:2008/11/25(火) 21:39:49
>>260
あと、それだけじゃないよ。唯一性とは言ったが、答えは無限にあるので……
と言ってもry
263132人目の素数さん:2008/11/25(火) 21:46:07
こんな有名問題出るわけねーだろカス
264132人目の素数さん:2008/11/25(火) 23:06:26
いやもう出るとか出ないとか気にしてる人いないと思う。
265132人目の素数さん:2008/11/25(火) 23:17:12
微分可能なやつだったら解けるが
266132人目の素数さん:2008/11/26(水) 00:59:25
>>260
これFラン用?Eランク大学の俺様には簡単すぎて欠伸が出るんだけど。
くらいの意味だと思っといたほうがw

>>263
微分可能って条件があるのだったら見たことあるけど、
それが無い奴はそんなに有名でも無いと思うけど。

まあ定義に従って微分を求めたら求められた気もしたけど。
267132人目の素数さん:2008/11/26(水) 10:36:03
f(x)を実数において定義され実数値をとる連続な関数とし、さらに
f(f(f(x)))-3f(f(x))+6f(x)= 4x+3
を満たすとする。

(1)実数a,bに対してf(a)=f(b)が成立するときa=bであることを示せ。
(2)f(x)は単調増加であることを示せ.
(3)f(x)を求めよ.
268132人目の素数さん:2008/11/26(水) 13:30:21
>>260
お願いだからギャグと言ってくれ
269132人目の素数さん:2008/11/26(水) 14:14:00
>>253
0.01011x11=1.00001.
270260:2008/11/26(水) 14:22:36
釣れたーーーーーーーーーー^^;
271132人目の素数さん:2008/11/26(水) 15:03:01
>>261
これ本当に解けるか?
272132人目の素数さん:2008/11/26(水) 18:08:34
うるさい。
273132人目の素数さん:2008/11/26(水) 18:40:21
x,yは正の整数、またdをx,yの最大公約数とする。
方程式:d^3+x+y^2=dxy
を満たすx,yをすべて求めよ。
274132人目の素数さん:2008/11/26(水) 20:25:58
275132人目の素数さん:2008/11/26(水) 21:00:10
>>261
なんかミスってた&ヘタクソなことしてた(´・ω・`)
ttp://www.csync.net/service/file/view.cgi?id=1227700775
276132人目の素数さん:2008/11/26(水) 21:18:36
文 科 
第 一 問(文理共通)

sin2009°の小数第一位から少数第五位までのそれぞれの数を,
a,b,c,d,eとして,このとき,
    f(x) = ax^3+bx^2+cx+d
とする.
(1) sin2009°とsin1877°の大小を比較せよ.
(1) a,b,c,d,e,f(e)のそれぞれの値を求めよ.
(2) f(x)の極大値と極小値を求めよ.

(1)は東大の易化にあわせたつもりだが簡単すぎる.
ちなみに1877年は東大設立の年.
sin1877°> sin2009°になることはどうみても明らかだけども,
懐古的な(といっても1877年だと古すぎるが)教授なら,
2009年よりも1877年のほうが東大は輝いていたに違いない―,なんて言うかもしれないと思って.
277132人目の素数さん:2008/11/26(水) 21:39:44
(2)はかなり面倒じゃない?
sin 29°の値なんて手計算させてどうすんの?
sin 2010°なら意味分かるけども。

三倍角の公式を使って三乗根の値を概算、なんてやってたら
(最近の学生はゆとり教育の結果計算力がなくなっているとかそういうことではなくて)
三十分じゃ全然時間が足りないと思うけど。
278132人目の素数さん:2008/11/26(水) 22:17:29
>>276
電卓があれば解ける問題って……いくらなんでもあり得ないのでは?
279132人目の素数さん:2008/11/26(水) 22:27:19
>>261-262
 Fラン用?
 f(x) = cos(ax),
 g(x) = sin(ax),
280132人目の素数さん:2008/11/26(水) 22:50:11
もうええからそれは
281132人目の素数さん:2008/11/27(木) 02:45:44
>>275
一生懸命タイプして完成して嬉しい気持ちでうpしたんだろうなと思ったら萌えた
282132人目の素数さん:2008/11/27(木) 18:33:56
n^n + 2 (n∈N)が素数になるような n が無数に存在することを証明せよ.
283132人目の素数さん:2008/11/27(木) 20:16:19
1877 = 1800+77 より sin1877゜= sin77゜
2009 = 1800+180+29 より sin2009゜= -sin29゜

符号を見て分かる通り、sin1877゜ > sin2009゜
・・・・ダメ?
284132人目の素数さん:2008/11/27(木) 20:26:54
>>267
(1)f(a)=f(b)のときf(f(a))=f(f(b)),f(f(f(a)))=f(f(f(b)))なので
4a+3=4b+3
ゆえにa=b

(2)(1)よりf(x)は単射の連続関数なので
単調増加または単調減少
単調減少と仮定すると
f(f(x))は単調増加、f(f(f(x)))は単調減少なので
f(f(f(x)))-3f(f(x))+6f(x)は単調減少
だが右辺の4x+3は単調増加で矛盾する。
285132人目の素数さん:2008/11/27(木) 22:33:03
>>283
elegant
286132人目の素数さん:2008/11/28(金) 02:12:30
eelegantか?普通じゃないの
287132人目の素数さん:2008/11/28(金) 03:10:33
>>256

1つの整数解を (x_1, y_1) とし、
 α = x_1・√m - y_1・√n,
 β = x_1・√m + y_1・√n,
とおくと
 αβ = m(x_1)^2 - n(y_1)^2 = 1,
また、α、βは奇数乗しても
 α^(2k+1) = x_(2k+1)・√m - y_(2k+1)・√n,
 β^(2k+1) = x_(2k+1)・√m + y_(2k+1)・√n,
の形を保つ。 そして漸化式
 x_(2k+1) = {m(x_1)^2 + n(y_1)^2}x_(2k-1) + {2n(x_1)(y_1)}y_(2k-1),
 y_(2k+1) = {2m(x_1)(y_1)}x_(2k-1) + {m(x_1)^2 + n(y_1)^2}y_(2k-1),
から、 x_( ), y_( ) はすべて整数となる。
288132人目の素数さん:2008/11/28(金) 11:15:15
>>267
cを任意の実数とし数列x[n]を漸化式x[1]=c, x[n+1]=f(x[n])で定める。
与方程式から

x[n+3]-3x[n+2]+6x[n+1]=4x[n]+3 (n=1,2,...)となる。

階差数列をy[n]=x[n+1]-x[n]とおけば
y[n+3]-1=-8(y[n]-1) となるので

y[3n+1]=(-8)^n*(y[1]-1)+1

を得る。ゆえにy[1]≠1と仮定すると十分大きなnに対してy[u]>0,y[v]<0となるような番号u,vが
それぞれ存在する。
ゆえにf(x[u])>u,f(x[v])<vとなるような実数x[u],x[v]が存在するが、f(x)は連続なので
中間値の定理からf(w)=wとなる実数xが存在する。
これを与方程式に代入すればw-3w+6w=4w+3⇔0=3となり矛盾する。

従ってy[1]-1=0でなければならず、すなわちx[2]=x[1]+1,つまりf(c)=c+1である。
cは任意であったから任意の実数xに対して
f(x)=x+1でなければならない。
逆にこれは与えられた条件を満たす。ゆえにf(x)=x+1
289132人目の素数さん:2008/11/28(金) 15:42:04
lim{n->∞}{( 1 + 1/(n*(n-1)) )^n}を求めよ。

ネイピア数e = lim{n->∞}{(1 + 1/n)^n}(表記の参考までに)
290132人目の素数さん:2008/11/28(金) 22:02:48
任意の実数a, bに対して
 F(2a) + F(2b) = 2F(a+b)F(a-b)
を満たし、かつ定数関数ではない関数F(x)がある。

F(p)=F(q)を満たす実数p, qに対して、F(p+q) と F(p-q)の少なくとも一方は1に等しいことを示せ。
291132人目の素数さん:2008/11/28(金) 22:28:20
>>290
a=b=0とすると
f(0)+f(0)=2f(0)^2⇔f(0)=0,1となるがf(0)=0と仮定すると
b=aとして,f(2a)=0が任意のaで成立し仮定に矛盾するからf(0)=1

さらにb=0としてf(2a)+1=2f(a)^2 ∀a ・・・(1)

a=(p+q)/2,b=(p-q)/2として
f(p+q)+f(p-q)=2f(p)f(q) …(2)

a=p,qとして
2f(p+q)f(p-q)=f(2p)+f(2q) ・・・(3)

(1)よりf(p)=f(q)のときf(2p)=f(2q) ・・・(4)

ゆえに(2)(3))(4)より
(1-f(p+q))(1-f(p-q))
=1-f(p+q)-f(p-q)+f(p+q)f(p-q)
=1-2f(p)f(q)+{f(2p)+f(2q)}/2
=1-2f(p)^2+f(2p)=0 (∵(1))

よって示せた。
292132人目の素数さん:2008/11/29(土) 04:25:44
>>290

b=a を代入すると、2F(2a){1-F(0)} = 0,
題意により F(2a)≡0 ではないから、F(0) = 1.
 |F(1)| < 1 のとき F(x) = cos(ax), ただし a = arccos{F(1)},
 F(1) > 1 のとき F(x) = cosh(a'x), ただし a' = arccosh{F(1)} = log{F(1)+√[F(1)^2 -1]},
293132人目の素数さん:2008/11/29(土) 16:21:57
円周率πは無理数であることが知られている。
πに1/mπ(m:0以外の実数)なる数以外の数を掛けたとき、その値が0でない有理数となることはあるか。
294132人目の素数さん:2008/11/29(土) 16:50:44
1/mπ(m:0以外の実数)は0以外のすべての実数を取りうるから
これ以外の実数はない。
よってない。
295132人目の素数さん:2008/11/29(土) 18:06:26
>>289
 n(n-1) = N とおくと
 N→∞, (n→∞)
(与式) = { (1 + 1/N)^N }^(1/(n-1)) → e^0 = 1. (n→∞)
296132人目の素数さん:2008/11/29(土) 20:10:03
n^2009の上2009桁がすべて1であるような正の整数nが存在することを証明せよ。
297132人目の素数さん:2008/12/01(月) 01:40:47
∫[x=1,0]x^(p-1)*(1-x)^(q-1)dxが収束するp,qの範囲を求めよ
298132人目の素数さん:2008/12/01(月) 01:47:39
>>297
そんなベータ関数の有名問題が出ると思ってんの?バカなの?
299132人目の素数さん:2008/12/02(火) 12:44:50
>>296
各桁がすべて1の2009桁の整数をa=111...11
とおく。
nが題意を満たす条件は
a*10^k≦n<(a+1)*10^k
を満たす0以上の整数kが存在することである。

a*10^k≦n<(a+1)*10^k
⇔loga+k≦logn<log(a+1)+k 

数列x[n]=lognについて考えると
x[n]→∞で、x[n+1]-x[n]=log(1+1/n)→0なので
x[n]の階差はいくらでも小さくなる。区間[loga+k,log(a+1)+k)の長さはlog(a+1)-loga=log(1+1/a)は0より大きい定数だから
k,nが十分大きければ
x[n]=lognが区間[loga+k,log(a+1)+k)に属するような正の整数nが存在する。

よって題意は示された。

300132人目の素数さん:2008/12/02(火) 17:31:16
>>299
nじゃなくてn^2009なんだが
301132人目の素数さん:2008/12/02(火) 23:18:35
∫[x=2π,0]√(2-2cost)dt
において
x=costと置換すると
積分区間は[x=1,1]となるが
これが0にならないとことを示せ
302132人目の素数さん:2008/12/02(火) 23:22:40
>>301
そりゃ、ルートがついたもん積分したら、中々0にゃならんだろ
303132人目の素数さん:2008/12/02(火) 23:45:15
>x=costと置換すると
>積分区間は[x=1,1]となるが
こういう間の抜けたことは東大入試の問題文には書かないかと
304132人目の素数さん:2008/12/02(火) 23:47:11
>>301
狙いは分かるが...
305132人目の素数さん:2008/12/02(火) 23:50:06
そういう盲点というか受験生の理解不足になりがちなポイントを、
うまく問題の中に潜ませるのがうまい問題だな。
突きたいポイントをずばり問題にしてしまったのでは駄作。

ま、俺には作れんが。
306132人目の素数さん:2008/12/03(水) 00:28:07
http://science6.2ch.net/test/read.cgi/math/1219648297/
京都大学入試作問者になったつもりのスレ@

の308で∫[x=0,1]√(2-2cos(2πx))dxが出てくる悪寒
・・・でもそんな置換はしないか
307132人目の素数さん:2008/12/03(水) 00:31:27
a,bを互いに素な自然数とし、f(1)=a,f(2)=b,f(n+2)=f(n+1)+f(n)による数列f(n)を考える。
f(n)が全て非素数になるa,bの組を一つ求めよ。存在しないのならその事を示せ
308132人目の素数さん:2008/12/03(水) 00:41:09
a,bを互いに素な自然数とし、f(1)=a,f(2)=b,f(n+2)=f(n+1)+f(n)による数列f(n)を考える。
いかなるa,bを選んでも、f(n)が合成数になるような無数に多くのnが存在することを示せ
309132人目の素数さん:2008/12/03(水) 01:26:56
>>308
mod2で考えれば 000… または …110110… だから,偶数の項は無限に存在する。
f(n)は単調増加だから4以上の偶数が無限に存在することになる。
310132人目の素数さん:2008/12/03(水) 21:37:11
>>301
変数変換したら被積分関数が閉区間[1,1]で存在しないことを
証明すればいいんだろ
311132人目の素数さん:2008/12/03(水) 22:03:41
312132人目の素数さん:2008/12/03(水) 23:07:08
>>306
半角の公式
313132人目の素数さん:2008/12/15(月) 04:36:01
1からnまでの数字が1つずつ書かれたn枚のカードがある。この中から1枚を引き、
出たカードの数字をX_1とする。さらに、カードをもとに戻して再び1枚を引き、
出たカードの数字をX_2とする。X_1, X_2のうち、小さくない方をXとする。次の問いに答えよ。
(1) Xの期待値Eを求めよ。
(2) kを自然数として、X≧kとなる確率をp_k、X≦kとなる確率をq_kとおく。
p_k≧1/2かつq_k≧1/2となるようなkの値をmとするとき、n=100に対するmの値を求めよ。
(3) lim[n→∞]E/mを求めよ。
314132人目の素数さん:2008/12/15(月) 04:44:24
nを自然数とする。2n桁の自然数で、上位n桁の和と下位n桁の和が等しいとき、
この自然数を「均衡数」と呼ぶことにする。
たとえば、1634は1+6=3+4により均衡数であるが、123401は1+2+3≠ 4+0+1により均衡数ではない。
(1) 0, 1, 2, 3, 4の5個の数字を用いて作られる4桁の均衡数の総数は70個であることを示せ。
(2) kを9以下の自然数として、0からkまでのk+1個の数字を用いて作られる4桁の均衡数の総数をkで表せ。
315132人目の素数さん:2008/12/15(月) 04:50:25
(1) √2>1.4を示せ。また、(1+√2)^5>99を示せ。
(2) ∫[0, π/2](sin 2x)/(1+sin^2 x)dx と ∫[0, π/2](sin x)/(1+sin^2 x)dxの大小を比較せよ。
316132人目の素数さん:2008/12/15(月) 07:14:48
ひとつの頂点に集まる面は3つ以上ある。
ひとつの頂点に集まる頂角の合計は360度未満である。
オイラーの定理V−E+F=2が成り立つ。
多面体の以上の性質を利用して、正多面体は正四面体、正六面体、正八面体、正十二面体、正二十面体の5種類しかないことを示せ。
317132人目の素数さん:2008/12/15(月) 21:38:14
sinθとcosθを用いてπを表せ。
318132人目の素数さん:2008/12/17(水) 00:26:56
>>315 (1)
問題がおかしくありませんか?(1+sqrt{2})^5 = 82.01...くらいだと思いますが。
319132人目の素数さん:2008/12/17(水) 00:53:24
>>318
ほんとだorz
書き間違えてました。
(1+√2)^5<99を示せ。
でした。ごめんなさい。
320132人目の素数さん:2008/12/17(水) 10:07:18
>>317
π + 0sinθ + 0cosθ
π(sin^2θ + cos^2θ)
321132人目の素数さん:2008/12/17(水) 12:05:06
>>316何年か前に海城高校で類題が出てたはず
322132人目の素数さん:2008/12/17(水) 12:12:15
>>320
π使ってるやんwwwwww
323132人目の素数さん:2008/12/17(水) 18:26:38
180 (sinθ)’/cosθ

ただし θ は度数法
324132人目の素数さん:2008/12/18(木) 02:31:50
>>315 (1) & >>319
 2 > 1.96 = 1.4^2,
 a_n = (1+√2)^n + (1-√2)^n,
とおくと
 a_n = 2*a_(n-1) + a_(n-2), a_0 = a_1 = 2,
 a_n - 1 < (1+√2)^n < a_n + 1,
を満たす。
 a_5 = 82 ゆえ、81 < (1+√2)^5 < 83,

>>315 (2)
 ∫[0,π/2] sin(2x)/{1+sin(x)^2} dx = ∫[0,π/2] 2sin(x)cos(x)/{1+sin(x)^2} dx
 = [ log{1+sin(x)^2} ](x=0,π/2)
 = log(2)
 = 0.69314718055994530941723212145818

 cos(x) = z とおくと、
 ∫[0,π/2] sin(x)/{1+sin(x)^2} dx = ∫[0,1] 1/(2-z^2) dz
 = (1/√8)∫[0,1] {1/(√2 -z) + 1/(√2 +z)} dz
 = (1/√8) [ log{(√2 +z)/(√2 -z)} ](z=0,1)
 = (1/√2) log(√2 +1)/(√2 -1)
 = 0.62322524014023051339402008025057

>>316
各面は正m角形、
1つの頂点に集まる面の数をn≧3,
とすると、
 mF = 2E = nV より V-E+F= (2/n -1 +2/m)E,
 {(m-2)/m}π*n < 2π より 2/m -1 +2/n > 0.
325132人目の素数さん:2008/12/19(金) 00:56:44
nを2以上の自然数とする。1/nと1/(n+1)が、10進数表記でともに有限小数になるnをすべて求めよ。

簡単かな。
326132人目の素数さん:2008/12/19(金) 01:48:31
受験生によって差が出そうな問題だ。4の倍数全て。
327132人目の素数さん:2008/12/19(金) 02:01:31
さっそく差が出たな
328132人目の素数さん:2008/12/19(金) 02:18:26
n=8でもう違ってる。簡単に考え過ぎたな
329132人目の素数さん:2008/12/19(金) 02:26:54
また頭の中で考えただけだけどn=(5^m-1)/2, (1/2)*(5^m-1)-1 (m: 自然数)
330132人目の素数さん:2008/12/19(金) 02:29:39
m=1だとn=1(<2)になるけどこういうのってアウトなんだろうな
331132人目の素数さん:2008/12/19(金) 02:46:50
n=5^j、n+1=2^(4k)の形になるもの(k、jは正整数)
またはn=2^(4k-2)、n+1=5^jの形になるもの(k、jは正整数)

酔った頭じゃこれ以上簡単にできない
332132人目の素数さん:2008/12/19(金) 19:12:24
kを0または自然数として
n=10k+4

どうだろう
333132人目の素数さん:2008/12/19(金) 20:31:29
とりあえず1/14を計算してみれば良いと思うよ。

2の冪と5の冪で隣り合うようなものの組を全て求めよっていう問題だよね。
334132人目の素数さん:2008/12/19(金) 20:54:58
>>324
(1) をつかって、(2) を示すんじゃないの?
335132人目の素数さん:2008/12/19(金) 21:39:10
1/n が10進小数で有限小数になる
⇔(ある自然数 N 、 k を用いて) 1/n = N/10^k と表わせる
⇔ nN = 2^k・5^k と表わせる
⇔ n の素因数は 2 か 5 のみ

よって n と n + 1 がともに 2 か 5 のみとなるような組を求めれば良いが、
片方が 2・5 = 10 の倍数ならば不適となることが直ぐに分かるので
2 の冪と 5 の冪で差 1 になるようなものの組 (2^n, 5^m) を求めれば良い。
a^n - b^n は a - b で割り切れ、また n が奇数のとき a^n + b^n は a + b で割り切れることに注意。

2^n = 5^m + 1 かつ m ≧ 1 のとき、
mod. 5 で両辺を比較して n が 4 の倍数となることが分かる。文字をおきなおして
2^(4n) - 1 = 5^m つまり 16^n - 1 = 5^m となれば良いが、左辺は 15 の倍数なので
この式を満たす n, m は存在しない。

2^n = 5^m - 1 のとき、
右辺が 24 = 5^2 - 1 で割り切れてはいけないので m は奇数。(*)
2^n + 1 が 2 + 1 で割り切れてはいけないので n は偶数。
2^(2k) = 4^k = 5^m - 1 = 4(1 + 5 + 5^2 + ......... + 5^(m-1))
つまり 4^(k-1) = 1 + 5 + 5^2 + ......... + 5^(m-1) となる。
mod. 4 で両辺を比較すると k > 1 のとき
0 ≡ m (mod. 4 )となる。従って m は 4 の倍数。
これは(*)に反するので k = 1、m = 1 が分かる。

したがって>>325の解は n = 4、n + 1 = 5 のみ。
336132人目の素数さん:2008/12/19(金) 21:41:37
あ、訂正

よって n と n + 1 【の素因数】がともに 2 か 5 のみとなるような組を求めれば良いが、

それから 2^n = 5^m + 1 かつ m = 0 の場合忘れてた。
(n , n + 1) = (1, 2)も解で、この二つか。
337132人目の素数さん:2008/12/19(金) 23:05:19
nは2以上の整数す
338132人目の素数さん:2008/12/20(土) 00:50:33
>>334
>>313-315の出題者ですけど、
当然、そういう意図の問題です。
339132人目の素数さん:2008/12/20(土) 13:24:48
340324:2008/12/21(日) 02:45:27
>>334,338
(1) から
 (1+√2)^5 < 89.6 = 64*1.4 < 64√2 = 2^6.5
  1+√2 < 2^1.3
よって
 (1/√2)log(√2 +1) < (1.3/√2)log(2) < (1.3/1.4)log(2)
かな?
341132人目の素数さん:2008/12/23(火) 13:39:50
Nメートルの紐を使ってエンブレムをつくりたい、正し、紐は2本に切って
それぞれからある形をつくる。その形の条件として、紐をA、Bとすると。
どちらかは円でなければならない、またもう片方は多角形でなければならない。
この多角形と円を組み合わせてエンブレムをつくるわけだが、どちらかが
片方に内接または外接してないといけない。このときエンブレムを構成する
円と多角形の面積の和の最大値を求めよ。

友達の東大生・東工生正答率30人中1人。
342132人目の素数さん:2008/12/23(火) 16:02:47
f(x)=an x^n + a(n-1) x^(n-1) + ...+ a1 x + a0
ただし a0,a1,a2,...,an は実定数で an≠0 とする.
また M=max[0≦k≦n] | ak | ( | a0 |,| a1 |,| a2 | ,...,| an | の最大値) とする.

このとき,次の性質が成立する 定数 R の例を1つ M を使って表せ.

| x | ≧ R を満たすすべての実数 x に対して f(x) ≠ 0
343132人目の素数さん:2008/12/24(水) 12:50:31
344132人目の素数さん:2008/12/24(水) 13:20:39
>>342
f(x)=xとする。
M=1
|x|>1、つまり-1<x<1の範囲でf(x)≠0であることを示せばよいが
x=0のときf(0)=0よりアウト

f(x)=x^2+x+a0とする
M=1
f(x)=0とおくと、解は(-1±√(1-4a0))/2
これが-1<x<1の外側にあればよい
そのとき|x|≧Rを満たすすべてのxについてf(x)≠0が成り立つ
345132人目の素数さん:2008/12/24(水) 13:21:35
>>344
間違えた

M=a0だね。
346132人目の素数さん:2008/12/24(水) 13:40:58
>>344
ギャグで言ってる?
347132人目の素数さん:2008/12/24(水) 16:21:14
ん?おかしかったかな

じゃあ別の解法で

|x|≧Rを満たすxの一つをφとする。つまり-R≦φ≦R
f(φ)=0であると仮定する
このφは同時にM=φを満たすから適当な数Hn(0≦Hn≦φ)を用いて
f(x)=Σ[n,k=0](φ-Hn)x^nとかける。ただし1つ以上xの項の係数はφである。
しかし、φ-Hn=0がありうることより、問題のanについてan=0が起こりうる。
これはan≠0の条件に反する。仮定が誤っていたことを意味する。
よってどんなφであっても|φ|≧Rを満たすφに対してf(φ)≠0は成立する。
(Q.E.D)
348132人目の素数さん:2008/12/24(水) 16:32:47
>|x|≧Rを満たすxの一つをφとする。つまり-R≦φ≦R
この時点で矛盾している。

>f(φ)=0であると仮定する
>このφは同時にM=φを満たすから
満たすとは限らない。

>適当な数Hn(0≦Hn≦φ)を用いて
いつの間にかφ≧0であることが仮定されている。意味不明。

>f(x)=Σ[n,k=0](φ-Hn)x^nとかける。
書けない。Σの中身にkが無いから、f(x)=(n+1)(φ−Hn)x^nになってしまう。

>ただし1つ以上xの項の係数はφである。
日本語になっていない。

>よってどんなφであっても|φ|≧Rを満たすφに対してf(φ)≠0は成立する。
問題の要求に答えていない。「|φ|≧Rを満たすφに対してf(φ)≠0が成り立つ」
ようなRを、Mを用いて構成せよと聞かれているのに、それをしていない。
349132人目の素数さん:2008/12/24(水) 16:35:27
ばれたか
350132人目の素数さん:2008/12/24(水) 17:26:49
ばれたか千里
351132人目の素数さん:2008/12/24(水) 19:02:42
>>347
頭悪すぎてひいてしまった
352132人目の素数さん:2008/12/24(水) 19:30:13
じゃあ回答しろよ、カス
353132人目の素数さん:2008/12/24(水) 19:56:48
M って最小値じゃなくて最大値?
書き間違いじゃない?

f(x) = εx^n + M のとき
f(x) = 0 の解 x に対して
|x| = (M/ε)^(1/n) → ∞ (ε→ 0 )

特に (M/ε^)(1/n) > R(M) ⇔ M/R(M)^n > ε だから
>>342のような性質が成立するような M の関数 R(M) は存在しない。
354132人目の素数さん:2008/12/24(水) 22:00:07
>>353
確かに。
多分、an=1 もしくは M=max[0≦k≦n] ( | ak | /| an | ) の間違いでしょうな。
もしそうなら、R=2M とかが答。
355132人目の素数さん:2008/12/24(水) 23:20:53
なんか頭悪い奴湧いてるみたいだから俺の高校の実力試験問題やってみろ

平面をn本の直線でα個の領域に分けることを考える。
直線はどの2本を選んでも完全に重なることはないとする。
αのとりうる最大・最小の値をそれぞれ求めよ。
356132人目の素数さん:2008/12/24(水) 23:36:23
最小は n+1 、最大は 1 + n(n+1)/2 じゃないかな。
理由はめんどいから書かないけど。
357132人目の素数さん:2008/12/24(水) 23:40:42
>>355
お前が一番頭悪そうw
358132人目の素数さん:2008/12/24(水) 23:47:28
>>356
なんで最大の領域の数が分数になるんだよwww
領域の数は普通整数だろうが
n(n+1)/2が整数になるときって条件つけとけ
359132人目の素数さん:2008/12/25(木) 00:38:27
>>358
君、面白いね
360132人目の素数さん:2008/12/25(木) 00:49:52
>>359
ありがとう
361132人目の素数さん:2008/12/25(木) 01:58:48
>>355
これの解法だれか教えて
最大の数がわからん
362カツオ:2008/12/25(木) 01:59:17
>>356連続してるから2で割れるんじゃあないの?
363132人目の素数さん:2008/12/25(木) 02:01:30
>>358はn本と言われてnを有理数だと思ってしまうのだろうか
364132人目の素数さん:2008/12/25(木) 02:12:06
>>362
n(n+1)が2で割れるとしたらn(n+1)=2kとおける
展開してn^2+n-2k=0
n=(-1±√(1+8k))/2っていう風になるから割り切れなくね?
365132人目の素数さん:2008/12/25(木) 02:19:29
nもn+1も連続する自然数なのでどちらかは偶数。よってn*(n+1)は2を因数に含む。
nC2だってn(n-1)/2なのに鈍すぎバカすぎ。
しかも>>364ではn(n+1)/2=kとしてn(n+1)/2が全整数を取るかのような妄想までしてて悲惨
366132人目の素数さん:2008/12/25(木) 02:28:31
>>365
よくわかんない・・・
nC2は整数なの?
367132人目の素数さん:2008/12/25(木) 02:39:24
>n=(-1±√(1+8k))/2っていう風になるから割り切れなくね?
kは特殊な値しか取らない。具体的には、(-1+√(1+8k))/2が
整数になるような値しか取らない。

お馬鹿の366のための解説:
nが偶数ならn=2mと表せてn(n+1)2/=m(2m+1)
nが奇数ならn=2m+1と表せてn(n+1)/2=(2m+1)(m+1)
よって必ずn(n+1)/2は整数。
368132人目の素数さん:2008/12/25(木) 02:41:11
>>366
n個のおかしから2個を選ぶ組み合わせの総数は?
n個のうんちから2個を選ぶ組み合わせの総数は?
369132人目の素数さん:2008/12/25(木) 02:42:15
>>366
nが整数ならばn(n+1)は偶数
これがどうしても分からないなら、nが偶数のときと奇数のときで場合分けしてみて
君の発言が釣りであることを祈るよ
370カツオ:2008/12/25(木) 02:48:32
ああ。nは正の整数だからだよ!その解だとnは無理数もあり得る感じだけどn本て明らか正の整数じゃあない?
371132人目の素数さん:2008/12/25(木) 02:52:44
>>370
「nが整数のときn(n+1)/2は全ての整数をとるわけじゃない」
y=x(x+1)/2だとxy平面上で放物線をなし、xの値に応じてyは全実数をとれる。
君は県立高校の1年生とかそんなところかな。よく他人の書き込みを自信満々にバカにできたね……
372( °┌・・ °) ホジホジ:2008/12/25(木) 02:55:52
納k=1,n]a[k]=nであり
f(x)=納k=1,n]a[k]coskx
としたとき
常にf(x)≧-1
となるような実数a[k]が任意の自然数nに対して存在することを示せ
373132人目の素数さん:2008/12/25(木) 02:58:34
何だ、カツオはおばかな366トは別か
374132人目の素数さん:2008/12/25(木) 03:03:08
>>361 3本くらいで実験してみな。ごく普通の前科式の問題だよ。
375カツオ:2008/12/25(木) 03:05:13
>>371いや馬鹿にしたんではなくて気付いたこと書いただけかなぁ。実際そんな頭良くないし間違ってたら頭いい人が訂正してくれるし…。あとなんかそれ言いたいことが違う気がする…
376132人目の素数さん:2008/12/25(木) 03:05:54
>>375
勘違いだ、ごめん。358と間違えたんだ。
377カツオ:2008/12/25(木) 03:08:53
いえいえ!大丈夫です!なんかビックリしてしまった(笑)
378132人目の素数さん:2008/12/25(木) 03:14:09
>>367
これnが偶数のときの説明がおかしい
m=1/2としたらそのときn=mになるけどm(2m+1)は分数だもの。
よって命題は成り立たない
379132人目の素数さん:2008/12/25(木) 03:16:19
>m=1/2としたら
しません
380132人目の素数さん:2008/12/25(木) 03:19:09
なんで?
mには制約ないでしょ

てかそのときnはそもそも整数じゃなくなるからn=2m自体が成り立たないよ。
仮定がおかしい
381132人目の素数さん:2008/12/25(木) 03:22:21
>mには制約ないでしょ
あります
>てかそのときnはそもそも整数じゃなくなる
そのときの想定は無用です
>仮定がおかしい
おかしくありません。n本の線を引くとあるのだから、nが自然数 or 0だと想定できます
382132人目の素数さん:2008/12/25(木) 03:22:52
ここバカばっかじゃん
383132人目の素数さん:2008/12/25(木) 03:28:30
言語能力0
数学能力0

他者罵倒力 ∞ - 計測不能
傲岸不遜力 ∞ - 計測不能
384132人目の素数さん:2008/12/25(木) 12:10:05
358はまじで中学からやりなおしたほうがいい。
385132人目の素数さん:2008/12/25(木) 12:36:49
確かに。n(n+1)が偶数でないことを知らないなんて。
386132人目の素数さん:2008/12/25(木) 12:42:16
知らなくても少し考えれば分かることなのに
387132人目の素数さん:2008/12/25(木) 12:59:21
釣られすぎ
388132人目の素数さん:2008/12/25(木) 13:01:37
釣れた
389132人目の素数さん:2008/12/25(木) 13:02:22
俺が主犯だけどwwwお前らつられすぎてて吹いたwwwww

東北医ですサーセンwwwwwww
390132人目の素数さん:2008/12/25(木) 13:32:48
東北医(笑)
391132人目の素数さん:2008/12/25(木) 13:55:21
おっスレが伸びてるな、と思ったら
基地外が乱入してたのね
392132人目の素数さん:2008/12/25(木) 17:20:54
>>389
ほんとに東北医なの?
fusianasanって名前欄に入れて書き込んでみなよ
393132人目の素数さん:2008/12/25(木) 17:31:09
いや、だって今地元に帰ってるし
394132人目の素数さん:2008/12/25(木) 17:39:37
じゃあ、難しい問題解いてみてよ
>>372とかさ
395132人目の素数さん:2008/12/25(木) 17:46:43
今考えてるけどよく分からん
フーリエ級数みたいだね
396132人目の素数さん:2008/12/25(木) 18:06:21
>>372
納k=1,n]a[k]=n
これってa[k]=1にしかならないと思うんだが・・・
397132人目の素数さん:2008/12/25(木) 18:11:09
おいおい、もう釣りはいいから
cosx+cos2x≧-1
とか常には成り立たないだろ
もしかしてa[k]を自然数と勘違いしてるのか?
398132人目の素数さん:2008/12/25(木) 20:57:36
>>396-397
 nを固定して考えれ。

n=2 の場合
 (左辺) + 1 = a[1]cos(x) + a[2]cos(2x) +1 = 2a[2]cos(x)^2 +a[1]cos(x) + (1-a[2]),
判別式は
 D = a[1]^2 -8a[2](1-a[2]) = a[1]^2 + 8(a[2] -1/2)^2 -2,  
 D=0 は楕円で、原点でa[1]軸に接し、(4/3, 2/3) で a[1} + a[2] = 2 に接する。
∴ a[1]=4/3, a[2]=2/3,
このとき
 (左辺) = (4/3)cos(x) + (2/3)cos(2x) = (1/3){2cos(x) +1}^2 -1 ≧ -1,
399132人目の素数さん:2008/12/25(木) 21:06:51
>>396-397
 nを固定して考えれ。

n=2 の場合
 f(x) + 1 = a[1]cos(x) + a[2]cos(2x) +1 = 2a[2]cos(x)^2 +a[1]cos(x) + (1-a[2]),
 = 2a[2]{cos(x) + a[1]/4a[2]}^2 - D/(8a[2]),
これが常に非負となるから、D≦0,
 D = a[1]^2 -8a[2](1-a[2]) = a[1]^2 + 8(a[2] -1/2)^2 -2,  
 D=0 は楕円で、原点でa[1]軸に接し、(4/3, 2/3) で a[1} + a[2] = 2 に接する。
∴ a[1]=4/3, a[2]=2/3,
このとき
 f(x) = (4/3)cos(x) + (2/3)cos(2x) = (1/3){2cos(x) +1}^2 -1 ≧ -1,
400132人目の素数さん:2008/12/25(木) 22:39:03
>>372

n=3 のときは
 a[1]=3/2, a[2]=1, a[3]=1/2,
 f(x) = (3/2)cos(x) + cos(2x) + (1/2)cos(3x) = 2{1+cos(x)}cos(x)^2 -1 ≧ -1.
かな。
401132人目の素数さん:2008/12/26(金) 00:07:30
a[k]を具体的に求める方法ってあるの?
n=2なら偶然見つけられたけどn=3以上になると全然見つけられない
402132人目の素数さん:2008/12/26(金) 00:55:49
n=k(k≧3)で固定すれば容易
403132人目の素数さん:2008/12/26(金) 10:17:31
>>372

任意の実数α(≧0)ついて、
納k=1,n]a[k]=α  f(x)≧-α/n
を満たすa[k]があることを帰納法で示す。

n=1は省略
404403:2008/12/26(金) 10:20:57
すまぬ。出来たつもりで書こうとしたら出来てなかったorz
405132人目の素数さん:2008/12/26(金) 10:29:31
期待してるよー
406132人目の素数さん:2008/12/26(金) 15:18:34
n≧kをみたす任意の整数nに対して
n<m^2<(2009/2008)*n
となるような整数mが存在するような正の整数kのうち最小のものを求めよ。
407132人目の素数さん:2008/12/28(日) 08:55:36
入試問題の多くはソ連科学アカデミーの天才養成よう難問集がネタ本だよ。
408132人目の素数さん:2008/12/28(日) 09:08:46
n<m^2<(2009/2008)*n
(2009/2008)*n-n>1
a^x-x>1
f=e^xloga-x
df/dx=logae^xloga-1=0
e^xloga=1/loga
xloga=-logloga
x=-logloga/loga=n=16.950
409132人目の素数さん:2008/12/28(日) 09:25:07
「アメリカのマサチューセッツ工科大学(MIT)コンコースプログラムに使われた問題が載
っており、その出典はユーリがロシア(当時はソ連)のモスクワ大学助教授時代に国内で行わ
れていた「オリンピヤード(数学コンテスト)」や大学入試の問題、難解でひねりの利いたク
イズなどです。ちなみに題名の「ミンスク」はベラルーシ共和国(旧ソ連白ロシア共和国)の
首都です。」という本です。

「ペレリマン,ヤコフ・イシドロヴィチ〈Перелъман,Яков Исидорович〉」
410132人目の素数さん:2008/12/28(日) 09:26:15
ペレリマン1882.10.17-1942.3.16 『遊びの数学』(藤川健治訳)現代教養文庫958, 社会思想社 (1978) Yakov Isidorovich Perel’man
ペレリマン 『数のはなし』(金光不二夫訳)東京図書(1987) Ya.I.Perel’man
ペレリマン 『代数のはなし』(山崎昇訳)東京図書(1987) Ya.I.Perel’man
ペレリマン 『幾何のはなし』(金光不二夫訳)東京図書(1987) Ya.I.Perel’man
ペレリマン 『数学のはなし』(三橋重男訳)東京図書(1987) Ya.I.Perel’man「生きた数学」「おもしろい数学」などのタイトルで出版されていたもの
411132人目の素数さん:2008/12/28(日) 09:27:37
412132人目の素数さん:2008/12/28(日) 09:37:22
入学までにこれくらいは目を通してね
春休みとかに

No10131 ソ連教育科学アカデミー版 基礎数学(全6巻揃)、東京図書、1966-1967年、15,750円
1、数と集合1 第1部:記数法の起源/第2部:集合、群、環、体、数の体系 バシュマーコヴァ・ユシケーヴィチ/プロスクリャーコフ 1967 4刷
2、数と集合2 第3部:数論/第4部:暗算と筆算、計算の補助手段 ヒンチン/ブラジス 1966 2刷
3、代数1 第1部:ベクトル空間と一次変換/第2部:方程式の数値解法と図式解法 ウスコフ/ドモリヤード 1966 2刷
4、代数2 第3部:多項式環と有理式体 オクニヨーク 1966 2刷
5、解析1 第1部:実変数の初等函数、数列と函数の極限、函数の一般概念 ゴンチャロフ 1966 2刷
6、解析2 第2部:微分、積分、級数/第3部:複素変数の初等函数 ナタンソン/ゴンチャロフ 1966 2刷
413132人目の素数さん:2008/12/28(日) 09:39:13
ここはお前の日記帳じゃねーよ
414132人目の素数さん:2008/12/28(日) 09:39:21
スミルノフ高等数学教程 1〜12 12冊セット
スミルノフ高等数学教程
著者名 : スミルノフ,B.I.著 福原満洲雄・彌永昌吉他監
出版社 : 共立出版
発行年度 : 昭和36年

販売価格 : \14,000
415132人目の素数さん:2008/12/28(日) 10:48:51
春休みならそれくらい読めそうだな。面白そう
416132人目の素数さん:2008/12/28(日) 14:25:52
三日でスミルノフ一冊ってのはちょっと無理だろ。
最初のほうの巻しか無理。
417132人目の素数さん:2008/12/28(日) 17:48:54
たぶんやっても「読んだだけ」で終わる可能性が高い
一部の天才を除いて数学って実際にペンを取って理解をつけてくものだもの
418132人目の素数さん:2008/12/31(水) 19:59:23
>>372 >>401 >>405

 a[k] = 2(n+1-k)/(n+1),  (k=1,2,・・・,n)
 f(x) = (1/(n+1)){1-cos((n+1)x)}/{1-cos(x)} -1,
 等号成立は cos((n+1)x) =1 (ただし cos(x)≠1) のときで、
  x= 2π/(n+1), 4π/(n+1), ・・・・・, 2nπ/(n+1).
419132人目の素数さん:2009/01/01(木) 17:47:52
420132人目の素数さん:2009/01/01(木) 20:08:51
>>418
 f(x) = (1/(n+1))Σ[k=1,n] (n+1-k)*2cos(kx),

 2cos(kx) = {2cos(kx) - 2cos(kx)cos(x)}/{1-cos(x)}
      = {2cos(kx) - cos((k-1)x) - cos((k+1)x)}/{1-cos(x)},
を代入したな・・・
421132人目の素数さん:2009/01/07(水) 22:21:42
>>419
 7.Problems の 10番

10. Determine the maximal real number a for which the inequality
(x_1)^2 + (x_2)^n + ・・・・・ + (x_n)^2 ≧ a{x_1・x_2 + x_2・x_3 + ・・・・・ +x_(n-1)・x_n},
 holds for any n real numbers x_1, x_2, ・・・・・, x_n.

答は a = 1/cos(π/(n+1)) らしいんですけど、どうやって解くんでつか?
422132人目の素数さん:2009/01/07(水) 22:54:59
>>421
2次形式の行列が半正定値であればよい。
この行列の固有多項式は第2種チェビシェフ多項式を使って表せるから固有値が簡単に求まる。
423132人目の素数さん:2009/01/08(木) 02:58:30
>>421
対角化すると・・・・
(左辺) - (右辺) = (a/2)納k=1,n-1] s_k・s_(k+1) {x_k/s_k - x_(k+1)/s_(k+1)}^2,
 等号成立は x_k = s_k (の定数倍) のとき。
 ここに s_k = sin(kπ/(n+1)),
424132人目の素数さん:2009/01/08(木) 23:45:49
>>422
 (左辺) - (右辺) = xFx†
とおくと、
 det|F-λI| = (a/2)^n U_n((1-λ)/a),
U_n の零点は cos(kπ/(n+1)), (k=1,2,・・・・,n) だから、
Fの固有値は λ_k = 1 - a・cos(kπ/(n+1)), (k=1,2,・・・・,n)
最小の固有値は λ_1
Fが半正値 ⇔ λ_1 =0 ⇔ a=1/cos(π/(n+1)).
425132人目の素数さん:2009/01/09(金) 23:32:50
>406

nを超える最小の平方数を f(n) とおく。
 f(n) = M  ⇔  (M-1)^2 ≦ n < M^2,
上記の2M-1個のnについて題意が成り立つためには、n=(M-1)^2 について成り立てばよい。
 M^2 < (2009/2008)*(M-1)^2,
 √(2009/2008) < 1 - 1/M,
 M > 2009 + √(2008*2009) = 2009 + 2008.5 -ε = 2017.5 - ε,
 M ≧ 2018 のとき、すべてのnについて題意が成り立つ。
 M=2017 のときは n ≧ 2016^2 +1 について成り立つが、 n = 2016^2 については成り立たない。
これが最大の反例だから、k= 2016^2 +1.
426425:2009/01/10(土) 01:21:39
>>406 (訂正)

 M^2 < (2009/2008)*(M-1)^2,
 √(2008/2009) < 1 - 1/M,
 M > 2009 + √(2008*2009) = 2009 + 2008.5 -ε = 4017.5 -ε, (ε<<1)
 M ≧ 4018 のとき、上記のすべてのnについて題意が成り立つ。
 M=4017 のときは n ≧ 4016^2 +1 について成り立つが、 n = 4016^2 については成り立たない。
これが最大の反例だから、k= 4016^2 +1.
427132人目の素数さん:2009/01/10(土) 05:45:29
>>406

http://science6.2ch.net/test/read.cgi/math/1219648297/666
 4016^2 +1 = 16128257
K大入試作問者スレ(1)
428132人目の素数さん:2009/01/11(日) 15:46:13
age
429132人目の素数さん:2009/01/11(日) 20:56:23
A+B+C=π のとき次式を示せ。
 a'=sin(A/2), b'=sin(B/2), c'=sin(C/2) とおく.

(1) a'/(a'+b'+c') + b'/(b'+c'+a') + c'/(c'+a'+b') = 1,

(2) a'/(a'+b'c') + b'/(b'+c'a') + c'/(c'+a'b') = 2,

(3) 1/{tan(A)tan(B)} + 1/{tan(B)tan(C)} + 1/{tan(C)tan(A)} = 1,

(4) tan(A/2)tan(B/2) + tan(B/2)tan(C/2) + tan(C/2)tan(A/2) = 1,

http://science6.2ch.net/test/read.cgi/math/1230393194/40

おながいします。
430132人目の素数さん:2009/01/11(日) 21:10:27
スレチかつマルチ
431132人目の素数さん:2009/01/12(月) 08:59:36
一辺が1の正n角形x1x2x3…xnの内部に点Pをとる。
L=x1P+x2P+…xnP とするとき、Lの最小値を求めよ。
432132人目の素数さん:2009/01/16(金) 02:01:35
>>429
(1) は明らか。
 (2) a' + b'c' = a' + (1/2)cos((B-C)/2) - (1/2)cos((B+C)/2)
   = a' + (1/2)cos((B-C)/2) - (1/2)sin(A/2)
   = (1/2)cos((B-C)/2) + (1/2)sin(A/2)
   = (1/2)cos((B-C)/2) + (1/2)cos((B+C)/2)
   = cos(B/2)cos(C/2),
  (左辺) = {sin(A)+sin(B)+sin(C)}/{2cos(A/2)cos(B/2)cos(C/2)} = 2,

 (3) tan(C) = tan(π-A-B) = -tan(A+B) = -{tan(B)+tan(C)}/{1-tan(B)tan(C)},

 (4) tan(C/2) = tan((π-A-B)/2} = 1/tan((B+C)/2) = {1-tan(B/2)tan(C/2)}/{tan(B/2)+tan(C/2)},
433432:2009/01/16(金) 04:07:22
>>429
やってしまった・・・・ 訂正すまそ。

 (3) tan(A) = tan(π-B-C) = -tan(B+C) = -{tan(B)+tan(C)}/{1-tan(B)tan(C)},

 (4) tan(A/2) = tan((π-B-C)/2} = 1/tan((B+C)/2) = {1-tan(B/2)tan(C/2)}/{tan(B/2)+tan(C/2)},
434132人目の素数さん:2009/01/18(日) 20:19:41
東大の2002年前期3番って、円周率が\pi < 60/19=3.1578…を満たすことを証明する必要があったのでしょうか?
http://hiw.oo.kawai-juku.ac.jp/nyushi/honshi/02/t01.html
もしあったなら、この翌年の「\pi > 3.05を示せ」よりずっとエグいと思いますが。
円に外接する正12,24角形ではアウトなので、正36角形を持ち出すか、あるいは>>235のようなトリックが必要です。
もちろん三角関数表はない状況での話です。
435132人目の素数さん:2009/01/18(日) 23:49:21
必要ない
436ボケ:2009/01/19(月) 00:13:07 BE:624895564-2BP(1028)
>>431
Min_L(N) = N / √[2*{1-cos(2π/N)}]
437132人目の素数さん:2009/01/20(火) 02:10:23
正確に動いている時計がある。この時計の短針、長針、秒針がすべて重なる時刻は12時00分00秒だけであることを示せ。
ただし時計の針は3本とも等速円運動をしているとする。
438132人目の素数さん:2009/01/20(火) 04:39:46
>>437
レベル的には中学入試か?
439132人目の素数さん:2009/01/20(火) 05:19:33
>>437
普通に一致するだろ
例えば12:00:00から3600/59秒=61,01...秒後
440132人目の素数さん:2009/01/20(火) 05:25:46
すまんボケてた
3600/59分後だな
441132人目の素数さん:2009/01/20(火) 09:13:47
まだボケてる>>440
全然話にならないくらいにボケてる
442132人目の素数さん:2009/01/20(火) 09:35:48
どちらにせよスレ違い
443132人目の素数さん:2009/01/20(火) 10:00:28
>>441
どこが?
444132人目の素数さん:2009/01/20(火) 11:05:24
>>443

とりあえず、3600/59 分間に
 長針は 366.10・・・度
 短針は 30.50・・・度
それぞれ回転する。明らかにその差は360度の整数倍ではない。
445132人目の素数さん:2009/01/20(火) 11:20:17
>>444
本気で言ってる?
だとしたら相当頭悪い
446132人目の素数さん:2009/01/20(火) 11:22:47
>だとしたら相当頭悪い

自分のこと、か?
447132人目の素数さん:2009/01/20(火) 11:24:05
>>446
訂正するならしていいよ
448132人目の素数さん:2009/01/20(火) 11:26:04
>>445

3600/59 は大体61。12時ちょうどの61分後は1時1分。そのときに長針と短針が重なるのか、お前の星の時計だと。
449132人目の素数さん:2009/01/20(火) 11:33:40
何にしても、分母に59などと書いてる時点で、吟味する価値もないかと。55ならまだしも。

いい加減すれ違いだからやめれ。
450132人目の素数さん:2009/01/20(火) 12:33:52
59は出てくるがどうやったら55なんて出てくるんだ?
451132人目の素数さん:2009/01/20(火) 13:47:01
wwwwwwwwwwwwwwwwwwwwwwwwwwwwwwwwwww
452132人目の素数さん:2009/01/20(火) 14:04:35
>>450
ヒント:長針は12時間で一周する
453132人目の素数さん:2009/01/20(火) 14:05:14
>>431
これって実質Pが正多角形の中心であることを示せってことでしょ?俺の解答見てくれ。

xy座標を取り、多角形の中心が原点になり、y軸対称になるように置く。
このとき、P0をy軸上に、P1をその他の場所におく。(ただしP0とP1のy座標は同じ)

(続く)
454132人目の素数さん:2009/01/20(火) 14:11:25
>>452

オイオイw
455132人目の素数さん:2009/01/20(火) 14:14:36
(以降、L(P)=x1P+x2P+…+xnPと表記する。)

L(P0)<L(P1)をまず下に示す。

x1〜xnまでのn個をy座標の値でグループ分けする。

(1)1グループに一つ点がある場合(n:oddに限る)
その点をAとする
このときAP0<AP1

(2)1グループに二つ点がある場合。(つまりほとんどの場合)

その2点をB、Cとする

このときBP0+CP0<BP1+CP1となる。(図を書いて楕円の性質を考えれば明らか。)

x1からxnまでnこの点を前述の通りグループ分けすれば、任意のグループについて(1)、(2)が成り立つ。

従ってL(P0)<L(P1)

(続く)
456132人目の素数さん:2009/01/20(火) 14:22:05
L(P0)<L(P1)が意味する事は、正多角形についてそれを対称に分割する直線(直線Sと呼ぶ)を引いた場合、

「L(P)を最小とするPは必ずその直線上にある」という事が成り立つということである。

任意の直線Sの上にある点は、多角形の中心のみである。

よってminL(P)=N*(2sin(π/2n))^(-1)

従ってPは多角形の中心。
457132人目の素数さん:2009/01/20(火) 14:22:55
訂正

最後の2文の順番が逆。
458132人目の素数さん:2009/01/20(火) 14:26:11
さらに訂正
(n:oddにかぎる)は必要ない。

あとB,Cのy座標が0の時は

BP0+CP0≦BP1+CP1
459132人目の素数さん:2009/01/20(火) 14:39:17
>>445
お前の星だと、長針と短針は一分当たり(あるいは一時間当たり)それぞれ何度回るのか言ってくれ。
460132人目の素数さん:2009/01/20(火) 20:54:18
>>431
三角不等式つかいまくるだけなんじゃ?
461ボケ:2009/01/20(火) 20:56:45 BE:1458089287-2BP(1028)
>>456
その式で 正4角形を計算すると2√2にならんのだが・・・
462ボケ:2009/01/20(火) 21:51:39 BE:911306257-2BP(1028)
L=n/(2sin(π/n)) か。

漏れは 局座標系を使って全部計算で解いた。
原点を正n角形の中心点にとる。x1,x2,...,xN は、 (x_i,y_i)=(rcosθ,rsinθ), θ=2π/n*i, i=1,...,N
最小となる点が中心となることの証明も計算で・・・
原点と異なる点Pとx1,x2,...xNとの距離の和L(P)を計算する
点Pを(x,y)=(r_p cosθ_p, r_p sinθ_p)
L(P)=n(r^2+r_p~2) - 2rr_p把osX_i, X_i=2π/n*i-θ_p
となるが把osX_i はゼロになることが敗inX=0と加法定理から導ける


463ボケ:2009/01/20(火) 21:54:06 BE:703008239-2BP(1028)
結局、最小になるのは r_p = 0 のときで、点P=原点のときとなる。
464ボケ:2009/01/20(火) 22:03:42 BE:208299124-2BP(1028)
>このときBP0+CP0<BP1+CP1となる。(図を書いて楕円の性質を考えれば明らか。)
どのような楕円の性質を考えたらいいのか 教えてもらえないか?
465ボケ:2009/01/20(火) 22:32:43 BE:650933055-2BP(1028)
>>456
論理の飛躍があるような気がするんだけど・・・
>L(P0)<L(P1)が意味する事は、正多角形についてそれを対称に分割する直線(直線Sと呼ぶ)を引いた場合、
>「L(P)を最小とするPは必ずその直線上にある」という事が成り立つということである。

確かにYが同じなら 対称に分割する直線上の点が最小になるわけだけど 
じゃぁ その直線上では どこが最小になるのか 言えてるのか?
466ボケ:2009/01/20(火) 22:37:42 BE:546783473-2BP(1028)
あっ 漏れの方法ダメかも orz
467452:2009/01/20(火) 23:53:07
>>454
しまった!
468456:2009/01/20(火) 23:54:04
>>464
2chでは図が使えないから説明しにくいけど頑張る。

B,Cを焦点とし、P0を通る楕円を書いてください。その曲線状の任意の点Gについて
BG+CG=BP0+CP0がなりたちます(これが楕円の性質というか定義というか)

P1についても同様に楕円を書いてくれれば、後はその図で分かると思う。
469456:2009/01/20(火) 23:58:54
>>465
>じゃぁ その直線上では どこが最小になるのか 言えてるのか?

まず、その直線上にPが存在することは必要条件。次に、多角形を回転し、別の対象に分割する線をY軸に一致させる。
この場合でもPはY軸上に存在するのが必要。
こうなると二つの線の交点は原点の一箇所しかないからPはそこになる。

ところで最後の最後にくだらない計算ミスしてすいません。
470456:2009/01/21(水) 00:00:20
訂正:

別の対象に分割する線を→別の「対称に分割する線」を
471ボケ:2009/01/21(水) 00:31:13 BE:260373825-2BP(1028)
やっぱり 計算間違えてたw
距離計算するのにル〜トとるの忘れてたわ
けど、不等式つくって r_p=0のとき最小になると示せる
使った不等式は 嚢_i ≧ √(嚢_i^2)
472456:2009/01/21(水) 13:31:58
>>462
敗inX=0は分かるんですけど、そこから把osX=0を導く過程、教えてください。

あと、僕の解答は納得いただけましたか?
473132人目の素数さん:2009/01/21(水) 19:32:31
∫(dx/x) は有理式(整式の商)ではない事を示せ。
ただし ∫(dx/x) は未知とする。
474132人目の素数さん:2009/01/21(水) 20:34:46
A={(x,y) | x^2+y^2=1,x,yはともに有理数} とする.
P,Q,R ∈ A として三角形PQRをつくるとき,
三角形PQRの最大値は存在しない事を示せ.
475132人目の素数さん:2009/01/21(水) 21:49:42
>>473
未知とすると言われても・・・
(d/dx)log|x|=1/xを証明させたいんかな
476132人目の素数さん:2009/01/21(水) 22:20:26
>>474
三角形PQRの最大値って何?面積の最大値ってこと?
477132人目の素数さん:2009/01/21(水) 23:37:32
>>474
有理式を微分しても1/xにはならないって事
478狂介:2009/01/22(木) 10:54:03
>>474
まず、儕QRが正三角形でないことを示す。
(証明)
有理ベクトルOPについて、R(120°)OPは有理ベクトルじゃない。
ただしR(θ)はθ回転の行列
(証明終わり)
479狂介:2009/01/22(木) 10:59:20
次に、x^2+y^2=1の円上に二つの点を選び、その2点にはさまれる弧を選ぶと、その中に有理点があることを示す。
(証明)
pは有理数
(x,y)=(1,p)と(x,y)=(-1,0)を結ぶ直線と円の交点(のうち(-1,0)でない方)は有理点になる
また、いかなる二つの数に対しても、その間に有理数pは存在する
(証明終わり)
480狂介:2009/01/22(木) 11:05:55
次に本題の証明。
いかなる儕QRについても、それより面積が大きいものがあることを示す。

儕QRは上述の通り正三角形でないので、PQ≠PRとする。

QRに垂直二等分線を引き、それと円弧の交点のうち、QRに対してPと同じ側にある点を、Hとする。

上述の通り、HとPとの間には、必ず有理点があり、そのうち一つをP'と呼ぶ。

このとき面積は、儕QRより儕'QRの方が大きい。

以上より、三角形PQRの最大値は存在しない。
481ボケ:2009/01/22(木) 22:35:01 BE:156224232-2BP(1028)
>>472
正n角形の中心を原点とし、Y軸上に頂点x1を置く。
各頂点の角度θはX軸を基準にするとθ=2π/n*i+π/2ただし(i=0,1,..,n-1)となる。
このとき把os(θ)=0である。なぜなら各頂点はY軸に対して対称で
対称な頂点ではcos(θ)の値は符号が逆で大きさは同じ為。一方で、
把os(θ)=把os(2π/n*i+π/2)=倍cos(2π/n*i)cos(π/2)-sin(2π/n*i)sin(π/2)}
=倍cos(2π/n*i)*0-sin(2π/n*i)*1}=-1*敗in(2π/n*i)=0
すなわち敗in(2π/n*i)=0となる

X_i=2π/n*iと置くと
敗in(X_i)=0, 把os(X_i)=0

敗in(X_i+β)=倍sin(X_i)cos(β)+cos(X_i)sin(β)}
=cos(β)敗in(X_i)+sin(β)把os(X_i)
=0
同様に
把os(X_i+β)=0

482ボケ:2009/01/22(木) 22:41:14 BE:1275828277-2BP(1028)
>>481 追加
把os(X_i)=0 は 
敗in(θ)=敗in(2π/n*i+π/2)=倍sin(2π/n*i)cos(π/2)+cos(2π/n*i)sin(π/2)}
=倍sin(2π/n*i)*0+cos(2π/n*i)*1}=把os(2π/n*i)=0
と導ける
483ボケ:2009/01/22(木) 23:20:47 BE:468671292-2BP(1028)
>>472
>>469で、対称線上にL(P)があるのは確かだが
いきなり交点がL(P)と断言するのは飛躍してると思う。

対称軸Y1上の点Py1を、別の対称軸Y2を基準に評価すると
楕円の方法論で、Py1からY2軸に垂直に交わる点Py2をとると
原点からの距離は、Py2<Py1となり
L(Py2)<L(Py1)となり、これを繰り返すと
最終的に対称軸の交点が最小となることを
示した方がいいと思う。

実は交点じゃなくて 端っこの点の方が最小で最小点はn個というケ〜スもあると思う。
あるいは どの点でもL(Py)の値は同じかもしれんし
まぁ もれの頭が足りないだけなのかもしれんが orz
484狂介(=472):2009/01/23(金) 11:41:10
>>483
>>469で、対称線上にL(P)があるのは確かだが
>いきなり交点がL(P)と断言するのは飛躍してると思う。

対称軸にY1,Y2,Y3…と名前をつけます。
このとき、L(P)に最小値を与えるPならば、「そのPはY1上にある」ということが必要条件です。
同様に、「PはY2上にある」「PはY3上にある」も、すべて必要条件です。これらすべての必要条件を満たすのは唯一多角形の中心なので僕の考えはあってるかと。
485狂介:2009/01/23(金) 11:45:00
>>483
>対称軸Y1上の点Py1を、別の対称軸Y2を基準に評価すると
>楕円の方法論で、Py1からY2軸に垂直に交わる点Py2をとると
>原点からの距離は、Py2<Py1となり
>L(Py2)<L(Py1)となり、これを繰り返すと
>最終的に対称軸の交点が最小となることを
>示した方がいいと思う。

その操作を繰り返して、Py1,Py2,Py3…と作っていってもPynが多角形の中心になることはないと思う。(極限値は収束するけど)
だからその場合は中心について議論したことにならない希ガス。
486132人目の素数さん:2009/01/24(土) 07:53:36
y2=x^3+x^2-x という楕円曲線上の有理点をすべてもとめなさい。 30点
487132人目の素数さん:2009/01/24(土) 12:22:30
>>485
君が示したのはL(P0)<L(P1)のみ。これが意味するのは、
「対称軸上にない点は最小値になりえない」ということだけ。
よって、

(1)もし最小値が存在するならば

L(P)が最小になる点は原点しかありえないということになり、
証明は終わる。ところが、

(2)もし最小値が存在しないならば

もともと最小値が無いのだから、君が示したことは意味が無い。
君が示すべきは(1)。「そんな必要ないだろ」と思うかもしれないが、
これは絶対に必要。君がやっているロジックは、以下の議論と同じなのだ。

・自然数の最大値は1である。
証明:n≠1のときは、nより大きな自然数が存在する。これは、
1でない自然数は 自然数の最大値になりえないことを意味する。
よって、n=1が最大値である。
488132人目の素数さん:2009/01/24(土) 12:47:14
>>471
>使った不等式は 嚢_i ≧ √(嚢_i^2)
これは間違い。Σ|X_i|≧√(嚢_i^2)が正解。
489ボケ:2009/01/24(土) 16:02:10 BE:312448234-2BP(1028)
>>488
長さに負は存在しないから X_i>0 (書くの省略してました)
490ボケ:2009/01/24(土) 16:05:02 BE:416597344-2BP(1028)
公式として覚えてたわけじゃなくて、
この問題用に その場で証明して使っただけなんで・・・
491狂介:2009/01/24(土) 16:08:09
>>487
おっしゃるとおり、僕の解答には不備があるようです。「Pが原点以外のとき、最小値をとりえない。」
これが僕の示したところとなりますね。

>もともと最小値が無いのだから、…
これについては何かおかしいかと。何とかして最小値があることを示せば僕の解答は正しいことになるし、最小値は存在すると思いますが、いかがでしょう?
492ボケ:2009/01/24(土) 16:15:08 BE:1093567076-2BP(1028)
あ X_i≧0 ねw
493132人目の素数さん:2009/01/24(土) 16:15:14
>>471
その不等式を使っては示せないと思うが。
494132人目の素数さん:2009/01/24(土) 16:15:37
>>491
「もし(2)だったら、もともと最小値が無いことになって、意味が無い」
ということ。何もおかしくない。文脈上は

なぜ(1)を示さなければいけないのか? → もし(2)だったら意味が無いから

こういう流れで書いている。
495ボケ:2009/01/24(土) 16:56:33 BE:624895564-2BP(1028)
>>493
orz
496狂介:2009/01/24(土) 16:59:39
>>494
ごめんなさい。読み違えました。
497132人目の素数さん:2009/01/24(土) 18:37:27
平面上に9個の相異なる点があり、これらのうち少なくとも3点をとおる直線がn本ある。
nの最大値を求めよ
498ボケ:2009/01/24(土) 19:09:45 BE:520746454-2BP(1028)
>>493
不等式使うの辞めた。「嚢_iが最小 ⇔ (嚢_i)^2が最小」 を使うことにした。
内点Pと各頂点の距離をR_iとし、X_i=2π/n*iとする
(燃_i)^2を展開すると
結局、把os(X_i+β)=0だし、積和公式使うと把os(X_i+β)cos(X_j+β)=0となるので
(燃_i)^2=r^2+r_p^2   
となり、これが最小なのは r_pを0とした時となる
ただし
各頂点(x,y)=(r*cosX_i,r*sinX_i)
内点Pを(x,y)=(r_p*cosβ,r_p*sinβ)
とした
499ボケ:2009/01/24(土) 19:22:56 BE:416597928-2BP(1028)
>>498
あぅ また早とちりしてた orz
500132人目の素数さん:2009/01/24(土) 19:45:25
うん、BeをNGに入れたから
501Be:2009/01/25(日) 12:49:54 BE:833193784-2BP(1028)
>>431
L(P)を、極座標の動径rで微分するとr>0で正
r→0のlimで0になるからr=0のとき最小

じゃダメ?
502狂介:2009/01/25(日) 16:08:47
>>501
計算複雑そうでやる気しないんだが、r=0は不定義点なの?
503Be:2009/01/25(日) 19:15:03 BE:1874686289-2BP(1028)
>>502
定義されてる。
ちょっと、また勘違いしてて、別の計算の途中でr=0だと困るという勘違いしてた
504Be:2009/01/25(日) 20:51:55 BE:2109021899-2BP(1028)
正n角形の各頂点x_iは(x,y)=(rcos(2π/n*i),rsin(2π/n*i))
内点Pの座標は(x,y)=(rcos(θ_p), rsin(θ_p))

L(P)=煤縵r^2+r_p^2-2*r*r_p*cos(2π/n*i-θ_p)}
=煤綣{r_p-r*cos(2π/n*i-θ_p)}^2+{r*sin(2π/n*i-θ_p)}^2]
a_i=r_p-r*cos(2π/n*i-θ_p)
b_i=r*sin(2π/n*i-θ_p)
とおくと、L(P)=煤(a_i^2+b_i^2)

L(P)を動径r_pで微分すると
d{L(P)}/dr_p = d{煤(a_i^2+b_i^2)}/dr_p
=倍a_i/√(a_i^2+b_i^2)}
=納i≠k]{a_i/√(a_i^2+b_i^2)} ただし r_p≠0 また 存在するならばkのときa_k=0
=納i≠k]{1/√(1+(b_i/a_i)^2)}
>0
r_p=0のとき
d{L(P)}/dr_p = 納(-1*r*cos(2π/n*i-θ_p))/√{(r*cos(2π/n*i-θ_p))^2+(r*sin(2π/n*i-θ_p))^2}]
=納(-1*r*cos(2π/n*i-θ_p))/√(r^2)]
=-1*把os(2π/n*i-θ_p)
=0

以上から、L(P)はr_p=0のときL(P)が最小でr_p>0のとき増加

どうでしょう?
505Be:2009/01/25(日) 20:52:46 BE:2109021899-2BP(1028)
>内点Pの座標は(x,y)=(rcos(θ_p), rsin(θ_p))
内点Pの座標は(x,y)=(r_p*cos(θ_p), r_p*sin(θ_p))
506Be:2009/01/25(日) 21:31:00 BE:468671292-2BP(1028)
あっ やっぱダメだ orz a_iの符号が・・・a_i/|a_i|になって こまるぅ
507132人目の素数さん:2009/01/25(日) 22:43:01
>>473
f(x)=P(x)+Q(x)/R(x)とおく。ただしP,Q,Rは整式でQはn-1次以下、Rはn次とする。
f'(x)=P'(x)+(Q'(x)R(x)-Q(x)R'(x))/(R(x))^2
第2項の分子は2n-2次以下、分母は2n次だからf'(x)≠1/x
508132人目の素数さん:2009/01/26(月) 00:13:09
>>431
r=1/(2sin(π/n))とする。√は凹関数だから
L/n=(1/n)Σ[k=0,n-1]√(x^2+y^2+r^2-2r(xcos(2πk/n)+ysin(2πk/n)))
≧√((1/n)Σ[k=0,n-1](x^2+y^2+r^2-2r(xcos(2πk/n)+ysin(2πk/n))))
=√(x^2+y^2+r^2)≧r
x=y=0のとき実際にL/n=rを達成できるからLの最小値はn/(2sin(π/n))
凸不等式使うところはシュワルツの不等式でも行ける。
509132人目の素数さん:2009/01/26(月) 00:22:08
>>508
その式は間違ってる。不等号の向きが逆(√xは上に凸だから)。
510132人目の素数さん:2009/01/26(月) 01:01:33
>>455-456まで出ていながら 未だに誰も解いていないのが不思議でならない。


n本の対称軸によって、R^2は2n個の領域に分割される。それらの領域を
D1,D2,…,D2nと置く。x=x1∈R^2−{o}を任意に取るとき、x1∈Dkを満たす
kが少なくとも1つ存在する。Dkの境界は2本の対称軸(の一部分)であるから、
x1からそれらの対称軸のどちらかに垂線を下ろし、交わった点をx2とおく。
以下、図のようにしてx3,x4,…を作っていくと、xi→o in R^2である。

ttp://www.csync.net/service/file/view.cgi?id=1232899080

L:R^2→[0,∞)は連続であることに注意すると、L(xi)→L(o)が成り立つ。
また、>>455-456の議論から、L(x1)>L(x2)>L(x3)>… が成り立つ。
つまり、L(xi)はiの数列として狭義単調減少である。これとL(xi)→L(o)から、
L(x1)>L(o)が成り立つ。以上をまとめると、
x1∈R^2−{o} ⇒ L(x1)>L(o)
ということである。これは、原点oが最小値であることを意味する。
511132人目の素数さん:2009/01/26(月) 01:07:12
>以下、図のようにしてx3,x4,…を作っていくと、xi→o in R^2である。
↑図がいい加減で分かりにくいかもしれないが、要するに、垂線を下ろす操作を繰り返す。
512132人目の素数さん:2009/01/26(月) 01:12:17
>>509
あー俺もう駄目かも
513132人目の素数さん:2009/01/26(月) 01:31:22
別解:>>487の(1)を示す。これは よくやるオーソドックスな方法。
高校の範囲を少し超えるけどな。

まず、L:R^2→(0,∞)は明らかに連続である。また、簡単な評価によって
lim[|P|→∞]L(P)=∞ となることが分かる。よって、あるM>0が存在して、
「|P|>MならばL(P)>L(o)」…(*)が成り立つようにできる。
このMに対して、原点中心、半径Mの閉円盤Dを考え、LをD上に制限する。
DはR^2の有界閉集合だから、LはD上で最小値を持つ。その値をmとすると、
o∈Dだからm≦L(o)である。実は、mはR^2上におけるLの最小値にもなっている。
実際、P∈R^2−Dのときは、(*)によってL(P)>L(o)≧mとなるので。
よって(1)は成り立つ。つまり、LはR^2上で最小値を持つ。
514132人目の素数さん:2009/01/26(月) 01:48:35
円周上に五点を取って五角形を取るとき
その面積が最大になるのはどのような場合か、
というような問題も似たような話だよね
515Be:2009/01/26(月) 08:05:56 BE:911305875-2BP(1028)
>>511
それは>>483で書いてる
516132人目の素数さん:2009/01/26(月) 12:27:56
東大にこんな問題はでない
517132人目の素数さん:2009/01/26(月) 13:40:08
文系ですが失礼します

http://www.hotdocs.jp/file/90845
このサイトで東大数学の古い過去問を見つけたんですが回答がなくて困ってます

誰か回答もってないですか?
っていうか解いてくr いや、くださいwww
518狂介:2009/01/27(火) 18:36:40
>>513
ありがとう。それを僕の解に付け加えれば完璧になる。(といいながら高校の範囲超えてるから良く分からないけど)
519132人目の素数さん:2009/01/27(火) 22:12:49
>>517
この本を買えば載っていますよ。
http://www.amazon.co.jp/dp/4792210410/
あと、文系の問題は載っていませんがここは解答も充実しています。
http://www.j3e.info/ojyuken/math/
520132人目の素数さん:2009/01/28(水) 13:02:26
サイコロを3回なげ1回目に出た目の数をa二回目b三回目cとする
X=abcとする

@Xが奇数になる確率を求めよ
AX=12になる確率を求めよ
By=ax^2+bx+cがx軸からきりとる線分の長さが1/2以上になる確率を求めよ

おねがいしまあす
521132人目の素数さん:2009/01/28(水) 13:49:15
>>520
マルチ
522132人目の素数さん:2009/01/28(水) 22:02:44
tan1°は超越数か。
523KingGold ◆3waIuSKark :2009/01/28(水) 22:03:59
Reply:>>522 超越数だ。
524132人目の素数さん:2009/01/28(水) 22:09:54
oui
525132人目の素数さん:2009/01/28(水) 22:22:14
リンデマンの定理
a[1],…,a[n]を相異なる代数的数としたとき、
e^a[1],…,e^a[n]は代数的数体上線型独立である。
この定理を使う。

代数的数α≠0(cosα≠0)に対して、tanαが代数的数であるとすると
tanα=sinα/cosα=(e^(iα)-e^(-iα))/((e^(iα)+e^(-iα))i)より
(itanα-1)e^(iα)+(itanα+1)e^(-iα)=0
itanα±1は代数的数で同時に0とはならない。
これはリンデマンの定理から矛盾する結果である。
したがってtanαは超越数。

特にα=1は代数的数だからtan1は超越数。したがって、tan1は無理数。
526KingGold ◆3waIkAJWrg :2009/01/28(水) 22:39:40
Reply:>>523 お前は誰か。何が超越数か。
527132人目の素数さん:2009/01/28(水) 23:33:46
>522

tanの加法公式から、
 tan(nθ) = F(tanθ) / G(tanθ),
と書ける。F と G は n-1次とn次の整係数多項式。
n=45, θ=1゚ とおくと、
 F(tan(1゚))/G(tan(1゚)) = 1,
となるから、tan(1゚) は 45次の整係数方程式の根、よって代数的数。

528132人目の素数さん:2009/01/29(木) 02:30:54
>>519
丁寧にありがとうございます。
いいサイトですねww

背伸びして頑張ってみます。
529132人目の素数さん:2009/01/29(木) 03:10:36
そこで草を生やす意味が分からない
530132人目の素数さん:2009/01/29(木) 05:35:12
531132人目の素数さん:2009/01/31(土) 15:35:25
分からない問題はここに書いてね300
http://science6.2ch.net/test/read.cgi/math/1232536981/722

722 名前:132人目の素数さん[] 投稿日:2009/01/26(月) 20:35:36
時針、分針、秒針すべての長さが等しい時計がある。
針の先端がつくる三角形の面積が最大になる時刻はいつか。
532132人目の素数さん:2009/01/31(土) 16:46:57
497 名前: 132人目の素数さん [sage] 投稿日: 2009/01/24(土) 18:37:27
平面上に9個の相異なる点があり、これらのうち少なくとも3点をとおる直線がn本ある。
nの最大値を求めよ
533132人目の素数さん:2009/01/31(土) 21:50:11
>>525
おいこれ昔俺が益田んとこに書いたレスじゃねえかよ
534132人目の素数さん:2009/02/03(火) 23:24:05
益田とか懐かしいな
あいつ突然いなくなったけどめんどくさくなったんかな
535132人目の素数さん:2009/02/04(水) 00:25:33
リーマンショックで株価暴落して生活苦という説が有力。
536132人目の素数さん:2009/02/04(水) 00:26:50
株ニートかよw
537132人目の素数さん:2009/02/04(水) 01:57:38
ニートって、貧乏とかいう意味になって来てるんだろうか
>>536に限らず、本来の意味からどんどん離れてる気がする
スレ違いでごめん
538132人目の素数さん:2009/02/04(水) 05:31:34
l^2+m^2=n^2を満たす自然数l,m,nのうち
どれか1つは必ず2の倍数であることを示せ

これはできそうでできない難問
539132人目の素数さん:2009/02/04(水) 05:39:48
>>538
全て奇数と仮定して合同式はmod 4とすると(奇数)*(奇数)≡1, (偶数)*(偶数)≡0であり、
(左辺)≡2, (右辺)≡0により成立しないので、背理法により証明された。
540132人目の素数さん:2009/02/04(水) 06:34:31
>>539
大筋はいいとしても
途中で間違ってるぞ

541狂介:2009/02/04(水) 09:04:48
>>538
くそ簡単すぎてワロタ

l,m,nすべてが奇数だと、「奇数+奇数=奇数」となるのですべて奇数は否定された。
つまりどれか一つは偶数
542132人目の素数さん:2009/02/04(水) 13:42:13
>538
おい、顔真っ赤だぞ
543狂介:2009/02/04(水) 18:34:47
>>540>>542
>>538は数学的に合ってる
544132人目の素数さん:2009/02/04(水) 18:36:41
できそうでできない難問だというところが間違っているんじゃないの
545132人目の素数さん:2009/02/05(木) 01:35:16
>(右辺)≡0
(右辺)≡1だった
546538:2009/02/05(木) 01:42:59
お前ら解けんのかよ・・・
出直してきます
547132人目の素数さん:2009/02/05(木) 01:51:56
何だmod4じゃなくてmod2で解けちゃったのか
548132人目の素数さん:2009/02/05(木) 02:24:08
>>538は数学の素養に乏しいだろう
問題文がそれを如実に伝えているのである
> どれか1つは必ず2の倍数
の部分は
少なくとも1つは偶数
でいいからである
549132人目の素数さん:2009/02/05(木) 22:53:25
538が難問とか言ってる時点で素養もクソもなかろうw
550132人目の素数さん:2009/02/07(土) 15:01:42
538って中学生の教科書レベルでは?
まあ彼には難しいんだろうけどww
551132人目の素数さん:2009/02/07(土) 16:06:41
お前らつられ過ぎだろ
このスレ痛いやつばっかだな
552132人目の素数さん:2009/02/07(土) 16:38:32
今のがなければ>>550で最後だったのにな
心配しなくても黙ってれば終わるよ
出題者(笑)
553132人目の素数さん:2009/02/07(土) 16:41:02
1x2x4のブロックを7x7x7の立体にできるだけつめる問題
何個か
554132人目の素数さん:2009/02/08(日) 01:54:07
7^3 ÷ 1*2*4 で43より少ないことはたしかか。
555132人目の素数さん:2009/02/09(月) 03:31:38
2,2^2,2^3,…,2^2009の中で一番位が高い数字が1であるものの個数を求めよ
556狂介:2009/02/09(月) 18:55:39
>>555
(2^2009の桁数)-1
557132人目の素数さん:2009/02/09(月) 19:07:35
必ず桁が上がる時に1を経由するからね。
以前わからない問題スレにあった
2^555の桁数は168で最高位が1である
このとき2^n(n=1、2、3....555)の中で最高位が4の数は何個あるか
のほうがおもしろいね。
558132人目の素数さん:2009/02/10(火) 00:04:10
>>557
さすがに優秀な入試問題は練ってあるよね。
ちなみに早稲田教育の問題やね。しかも小問集合。鬼だw
559狂介:2009/02/10(火) 21:05:28
答え聞いた時は入試問題としては発想が難しすぎだなって思った。
数学は遊びだって人でも運が良くないと無理そう。
560132人目の素数さん:2009/02/12(木) 00:38:21
>>557は細かい計算が必要な気がしてごちゃごちゃやっててハッとした
561132人目の素数さん:2009/02/12(木) 18:51:55
次の連立方程式において、
0≦x,y<2πを満たす解はただ1組存在することを示せ。

sinx+cosx=sinycosy
sinxcosx=siny+cosy
562132人目の素数さん:2009/02/13(金) 06:38:06
媒介変数表示の求積問題だしてたな、あれはいかん。
563132人目の素数さん:2009/02/14(土) 14:01:50
>>561
それは偽だぞ。x'=x+π/4,y'=y+π/4とおくと
(1) sin(x')=-(1/2√2)cos(2y')
(2) sin(y')=-(1/2√2)cos(2x')
だからx'をπ-x'(あるいは3π-x')に置換しても同じ方程式になってしまう。
解がちょうど4つあることは次のように示すことができる。
(2)をsin(y')=(1/2√2)(2sin^2(x')-1)と思って(1)を代入すると、
(3) sin^4(y')-sin^2(y')-2√2sin(y')-3/4=0
という方程式になり、t^4-t-2^2-2√2t-3/4=0は[-1,1]の範囲に唯一の解を持つ。
この解は実は(-1,0)の範囲に含まれていてy'は2通りある。
(1)(2)と(1)(3)は連立方程式として同値だから、
sin(y')=tのもとで(1)の解の個数を調べればよいが、
sin(x')=-(1/2√2)(1-2t^2)は2解を持つので(x',y')の組は4通りある。
564132人目の素数さん:2009/02/15(日) 13:00:01
cos(x)+sin(x)=cos(y)sin(y)=a。
cos(x)sin(x)=cos(y)+sin(y)=b。
a^2=1+2b。
b^2=1+2a。
(a^2−1)^2=4(2a+1)。
a^4−2a^2−8a−3=0。
(a^2+2a+3)(a^2−2a−1)=0。
a=1−√(2)。
b=1−√(2)。
(cos(x)−sin(x))^2=2√(2)−1。
565132人目の素数さん:2009/02/15(日) 16:27:35
>>561

0≦x、y≦π に訂正を。
566132人目の素数さん:2009/02/15(日) 20:34:59
  = (a-b)(b-c)(c-a),
を差積とか Vandermonde 行列式とか 言うらしい。

〔問題〕
a,b,c≧0 のとき |處 ≦ (2/√3)(t/s)(s^2 -3t),
ここに、s=a+b+c, t=ab+bc+ca, u=abc.

http://science6.2ch.net/test/read.cgi/math/1179000000/737 , 739
不等式スレ3
567132人目の素数さん:2009/02/22(日) 04:36:56
>>557

題意より 168 ≦ n・log(2) < 168 + log(2),
最上桁が '1'
 [ n・log(2) ] = 76 個,
最上桁が '2' か '3'
 [ (n-1)log(2) ] +1 = 77個,
最上桁が '5'〜'9'
 [ (n+1)log(2) ] = 77個,
最上桁が '4'
 n - (76+77+77) = 25個, (← 題意より n=255)
568132人目の素数さん:2009/02/22(日) 06:26:20
>>567
全然違うぞ
569132人目の素数さん:2009/02/22(日) 06:27:57
>>522, 527

F(t) = 45t -14190t^3 +1221759t^5 -45379620t^7 +886163135t^9 -10150595910t^11
 +73006209045t^13 -344867425584t^15 +1103068603890t^17 -2438362177020t^19 +3773655750150t^21
 -4116715363800t^23 +3169870830126t^25 -1715884494940t^27 +646626422970t^29 -166871334960t^31
 +28760021745t^33 -3190187286t^35 +215553195t^37 -8145060t^39 +148995t^41 -990t^43 +t^45,

G(t) = 1 -990t^2 +148995t^4 -8145060t^6 +215553195t^8 -3190187286t^10
 +28760021745t^12 -166871334960t^14 +646626422970t^16 -1715884494940t^18 +3169870830126t^20
 -4116715363800t^22 +3773655750150t^24 -2438362177020t^26 +1103068603890t^28 -344867425584t^30
 +73006209045t^32 -10150595910t^34 +886163135t^36 -45379620t^38 +1221759t^40
 -14190t^42 +45t^44,
570132人目の素数さん:2009/02/22(日) 06:40:58
>>527
蛇足だが
 F(t) = Σ[k=0,[(n-1)/2]] (-1)^k C[n,2k+1] t^(2k+1),
 G(t) = Σ[j=0,[n/2]] (-1)^j C[n,2j] t^(2j),

(略解)
複素数を使う。√(-1) =i とおく。
 1+it ∝ cosθ + i・sinθ = exp(iθ),  (← t=tanθ)
 cos(nθ) + i・sin(nθ) = exp(inθ) = {exp(iθ)}^n,
より
 G(t) + i・F(t) = (1+it)^n = {Σ[j=0,[n/2]] (-1)^j C[n,2j] t^(2j)} +iΣ[k=0,[(n-1)/2]] (-1)^k C[n,2k+1] t^(2k+1)},
 F(t) = Σ[k=0,[(n-1)/2]] (-1)^k C[n,2k+1] t^(2k+1),
 G(t) = Σ[j=0,[n/2]] (-1)^j C[n,2j] t^(2j),

n=45 のときは >>569
571132人目の素数さん:2009/02/22(日) 16:23:09
>>567
その理論、nが小さい時に試してみたら?
572571:2009/02/22(日) 16:31:07
>>567
「+1」が見えてなかった
スマン
573132人目の素数さん:2009/02/24(火) 10:43:35
2x2の行列のある部分集合Fは次の性質を持つ。
 ・Fの任意の元 a,b について、a-bはFに属する。
  そして、Fの任意の元 aは、a-a=0 となる。
 ・また、Fの任意の元a,bについてa*bはFに属する。
  そして、Fのある元eは次のような性質を持つ
   ・任意のFの元aについて a*e = e*a であり、これはFに属する。
   ・e*e=e
   ・-e * -e =e という等式が成り立つ。
 ・さらに、Fのある元iについて次の等式が成り立つ、i * i = -e
元、e,iを求めよ。
574132人目の素数さん:2009/02/24(火) 10:49:22
>>573
久しぶりだね
575132人目の素数さん:2009/02/24(火) 11:01:27
>そして、Fの任意の元 aは、a-a=0 となる。
この引き算は行列としての引き算じゃ無いの?
だとしたら問題文でわざわざ断るのは不自然。
(いや何でこんなこと書いたのかは分かるんだけどね)

2×2行列としての構造を無視するなら、
ただの濃度が |R| の集合に過ぎないんだから何も求めようがなくなる
576132人目の素数さん:2009/02/24(火) 12:02:31
流れ豚切ってすみません。

以前、ここか京大スレかのどこかで
lim[n→∞]a[n]=0 ⇒ lim[n→∞]Π[k=1,n]a[k]=0
は成り立つか?みたいな雰囲気の問題を見た覚えがあるんですけど、
(本物はこんなに簡単じゃなくて、アイディアが必要な問題でした・・・)
誰か詳細をご存じないでしょうか。過去ログ調べたんですが見つかりませんでした。
(ちなみに>>3の過去ログ倉庫はぶっ壊れたんでしょうか・・・)
何か情報をお持ちの方は、ご教示下さい。
577132人目の素数さん:2009/02/24(火) 12:36:05
>>576
> (ちなみに>>3の過去ログ倉庫はぶっ壊れたんでしょうか・・・)
会員登録すれば誰でも使えたYahoo!ブリーフケースが、有料会員専用に変更された
578132人目の素数さん:2009/02/24(火) 23:37:23
>>576-577
過去ログ倉庫の避難所を用意しました。
http://cid-d357afbb34f5b26f.skydrive.live.com/browse.aspx/.Public/
579132人目の素数さん:2009/02/25(水) 00:33:57
>>576
例示してるのと大差ない気がするけどこれ?

813 名前:132人目の素数さん[sage] 投稿日:2008/10/26(日) 14:27:34
じゃぁ要望にこたえて、某有名、大学入試問題集から一問

次の命題の真偽を調べ、真ならば証明し、偽ならば反例を示せ。
「すべての非負整数 n について、0<a(n)<1 ならば、
lim[n→∞]a(1)a(2)a(3)****a(n)=0 」
580132人目の素数さん:2009/02/25(水) 12:33:46
>>579
んんっ…あっ…これです!!!
どうも有り難うございました。
>>578さんもお疲れ様ですm(_ _)m
581132人目の素数さん:2009/02/25(水) 21:55:13
>>580
んんっ・・・あっ・・・

エッチぃのは嫌いです><
582132人目の素数さん:2009/02/26(木) 00:07:33
>>579
当然偽だね。
logをとって考えれば,結局「負の数を無限個足せば-∞に発散する」という主張をしていることになるが,
もちろんそんなことは成り立たない。-1/2^n とかを考えれば明らか。
583132人目の素数さん:2009/02/26(木) 22:27:58
>>579
判例
 a[k] = {k/(k+1)}*{(k+2)/(k+1)},
 a(1)a(2)・・・・・a(n) = {1/(n+1)}*{(n+2)/2} → 1/2. (n→∞)


13 24 35 46
---------------------
22 33 44 55


584132人目の素数さん:2009/02/27(金) 00:05:35
>>579
凡例
 0<b<1 として, bに収束させる。
 a[k] = {(k-1+2b)/(k+2b)}*{(k+2)/(k+1)},
 a(1)a(2)・・・・・a(n) = {2b/(n+2b)}*{(n+2)/2} → b. (n→∞)
585132人目の素数さん:2009/02/28(土) 00:37:59
フィボナッチ数列を三角関数で表現しなさい
586132人目の素数さん:2009/02/28(土) 00:47:48
>>579 (別解)

 0<b<1 とすると、(sinθ)/θ = b となるθが(0,π) にある。
 a[k] = cos(θ/(2^k)) = sin(θ/{2^(k-1)})/2sin(θ/(2^k)),
 a(1)a(2)・・・・・a(n) = sinθ/{(2^n)sin(θ/(2^n))} → sinθ/θ = sinc(θ) =b, (n→∞)
587132人目の素数さん:2009/02/28(土) 00:56:31
579
有界単調減少ー>収束
limΠa(i)=c>0
c*.9<c
になるので
c>0は矛盾ー> c=0
588132人目の素数さん:2009/02/28(土) 02:42:35
今年も東大より京大の方が面白い
589132人目の素数さん:2009/02/28(土) 07:17:48
次の命題の真偽を調べ、真ならば証明し、偽ならば反例を示せ。
「すべての非負整数 n について、-1<a(n)<1 ならば、
lim[n→∞]a(1)a(2)a(3)****a(n)=0 」
590132人目の素数さん:2009/02/28(土) 08:20:25
>>582
-1/2^n の数列を無限個足していったら0になってしまわないかしら
591132人目の素数さん:2009/02/28(土) 08:39:35
どこをどう突っ込めばいいのやら
592132人目の素数さん:2009/02/28(土) 08:51:52
ああ、かけていったら、です。(-1)^n/2^(0.5n*(n+1))→0だよなあ
593132人目の素数さん:2009/02/28(土) 12:45:00
>>412あたりの本にちょっと興味あるのだけど、本当に買ってみるべきかしら?
他にもっとこれやれって本はあったりするのかしら
594132人目の素数さん:2009/02/28(土) 12:58:25
>>579 (別解)

 0<b<1 とする。
 a(k) = b^{(1/2)^k},
 a(1)a(2)・・・・・a(n) = b^{1 - (1/2)^n} → b (n→∞)
595132人目の素数さん:2009/02/28(土) 13:40:08
>>587
なにこれ?
596132人目の素数さん:2009/02/28(土) 14:37:08
597596:2009/02/28(土) 14:48:11
>>585 リンクミス、すまそ。

http://science6.2ch.net/test/read.cgi/math/1073918716/421-422
定理スレ
598132人目の素数さん:2009/03/01(日) 01:08:48
n枚の互いに異なるカードがひとつの山に重ねてある。初期の順序の状態を順序Mとする。
以下の試行によりカードを並び替える。ただしp,qはnの約数とする。
@山を上から順にp等分する。それぞれの山をA1、A2、・・・、Apとする。
AA1,A2・・・Apと順に一番上のカードを取っていく。Apのカードを取ったらA1に戻り山がなくなるまで繰り返す。
B先に取ったカードをが上になるように一つの山を作る。
この試行をZ(p)とする。

(1)順序MからZ(p)をm回繰り返した。このとき順序Mとなっている条件を求めよ。
(2)n≧pqとする。このとき、順序MからZ(p)をq回行った山のカードの順序と、順序MからZ(pq)を1回行った山のカードの順序が等しいことを示せ。
(3)順序MからZ(p)をa回行った後、その状態からZ(q)をb回行った。このとき順序Mとなっている条件を求めよ。
599132人目の素数さん:2009/03/01(日) 09:00:05
あ、間違えた
(2)順序MからZ(p)をq回行った山のカードの順序→順序MからZ(p)を行った後Z(q)を行った山のカードの順序
600132人目の素数さん:2009/03/01(日) 20:52:42
>>579

 B(0) = 1,
 B(n) は単調減少
 Lim[n→∞] B(n) = b,
を満たす数列 B(n) に対して
 a(n) = B(n) / B(n-1),
601132人目の素数さん:2009/03/03(火) 12:14:39
たった一つのことを使い回していくだけなので面白みに欠けるところがあるが
まあ入試なら差が付くだろうし,昔東大にも似たようなのあったからいいか。

2009^2009の各位の和を計算し,更にその各位の和を計算し…
と出てきた数の各位の和の計算をくり返していくとき,
最後に残る一桁の数字を求めよ。
602132人目の素数さん:2009/03/03(火) 12:54:15
パクり乙
603132人目の素数さん:2009/03/03(火) 14:21:37
>>601
答え 5
9で割った余りを求めればよい。2009~2009=2^2009=2^{6*334+5}=2^5=5 (mod 9)
604132人目の素数さん:2009/03/03(火) 14:36:10
糞問ばっかだな
605132人目の素数さん:2009/03/04(水) 23:49:36
円周率πと、√3+√2の大小を比較せよ。
606132人目の素数さん:2009/03/05(木) 01:11:47
>>605
まず π^2/6 = 2 - 1/(1^2*1*3) - 1/(2^2*3*5) - 1/(3^2*5*7) - … を導く。
1/(1^2*1*3) + 1/(2^2*3*5) + 1/(3^2*5*7) + …
=(1/1^2)(4/(1*3)-1) + (1/2^2)(16/(3*5)-1) + (1/3^2)(36/(5*7)-1) + …
=(4/(1*3)-1/1^2) + (4/(3*5)-1/2^2) + (4/(5*7)-1/3^2) + …
=(2/1-2/3-1/1^2) + (2/3-2/5-1/2^2) + (2/5-2/7-1/3^2) + …
=2 - π^2/6
従って
π^2 = 6{ 2 - 1/(1^2*1*3) - 1/(2^2*3*5) - 1/(3^2*5*7) - … }
= 12 - 2 - 1/10 - 2/105 - 1/168 - …
両辺の2乗は
(√2+√3)^2 = 5+2√6 = 10-(5-2√6) = 10-(√25-√24)
π^2 = 10 - (1/10+2/105+1/168+…) = 10-(5/42+1/168+…)
10から引かれる値の大小関係は
√25-√24 = 1/(√25+√24) < 1/(5+√16) = 5/45 < 5/42 < 5/42+1/168+…
従って (√2+√3)^2 > π^2 つまり √2+√3 > π
607605:2009/03/06(金) 00:17:24
>>606
素晴らしい!正解です。
608132人目の素数さん:2009/03/06(金) 14:28:59
>>607
>>607が用意した解答が>>606と同じならむずくねえかこれ?高校レベルなの?
>>606は5行目から6行目を導くためには和の順序交換をしなければならないが
無限級数の和の順序交換は一般にはできないため、絶対収束性の確認がいる
明らかに高校レベルを超えていると思われる
>>606にちょっと意見するならば
5行目が絶対収束することを示すには結局4行目に戻らなければならないので
(5行目で絶対値級数をとると∞発散してしまう)
4行目が絶対収束することを示して、和の順序交換ができることに言及した上で
4行目=(4/(1*3)+4/(3*5)+4/(5*7)+・・・)+(1/1^2+1/2^2+1/3^2+・・・)
と和の順序交換ができて、=6行目とするべきだと思う
609132人目の素数さん:2009/03/06(金) 17:44:21
小数点以下2桁で見分けつかないのはキツイ
610132人目の素数さん:2009/03/07(土) 13:30:37
>>605
>606しか解答がないわけか?
ちなみに俺は、正6*2^n角形でπの値を評価していく明らかに現実的じゃない方法しか思いつかないんだが。
611132人目の素数さん:2009/03/07(土) 13:51:32
>>235の途中から
π<2√6-4√3-2√2+8<2*2.45-4*1.732-2*1.414+8=3.144<1.732+1.414<√3+√2
612132人目の素数さん:2009/03/07(土) 13:52:59
でも、この方法もたまたまうまくいっただけで、試験中には現実的に無理か
613132人目の素数さん:2009/03/07(土) 14:11:57
cos2x+cos3x-6xの挙動を調べてx=π/12を代入して・・・
という方針を試してみたが、うまくいかなかった
誰か高校レベルでの解法を頼む
614605:2009/03/07(土) 23:28:22
>>610
正48角形を考えれば大丈夫。基本的には(もう消えているが)>>218と同様の方針。
p=\tan \pi/48とすると示すべきはp<(\sqrt{3}+\sqrt{2})/48
加法定理より 2p/(1-p^2)=1/(\sqrt{6}+\sqrt{3}+\sqrt{2}+2)
よりpは二次方程式p^2 +2p(\sqrt{6}+\sqrt{3}+\sqrt{2}+2) -1=0の正の解。
(\sqrt{3}+\sqrt{2})の形になっているから、>>218よりは簡単にいくと思われる。

というのと>>235のようにやるのが、一応想定した解答。
615132人目の素数さん:2009/03/08(日) 07:58:53
円周率π=3.14152と、√3=1.732050+√2=1.41421356の大小を比較せよ。
616132人目の素数さん:2009/03/08(日) 08:58:54
意味不明。
617132人目の素数さん:2009/03/08(日) 18:52:58
>>661
2007年度はBとCレベルばかりなのに、大学への数学からは>>288に書いてあるように
難しすぎる、難易度の調節が出来ないなら入試を作るのを止めろとまで罵られたらしい。
他の予備校はそこまでの極端な難化とは見ていないのに。

極端な例かも知れないが、例えばこんな問題を
http://science6.2ch.net/test/read.cgi/math/1220115988/605
何の誘導も無しに出すのが京大。懇切丁寧に誘導を付けて出すのが阪大。
618617:2009/03/08(日) 18:54:39
誤爆しました。
619132人目の素数さん:2009/03/08(日) 18:56:05
誤爆した理由はわかるが書き込むつもりだったスレが気になる
620132人目の素数さん:2009/03/08(日) 19:40:32
90年代の東大の入試問題作問者は首吊って死ななきゃいけないなw
621132人目の素数さん:2009/03/08(日) 20:08:11
あれで試験になってたんだよ
622132人目の素数さん:2009/03/08(日) 23:17:36
数学は実質的に120点満点の試験としての機能を果たしてなかったけどね。
六問中一問が解けてもう一問で部分点を貰えれば
どこの科類でも目指せた時代なので。
623132人目の素数さん:2009/03/08(日) 23:28:34
それが本当なら、確かに入試問題としておかしいなww
624132人目の素数さん:2009/03/09(月) 00:08:02
今見ると恐ろしく簡単に見える・・何であんなので2完だったんだ俺。
まぁうかったけど。
625132人目の素数さん:2009/03/10(火) 00:55:38
>>605
不等式への招待 第3章
http://science6.2ch.net/test/read.cgi/math/1179000000/451
(これ以降にも関連レス)

「√2+√3>πの証明」
ttp://www2.ocn.ne.jp/~mizuryu/toukou2/toukou56.html
626132人目の素数さん:2009/03/10(火) 02:37:23
超天下り式だが
∫[0→1]x^4(1-x)^4/(1+x^2)dx=22/7-π>0 より

√2+√3>1.414+1.732=3.146>22/7>π
627132人目の素数さん:2009/03/10(火) 02:41:54
これはすごい
628132人目の素数さん:2009/03/10(火) 02:54:08
22/7か、思いつかなかったな
629132人目の素数さん:2009/03/10(火) 02:57:38
πの近似値
630132人目の素数さん:2009/03/10(火) 03:44:47
7/22は円周率近似値の日だからね
631132人目の素数さん:2009/03/10(火) 03:51:28
これも頼む

去年の数検の問題
ttp://www.suken.net/img/2008-07dani.pdf

数値計算をせずに
π^4 + π^5 < e^6
であることを理論的に証明しなさい。
632132人目の素数さん:2009/03/10(火) 04:02:24
22/7をはさむのは思いつかなかった。それほど精度がいいもんなんだなあ。
633132人目の素数さん:2009/03/10(火) 04:06:14
>>626
感動がかなり大きいんだが
634132人目の素数さん:2009/03/10(火) 08:58:13
22/7を挟む発想より、22/7>πが簡単に証明できることに驚き
635132人目の素数さん:2009/03/10(火) 09:53:29
22/7 > π はずっと上で証明されてるけどな(>>240)
むしろ左辺の積分がどこから振ってきたのか
636132人目の素数さん:2009/03/10(火) 10:48:11
>>625-626
これはすごい
637132人目の素数さん:2009/03/10(火) 10:57:11
「π > 3.14 を示せ」は難しい?
638132人目の素数さん:2009/03/10(火) 14:27:59
>>637
不可能だと思うよ。
639132人目の素数さん:2009/03/10(火) 14:34:57
>>626が良くわからないorz
640639:2009/03/10(火) 14:38:02
ごめんわかった
641132人目の素数さん:2009/03/10(火) 14:51:21
(i^4)(1-i)^4が実数だから被積分関数は多項式+定数/(1+x^2) って事か
8乗したらもう少し良い評価になるのかな
642132人目の素数さん:2009/03/10(火) 15:22:43
自然数nに対して、(n!)^2≧n^nが成り立つことを示せ
643132人目の素数さん:2009/03/10(火) 15:27:09
n!*n!=Πk(n+1-k)
644132人目の素数さん:2009/03/10(火) 16:55:17
>>631
難しいな
ちょっと考えただけじゃ想像つかない
リンクの問題見てみたが、やる気が起きない問題ばっかだな
645132人目の素数さん:2009/03/10(火) 18:11:49
数検の段位問題とかのやる気の起きなさは異常だよなwww
646132人目の素数さん:2009/03/10(火) 18:28:06
つまんないってこと??
647132人目の素数さん:2009/03/10(火) 20:58:28
いかにも問題のための問題として作られたような不自然な煩雑さに満ちた問題だから。
648132人目の素数さん:2009/03/10(火) 22:38:52
>>644
スレ違いかも知れないが、数検段位で本当に難しいのは共通問題。
誰でも何かしらは答が書けるかもしれないが、数検側の眼鏡にかなった
答案を書くのは超至難の技。しかも配点は共通問題のほうが大きいらしい。

初段[1]は簡単。[2]は一見簡単そうで難しい。
2段[1]は今年の東大の問題5に似ているタイプで、今年の東大[5]より計算がやや易しい。

3段[1]は一松信氏の本で紹介されていたが、
http://www.amazon.co.jp/dp/4535609020/
証明は書いていなかった。
649132人目の素数さん:2009/03/10(火) 22:42:59
結局数板に解けるやつはいないのか
普段えらそうにしてる割にはいざとなると役に立たないんだな
その問題は興味がわきません(笑)
650132人目の素数さん:2009/03/10(火) 22:45:05
数オリの問題のが良問だな
651648:2009/03/10(火) 22:47:55
>>649
はい。斯く言う私も>>631は解けませんでした。
652132人目の素数さん:2009/03/10(火) 23:58:19
>>639-641

被積分函数は x^4・(1-x)^4 /(1+x^2) = x^6 -4x^5 +5x^4 -4x^2 +4 -4/(1+x^2),

 (左辺) = [ (1/7)x^7 -(2/3)x^6 +x^5 -(4/3)x^3 +4x -4arctan(x) ](x=0,1)
 = (1/7) -(2/3) +1 -(4/3) +4 -π
 = (22/7) - π,
653132人目の素数さん:2009/03/11(水) 00:03:30
>>652
それはわざわざ書いてもらうほどの事ではないな。
4乗を8乗に代えたたものがみてみたいのだ。
654132人目の素数さん:2009/03/11(水) 00:37:24
>>649
お前分かってないな
主張や結果が興味深かったり、それを導く過程が楽しかったりするから解くんだろうが
その点>>650の言うように数オリの方が勝る
655132人目の素数さん:2009/03/11(水) 00:52:48
>>653
xmaximaにやらせてみたところ

∫[0→1]x^8(1-x)^8/(1+x^2)dx=4π-188684/15015 (>0)

∴π>47171/15015=3.14159174…

今度は下から評価できた。
656132人目の素数さん:2009/03/11(水) 01:02:42
>>655
かなり正確な評価で驚いた
だが計算量を考えれば妥当なところなのかな
657132人目の素数さん:2009/03/11(水) 01:04:58
では 355/113>π であることを…
658132人目の素数さん:2009/03/11(水) 01:13:32
解く気がしないというのが、古くから結果は良く知られていた
有名問題を証明させる、みたいな感じの問題が結構多いんだよね。
まあJMOとかでも既出の問題が出題されることとかは以前はあったんだけど。

例の問題は(どっかのスレでも長々と書いたけど)
適当に積分の式を評価すれば解けるんじゃないの?
「数値計算をせずに」というのが何を意味するのか知らんが、
分数や小数の手計算くらいはしないと無理だと思う。
数値計算をせずに 3.1415926535<π<3.1415926536を示しなさい、と同程度に無理。
問題文を読んだ感じでは「プログラム組んで計算してみました」
的な解答でなく、手計算で求めれば良い、という意味かと。

しかし
>ガウス平面(R^1-I^1 数空間)を手懸りにしながら, R^2数空間とI^2数空間の
>関係について,あなたの見解を論述しなさい。
とかマトモな人が問題出してるのかと不安になるんだが大丈夫なのかね。

残りの共通問題も、出題者の主観を押し付けて立論させるような問題しかないし
659641:2009/03/11(水) 03:11:45
>>655-657
(i^m)(1-i)^nが正なら下から、負なら上から評価されるね
(m,n)=(6,8)で
π<3+25513/180180 =3.1415972…
(m,n)=(10,8)で
π<3+173483/1225224 =3.1415928…
660132人目の素数さん:2009/03/11(水) 06:28:19
>>626 の被積分関数を変更して、ln(2)<0.7が示せるね。

∫[0→1]x^4*(1-x)^2/(1+x^2) dx = -ln(2) + 0.7
661132人目の素数さん:2009/03/11(水) 06:31:37
お前らホント数学好きなんだね
662132人目の素数さん:2009/03/11(水) 06:55:56
>>660
でも、ln2の評価なら、分母は1+xでもいいんだし、やっぱり626の積分は\piの評価にこそふさわしいと思う。
663132人目の素数さん:2009/03/11(水) 11:00:51
>>631って不等式スレで何回か出てきたけど誰も証明できてなかったな
本当は証明できないんじゃねぇの
そもそも誤差か小さすぎだし
664132人目の素数さん:2009/03/11(水) 11:00:55
>>631って不等式スレで何回か出てきたけど誰も証明できてなかったな
本当は証明できないんじゃねぇの
そもそも誤差か小さすぎだし
665132人目の素数さん:2009/03/11(水) 11:17:40
大事なことなので二回言いました
666132人目の素数さん:2009/03/11(水) 17:11:59
√2+√3>πを変形して6>(x^2-5)^2/4 |_x=πとして、
x=3.142での値挟むのは既出だろうな
667132人目の素数さん:2009/03/11(水) 17:17:41
考えないはずが無いけど、誰も書いてないね
668132人目の素数さん:2009/03/11(水) 21:35:05
>>631
数値計算をしないというのは
e>2.718281828
π<3.14159266
を証明して
π^4+π^5<e^6
が成り立つことがわかってもダメってこと?
理論的というのがイマイチわからん
669132人目の素数さん:2009/03/11(水) 21:38:16
>>641, >>653

(蛇足だが・・・)
 I_n = {1/4^(n-1)}∫_[0,1] {x(1-x)}^(4n) /(1+x^2) dx
とおくと
 I_1 = (22/7) - π
  = 3.14285714285714… - π
  = 1.26448926734961868021375957764e-4,

 I_2 = π - 47171/(3*5*7*11*13) =
  = π - 3.14159174159174…
  = 9.1199805164672105164168776129246e-7,

 I_3 = 5606935373/(16*3*5*7*11*13*17*19*23) - π
  = 3.1415926543282176611023023729983 - π
  = 7.3842442263965898971879150324784e-10
より
 |I_n| 〜 2*c^{n*[1-(n-1)/111.87]},
ここに c = (1/2)I_1,
670669:2009/03/11(水) 21:51:57
>>641, >>653

訂正、すまそ
 I_1 = 1.26448926734961868021375957764e-3,
671132人目の素数さん:2009/03/11(水) 23:16:02
〔まとめ〕
 47171/(3*5*7*11*13) < π < 5606935373/(16*3*5*7*11*13*17*19*23) < 355/113 < 22/7 < √2 + √3,

(略解)
 47171/(3*5*7*11*13) = 3.14159174159174159174・・・
 π = 3.1415926535897932384626433832795
 5606935373/(16*3*5*7*11*13*17*19*23) = 3.1415926543282176611023023729983
 355/113 = 3.1415929203539823008849557522124
 22/7 = 3.1428571428571428571428571428571
 √2 + √3 = 3.1462643699419723423291350657156
672132人目の素数さん:2009/03/11(水) 23:51:44
1+1=2とか5x-3x=(5-3)x=2xとかは数値計算なんだろうか
673132人目の素数さん:2009/03/12(木) 00:41:05
それが禁止されたら数学では解けないな
674132人目の素数さん:2009/03/12(木) 00:43:04
採点官に「数値計算をしてない」って思ってもらえればいいんだろうけど、明白な基準がないからなぁ……
675132人目の素数さん:2009/03/12(木) 00:44:01
おそらく数値計算は関数電卓とかの使用を禁止します的なものだと思う
手計算では>>668の評価はできるけどかなり大変
676132人目の素数さん:2009/03/12(木) 00:48:34
まー、上のやり方と同じで考えれば被積分関数が常に0以上で、積分値がうまい具合利用できるような奴を考えるとか……
いや、こんなのがすぐに思い浮かぶ問題だったら、難易度低すぎて数学検定にならない事を考えると方向性違ってるかな?
677132人目の素数さん:2009/03/12(木) 00:53:10
http://www.google.co.jp/search?hl=ja&client=firefox-a&rls=org.mozilla%3Aja%3Aofficial&hs=5L6&q=e^6-(pi^4+%2B+pi^5)&btnG=%E6%A4%9C%E7%B4%A2&lr=lang_ja

いい加減な計算だと評価できないっぽい値だ
678132人目の素数さん:2009/03/12(木) 00:59:52
>>675
なるほど……
679132人目の素数さん:2009/03/12(木) 11:06:49
素朴に計算したらどんなことになるかやってみた

ニュートンの公式
π/6 = 1/2 + 1!!/(2!!*3*2^3) + 3!!/(4!!*5*2^5) + 5!!/(6!!*7*2^7) + …
で、10項目以下を
17!!/(18!!*19*2^19) + 19!!/(20!!*21*2^21) + …
< 17!!/(18!!*19*2^19) * (1 + 1/2^2 + 1/2^4 + …)
= 17!!/(18!!*19*2^19) * (4/3)
と押さえて評価すると
π < 13087828316373115/(2^32*3*7*11*13*17*19)

e は級数展開
e = 1/0! + 1/1! + 1/2! + 1/3! + …
で 1/11! の項までで打ち切ると
e > 13563139/(2^5*3^4*5^2*7*11)

この評価を緩めて少し簡単な分数にすると
p = 5*19*5989/(2^4*3*7^3*11) として
π < p
p+1 = 750059/(2^4*3*7^3*11)
x = 5*19*20543/(2^2*3^2*7^2*11*37) として
e > x

π^4(π+1)/e^6 < p^4(p+1)/x^6
= 3^7 * 11 * 37^6 * 5989^4 * 750059 / (2^8 * 5^2 * 7^3 * 19^2 * 20543^6)
< 1
(最後の計算も少し工夫できるけど、せこ過ぎるから割愛)
680132人目の素数さん:2009/03/12(木) 13:38:07
>>668の評価ができた所で
2.718281828の6乗とか3.14159266の5乗が手計算できない
681132人目の素数さん:2009/03/12(木) 17:03:10
適当に切り捨てたりしながらやれば何とかできそう
682132人目の素数さん:2009/03/12(木) 23:41:23
>>669
 ∫_[0,1] {x(1-x)}^m /(1+x^2) dx 〜 {1/√(2m)}(1/4)^m,
なので n>>1 のとき
 I_n 〜 {4/√(8n)}(c^n),
 c = (1/4)^5 = (1/2)^10,
683132人目の素数さん:2009/03/13(金) 20:37:38
http://www.yozemi.ac.jp/nyushi/sokuho/recent/tokyo/koki/index.html

総合問題Uが後期の数学ね(文系・理系共通)
684132人目の素数さん:2009/03/13(金) 20:58:37
∫[0→1]x^2*(1-x)^2/(1+x^2) dx = ln(2) -2/3

手計算でできるやつをやってみると色々面白い。
>>660と併せて  2/3<ln(2)<0.7 か
685132人目の素数さん:2009/03/13(金) 22:54:45
>>669 >>682

∫_[0,1] {x(1-x)}^m dx 〜 {1/√[1+(4/π)m]}(1/4)^m,
 4/π ≒ 1.273239544・・・・

∫_[0,1] {x(1-x)}^m /(1+x^2) dx 〜 {(π/4)/√(1+1.22675m)}(1/4)^m,
686132人目の素数さん:2009/03/19(木) 01:10:59
http://dic.nicovideo.jp/a/72 より・・・
> なお、2桁の自然数の中で60,84,90,96と並び、もっとも多くの約数を持つ数字であるが、
> 千早はその数字に割り切れない思いを抱いているようだ。
これを見て思いついた問題。今回は逆に巨乳・三浦あずさ(88cm)をねたにします。

(1) 自然数nについて、f(n)=(nの約数の個数)/nとする。
たとえば、f(1)=1, f(100)=9/100である。
このとき、任意の自然数nと素数pについて、f(pn)≦f(n)であることを示せ。
また、等号が成立するのはどのような場合か。

(2) 末広がりで縁起の良い数とされる、88はちょうど8個の約数
(1,2,4,8,11,22,44,88)を持つが、11nの約数の個数がn個となるような
自然数nはn=8以外には存在しないことを示せ。
687132人目の素数さん:2009/03/20(金) 00:30:23
>>686の(2)はミスです。
(×) n=8以外には存在しない
(○) n=8,12以外には存在しない
のように読み替えお願いします。
688132人目の素数さん:2009/03/27(金) 22:48:31
(84753+228i)^{87}は実数か?
689132人目の素数さん:2009/03/28(土) 10:07:14
>>688
実数じゃない
ちょっと遅くなりましたが、

> 242 名前: 240 投稿日: 01/10/10 02:25
>
> >>241
> こちらが想定したとおりの解法です。全部◎
> 解いてみた感想を聞かせて。


小問の誘導が適切で解きやすかったです。いい問題だと思いマスタ。
691132人目の素数さん:2009/03/31(火) 02:59:27
>>688
任意の自然数nについて(84753+228i)^nが実数でないこと、
もっと一般に(a+bi)^n (a,bは整数でa≠0,b≠0,a≠±b)が実数でないことを
証明したかったのですが挫折しました。

(84753+228i)^87が実数でないことは以下のようにわかります。

数列a(n),b(n)を次の漸化式で定めると、(84753+228i)^87=a(87)+b(87)iである。
a(1)=84753,b(1)=228,a(n+1)=84753a(n)-228b(n), b(n+1)=84753b(n)+228a(n)

以下合同式はmod(5)であるものとすると、
a(1)≡3, b(1)≡3, a(n+1)≡3a(n)-3b(n), b(n+1)≡3a(n)+3b(n)であるから、
a(2)≡0,b(2)≡3
a(3)≡1,b(3)≡4
a(4)≡1,b(4)≡0
a(5)≡3,b(5)≡3
以降周期4の繰り返しであるから、b(87)≡b(3)≡4よりb(87)≠0,
よってa(87)+b(87)iすなわち(84753+228i)^87は実数とはならない。
692132人目の素数さん:2009/03/31(火) 10:50:32
>>690
ちょっと遅いってレベルじゃねーぞ!ww
693132人目の素数さん:2009/03/31(火) 12:49:10
(a+bi)^n が実数でないことの証明って京大で出なかったっけ
694132人目の素数さん:2009/03/31(火) 18:23:14
πが無理数であることを証明せよ、

じゃなくて、πが無理数であることを
最初に証明した人について知るところを述べよ。
(50点)
695132人目の素数さん:2009/03/31(火) 20:09:39
Lambertとか数学科の学生でもほとんど知らんだろ。
696132人目の素数さん:2009/03/31(火) 21:46:55
一応できたつもりだが、何とも泥臭い(^q^)
とりあえず前半。合ってるかな?

(a+bi)^nが実数になるようなa,b∈Z,n∈Nを全て求める。
先に結論を書くと、
nが偶数のとき:a=±bまたはa=0またはb=0
nが奇数のとき:a=0またはb=0
となる。

STEP1:nが奇数のときにa=0またはb=0となることは後で証明することにし、
今はこれを認めて、nが偶数の場合のa,bを求める。
nが偶数なのにa≠±bかつa≠0かつb≠0であるようなa,bがあったとする。
n=2mと表せば、(a+bi)^n=(a^2−b^2+2abi)^mとなる。ここで、
A=a^2−b^2, B=2ab とおけば、A,Bもまた「A≠±BかつA≠0かつB≠0」を
満たす。実際、A≠0かつB≠0は明らかである。A≠±Bの方は、
A= B ⇔ a^2−b^2= 2ab ⇔ (a−b)^2=2b^2 ⇔ a−b=±b√2 ⇔ a−b=0かつb=0 矛盾
A=-B ⇔ a^2−b^2=-2ab ⇔ (a+b)^2=2b^2 ⇔ a+b=±b√2 ⇔ a+b=0かつb=0 矛盾
より、成立。
以上より、(a+bi)^n=(a^2−b^2+2abi)^m=(A+Bi)^mについて、mは奇数としてよい。
なぜなら、もしmが偶数のときは、m=2m',A'=A-2−B^2,B'=2ABなどと置けば
上の議論を繰り返すことができ、いずれ奇数に辿り着くからである。
そして、奇数のときの解はA=0またはB=0に限られるのだから、これは矛盾する。
697132人目の素数さん:2009/03/31(火) 22:08:09
>>690
遅すぎるんだTYO!
一応過去ログから問題と解答を再掲してやろう。

240 名前: 名無し 投稿日: 01/10/10 00:21
有名問題ですが,誘導付きにしてみました。これなら文科の問題としても
使えるか?理科には易しすぎ?解いたことのない人は解いてみて。
誘導も含めて講評して。

nを2以上の自然数とする。
(1) 2^k≦n<2^(k+1)となる自然数kを考える。1,2,・・・,nの中に2^kの倍数は何個あるか。
(2) 1,2,・・・,nの最小公倍数をSとする。S,S/2,S/3,・・・,S/n の中に奇数は何個あるか。
(3) 1+1/2+1/3+・・・+1/n は整数とならないことを示せ。

241 名前: 理T志望 投稿日: 01/10/10 01:37
>>240
(1) 2^(k+1)=2*2^k なので、2^k以上2^(k+1)未満の整数のなかで
 2^kの倍数は2^k自身しかない。
 2^k未満の自然数のなかに2^kの倍数はないので、
 よって答は1個。
(2) (1)のkをもちいると、Sは、
  S=(2^k)*(奇素数の積)
 とあらわされる。よって、1≦m≦nを満たす自然数mに対して、
  S/m が奇数 ⇔ m が2^kの倍数
 となるが、(1)の結果からこのようなmは1つだけ。
 ∴答は1個。
(3) 与式の分母をSで通分すると、分子は
 S+S/2+S/3+・・・+S/n であり、(2)からこれは奇数となる。
 一方Sは偶数なので、SはS+S/2+S/3+・・・+S/n の約数ではない。
 ゆえに与式は整数にはならない。(証終)
698132人目の素数さん:2009/03/31(火) 22:27:05
それ、高校生のときエルデシュの伝記みたいなやつに問題だけ載ってたな。懐かしい。
当時の俺はこうやって解いたぞ。

n≧2のときSn=奇/偶 となることを、数学的帰納法で証明する。n=2,3のときは
明らかに成り立つ。n≦2k のとき成り立つとすると、n=2k+1のときは、
Sn={1+1/2+…+1/(2k)}+1/(2k+1)=(奇/偶)+1/(2k+1)=(奇/偶)+(1/奇)
=(奇・奇+偶)/(偶・奇)=奇/偶 となり、成立。また、n≦2k+1のとき成り立つと
すると、n=2k+2のときは、

Sn={1+1/2+…+1/(2k+1)}+1/(2k+2)
={1+1/3+1/5+…+1/(2k+1)}+{1/2+1/4+…+1/(2k+2)} (この分け方がミソ)
=(整/奇)+(1/2)*{1+1/2+…+1/(k+1)}
=(整/奇)+(1/2)*(奇/偶)
=(整/奇)+(奇/偶)
=(整・偶+奇・奇)/(偶・奇)
=奇/偶
となり、やはり成立。
699132人目の素数さん:2009/04/01(水) 00:21:07
帰納法じゃなくて直接証明もできるな
700132人目の素数さん:2009/04/01(水) 14:06:18
>>696の続き。

nを自然数とする。整数係数多項式Tn(x)は、任意のx∈Rに対して
Tn(cosx)=cos(nx)を満たすとする(チェビシェフの多項式)。
Tn(x)の最高次の係数は2^(n−1)である。また、nが奇数のときは
Tn(0)=0だから、Tn(x)=xfn(x)なる整数係数多項式f(x)が取れる。…(*)
(f(x)とは書いたが、これはnに依存して決まるので、本来はfn(x)と
書いた方がよい。)


STEP2:nが奇数のときのa,bを求める。
a+bi=re^{ix}と極座標表示すれば、cosx=a/√(a^2+b^2)だから、cos(2x)=2cos^2x−1
=2a^2/(a^2+b^2)−1となり、cos(2x)は有理数となる。
(a+bi)^nが実数になるための必要十分条件は、x=kπ/n となるk∈Zが存在することである。
このようなkに対して、Tn(cos(2x))=cos(2nx)=cos(2kπ)=1 だから、c=cos(2x)とおけば
Tn(c)=1である。これと(*)より、cf(c)=1を得る。
cは有理数だったから、c=q/p (p,qは互いに素な整数)とおけて(q/p)f(q/p)=1を得る。
f(x)の次数をm≧0とすれば、r:=f(q/p)*p^mは整数になり、q*r=p^(m+1)となる。
この式からq|p^(m+1)となるから、pとqが互いに素であることより、q=±1となるしかない。
よってc=1/p となり(本当は±1/pだが、マイナスがつくときは−pを改めてpと置く)、
(1/p)f(1/p)=1となる。ここで、f(x)=Σ[i=0〜m]ai*x^iと表し、(1/p)f(1/p)=1の分母を
払って整理するとΣ[i=0〜m]ai*p^(m−i)=p^(m+1) となる。両辺をmod pで考えると
am≡0 (mod p)となる。すなわちp|amとなる。チェビシェフ多項式の最高次の係数は2のベキ乗
だったから、amは2のベキ乗であり、これとp|amより、pもまた2のベキ乗である。
つまり、あるk≧0に対してc=±1/2^k となる。
701132人目の素数さん:2009/04/01(水) 14:15:37
STEP2続き:まず、c=1/2^kのとき。c=cos(2x)=2cos^2x−1=2a^2/(a^2+b^2)−1だったから、
これとc=1/2^kより、式を整理して(2^k−1)a^2=(1+2^k)b^2…(**)となる。この式から
(2^k+1)|(2^k−1)a^2が分かるが、(2^k+1)と(2^k−1)は互いに素だから、(2^k+1)|a^2となる。
よってa^2=(1+2^k)sなる整数sが取れる。これを(**)に代入してb^2=(2^k−1)s となる。よって、
(ab)^2=(4^k−1)s^2となり、ab=±s√(4^k−1) となる。もしk≧1なら、4^k−1は平方数に
ならないから(***)、√(4^k−1)は無理数となり、よってab=0かつs=0となり、よって特に
「a=0またはb=0」を得る。k=0のときはab=±s√0=0となり、やはり「a=0またはb=0」を得る。

(***):4^k−1が平方数だとすると、4^k−1=y^2なる整数yが取れるはずだが、両辺をmod 4で考えて
−1≡y^2 (mod 4)となる(k≧1なので)。一方、y^2≡0,1 (mod 4)にしかならないので、矛盾。

あとはc=−1/2^kの場合を考える。上と同様にして、適当な整数sに対してa^2=(2^k−1)s,
b^2=(2^k+1)sとなるから、(ab)^2=(4^k−1)s^2となり、同様にしてa=0またはb=0に辿り着く。■
702691:2009/04/02(木) 00:04:40
>>696,>>700-701
凄い。cosで考えるのが突破口だったとは。
tanθ=b/aとして、例の「tan1°は有理数か」と同じようにtanの加法定理と
数学的帰納法でいけないだろうかと考えていました。
703696:2009/04/02(木) 02:29:42
間違い発見(^o^)修正します。
× nが奇数のとき:a=0またはb=0
○ nが奇数のとき:b=0
証明には ほとんど影響は無いと思う。


>>702
693が気になって調べてみたんだが、確かに京大に出ていた。

「pを素数、a, b を互いに素な正の整数とするとき、(a+bi)^pは実数ではないことを示せ。」
ttp://www.kyoto-math.jp/2000-4.html

で、リンク先の解答だと、チェビシェフの多項式なんて出てなくて、もっと初等的に解いている。
この結果を元にすると、「nが奇数のとき:b=0」となることが696みたいなやり方で証明できて、
nが偶数のときは696自身を使えばよくて、結局、チェビシェフの多項式を使わずに解けるという・・・
704696:2009/04/02(木) 03:05:18
まだ間違いが・・・

× nが偶数のとき:a=±bまたはa=0またはb=0

○ nが偶数だが4の倍数ではないとき:a=0またはb=0
   nが4の倍数のとき:a=±bまたはa=0またはb=0

証明は、>696の方に影響が出てしまうが、ちょっと修正すればすぐに直る。
705132人目の素数さん:2009/04/05(日) 14:37:50
イケメン東大生と変態したい。
706132人目の素数さん:2009/04/06(月) 23:57:46
漸化式a_1=a_2=1、a_{n+2}=a_{n+1}+a_{n}(nは自然数)で定められる数列について、a_{100}の桁数を求めよ。
但し、0.3010 <log_{10} 2 < 0.3011、0.4771 < log_{10} 3 < 0.4772、0.8450 < log_{10} 7 < 0.8451とする。
707 ◆Iyzrks/CZM :2009/04/07(火) 02:40:14
フィボナッチ
708132人目の素数さん:2009/04/07(火) 02:42:55
So what?
709132人目の素数さん:2009/04/07(火) 03:22:21
>>707の一般項を二次方程式の解の公式なりなんなりつかって出して
その第百項の対数をとるだけだろう
710KingGold ◆3waIkAJWrg :2009/04/07(火) 04:40:44
Reply:>>705 取引場所をMailで連絡せよ。
Reply:>>709 黄金比が出てくるが、それはどうするか。
711132人目の素数さん:2009/04/09(木) 18:26:33
>>706
φ=(1+sqrt(5))/2 とすると、この数列(フィボナッチ数列の一般項)は
a(n)={φ^n-(-φ)^(-n)}/sqrt(5) である。

nが大きいとき(-φ)^(-n)→0だから、まずb(n)=φ^n/sqrt(5)の桁数を評価する。

底の10は省略。
1.6<φ<1.62だから、
4log2-1<logφ<log2+4log3-2
0.2040<logφ<0.2099

log b(100)=100logφ-(1-log2)/2だから、
20.0505<log b(100)<20.64055
よって、b(100)は整数部分が21桁の数である。

一方、
a(100)=b(100)-(1/sqrt(5))(1/φ)^100>b(100)-1 であるが、
10^0.0505 >2^(1/6)>1.1 (∵0.0505*6=0.303>log2、1.1^6=1.771561)
だからb(100)>1.1*10^20であり、高々1を引いても桁数は下がらない。

∴ a(100)は21桁の整数 (答)
712706:2009/04/09(木) 23:38:18
>>711
素晴らしい!正解です。
713132人目の素数さん:2009/04/10(金) 02:21:17
>>712
後半(a(100)とb(100)の桁数が同じことを示す)は必要ですか?
以前、4^n+3の桁数についての問題を見たことがあって、
4^nの1の位は4,6の繰り返しだから3を加えても絶対繰り上がらない
→4^nと4^n+3は桁数が同じ、というところを厳密にやっていたので。
714132人目の素数さん:2009/04/10(金) 06:59:15
>>713
必要だと思います。まあほとんど自明ですが。
715132人目の素数さん:2009/04/10(金) 14:37:09
>>713
今の場合は0 < (-\phi)^{-100}/\sqrt{5} < 1で、a_{100}が整数だから、
\phi^{100}/\sqrt{5}の整数部分がa_{100}と論じてもっと簡単に
話を済ますことはできそうですね。
716132人目の素数さん:2009/04/16(木) 19:15:56
10^n-3の形で表される整数で、素数でないものはあるか。
717132人目の素数さん:2009/04/16(木) 22:07:17
関数f(x)=log2{x+√(x^2‐4)}‐1 について
f(x)=100をみたすxの整数部分の桁数を求めよ。
ただし、log10 2=0.301とする。
718132人目の素数さん:2009/04/16(木) 22:50:17
>>716
10=7+3だからn=7のとき素数じゃないな。
719132人目の素数さん:2009/04/16(木) 22:51:39
>>716
9997/13=769
720132人目の素数さん:2009/04/16(木) 23:43:54

確率1/100の宝くじを10回引いたときの当たる確率はいくつ?


721132人目の素数さん:2009/04/16(木) 23:49:14
>>720
こういう問の書き方を見ると、1/100はなんの確率なのか、とまず訊いてみたくなる。
722132人目の素数さん:2009/04/17(金) 07:29:55
いやオランウータンビーツでしょ
723132人目の素数さん:2009/04/17(金) 10:56:19

血液型分布は
近親婚がない、
且つ、ランダムに婚姻する、
という条件において、

最初の分布比率が保たれ
世代の更新により
変化しないことを証明しなさい。


生物屋のいうには変わらないらしい。
724132人目の素数さん:2009/04/17(金) 12:24:20
それは高校一年の生物でやるぞ
馬鹿なの?
725132人目の素数さん:2009/04/17(金) 18:32:20
問題の前提が不明だな
最初A型とB型しかいない場合は、途中でAB型が生まれてくるから、分布が変わるんだけど
726132人目の素数さん:2009/04/17(金) 22:19:38
>>723
その法則は Hardy の名前が付いている。
727132人目の素数さん:2009/04/17(金) 22:45:16
Hardy-Weinberg の法則は遺伝子の分布についての法則だから、
血液型の分布にはあてはまらないんだが……
遺伝子と表現型を混同してないか?
728132人目の素数さん:2009/04/17(金) 22:55:37
>>727
そうね。MN 型の話だった。
729132人目の素数さん:2009/04/17(金) 23:14:25
>>727
遺伝子頻度から遺伝子型の頻度が説明できるという話じゃないの。
730132人目の素数さん:2009/04/17(金) 23:33:56
>>729
何言ってるのかよく分からんけど、遺伝子の頻度は変わらなくても、
血液型の頻度は変わり得るでしょってことなんだけど
731132人目の素数さん:2009/04/18(土) 01:43:58
>>725
おいおいw

と思ったが、何事においてもある事柄のルールを知らない人間だと
その事柄の注目すべきレベルがわかんないのは当然か…
確かに数学的でもないし、前提が説明不足だわな
732132人目の素数さん:2009/04/18(土) 01:49:06
近親婚てどこまでが近親なんだよ
>>723がもし成り立つのなら別にこの仮定いらなそうな希ガス
自分以外を近親としないとしたときとかもね
なんとなくだけど
733132人目の素数さん:2009/04/18(土) 02:58:12
最初の世代の比率がA型とB型1:1だったら、子孫もAとB1:1ってこと?
734132人目の素数さん:2009/04/18(土) 14:13:00
>>716

n=1+6m, n=4+6m, n=11+16m, n=5+18m (m≧0) など。

(略証)
 10^6 = (10^3 -1)(10^3 +1) +1 = (10^3 -1)・7・11・13 +1 ≡ 1 (mod 7・13)
 10^16 = (10^8 -1)(10^8 +1) +1 = (10^8 -1)・17・5882353 + 1 ≡ 1 (mod 17)
 10^18 = (10^9 -1)(10^9 +1) +1 = (10^9 -1)・19・52631579 + 1 ≡ 1 (mod 19)
より
 10^(1+6m) -3 ≡ 10^1 -3 = 7 ≡ 0 (mod 7)
 10^(4+6m) -3 ≡ 10^4 -3 = 13・769 ≡ 0 (mod 13)
 10^(11+16m) -3 ≡ 10^11 -3 = 17・5882352941 ≡ 0 (mod 17)
 10^(5+18m) -3 ≡ 10^5 -3 = (19^2)・277 ≡ 0 (mod 19)
735132人目の素数さん:2009/04/18(土) 14:55:29
>>733
そういう意味にしか読めんわな
736132人目の素数さん:2009/04/20(月) 15:09:47
3次元直交座標空間に球面Sがある。この球面S上の任意点(p、q、r)について、p、q、rのうち2つの数字が整数ならば残りの1つも必ず整数である。
このような球面Sは何種類あるか。その半径として考えられるものをすべて求めよ。
ただし、球面Sは格子点を少なくとも1つ通るとする。
737132人目の素数さん:2009/04/20(月) 19:18:10
m:整数、n:1<n<100の整数の時
n!=m^2
となるn,mの組は存在しないことを示せ。

正直、きれいな証明じゃないので微妙。

ちなみに、これはベルトラン予想の限定。
実際は2以上のすべての自然数nにおいて上の式が成り立ちます。
738132人目の素数さん:2009/04/20(月) 21:40:43
>>736

 (x-1/2)^2 + y^2 + z^2 = 1/4,       半径 1/2,
 (x-1/2)^2 + (y-1/2)^2 + z^2 = 1/2,    半径 (√2)/2,
 (x-1/2)^2 + (y-1/2)^2 + (z-1/2)^2 = 3/4 or 11/4, 半径 (√3)/2 or (√11)/2,
 x^2 + y^2 + z^2 = 1,           半径1,
739132人目の素数さん:2009/04/20(月) 23:47:40
>>737
n!=m^2について
pを素数として
p!の素因数のうち最大の素数はpである。
p^2までにpより大きい素数が含まれていることが示せれば最大の素数の次数が1であり、平方数にならないことが示される。
またこの時pの次に大きい素数をqとして、p≦n<qである自然数nについても最大の素数はpである。…@
p=2ならp^2=4までに3がある
同様に
3(9)→7
7(49)→47
47(47^2)→101
(5は7,11〜43は47,53〜97は101が対応する)
よってp!が平方数になることはない。(1<p<100)
また、これと@より1<n<100についても示される。
740737:2009/04/21(火) 03:48:45
>>739
大体あってるけど、
>p^2までにpより大きい素数が含まれていることが示せれば最大の素数の次数が1であり、平方数にならないことが示される。
p^2じゃなくて2p。
なので、実際は
2 3 5 7 13 23 43 83 というステップになる。
741132人目の素数さん:2009/04/24(金) 01:46:23
・ 1/2 < e^π-π^e < 3/4 を示せ。
・ 納n=1〜∞]n^(-3) > 6/5 を示せ
742132人目の素数さん:2009/04/24(金) 08:31:10
こういうのは意外に高校生にはキツいかも
aを実数とする
xの方程式
x^5-x^4+ax^3+ax^2‐x+1=0
の5つの解のうち、少なくとも2つの解が一致する。
このとき、aの値及び、一致する解を求めよ。
743132人目の素数さん:2009/04/24(金) 08:44:17
メロンパンナのメロメロパ〜ンチ
だな。
ついでに問題の書き方がおかしいな。
>少なくとも2つの解が一致する
って実際そうなるか解答者側から見ればまだ分からないことジャン。
出すんだったら保証させることも含めろよ。
744132人目の素数さん:2009/04/24(金) 10:44:04
≫743
・・・2つの解が一致するとき、aの値及び・・・
でいいですか?

文章おかしくてスマソ
745132人目の素数さん:2009/04/24(金) 17:16:27
いや、そうじゃなくて、そもそも
「方程式
x^5-x^4+ax^3+ax^2‐x+1=0
の5つの解のうち、少なくとも2つの解が一致する」
ような実数aはあるのか、ということも含めて出題する
(これはa=0とすれば問題ないことだが)。
そして、どうせやるなら、「もしあれば」、2つの解が一致するようなaの値と
そのときの一致する解だけでなく「他のすべての解」を、それぞれすべて求めさせる。
こうした方がいい。
746132人目の素数さん:2009/04/24(金) 17:23:36
>>742の問題文が変だとは思わないが簡単すぎる
747132人目の素数さん:2009/04/24(金) 19:04:10
≫745
成程、確かにその方が親切ですね。
≫746
今の自分なら解けますが、高校時代なら最後までは
解けないな〜、って感じたので出題してみました。
748132人目の素数さん:2009/04/24(金) 21:12:11
xy平面上の2つのベクトルをp,qとし、p,qのなす角をθとするとき、次の式を示せ
p・q=|p||q|cosθ
749749:2009/04/24(金) 23:54:02
7=√(49)
750132人目の素数さん:2009/04/25(土) 00:38:26
>>744
なんで「重解をもつとき」ってしないの?
751132人目の素数さん:2009/04/25(土) 10:09:21
≫750
問題文より自明ですが、解く過程で重解となることを認識させたいので。
752132人目の素数さん:2009/04/25(土) 13:55:08
753132人目の素数さん:2009/04/25(土) 16:12:02
>>751
ふーん、重解の定義って、なんか小難しいわけ?
754132人目の素数さん:2009/04/26(日) 11:40:31
≫753
ここの住人は厳しいですね(^o^;
自分としては高次方程式を題材に、重解条件を考えさせたく
以下の2点をポイントとして出題してみました。
@式の特徴に着目して、解の候補を考える
Aより低次の方程式に帰着させる

問題文が不完全なのは認めますので、これくらいで
ご勘弁をm(__)m
ちなみに誰か解答お願いします。
755742:2009/04/26(日) 12:03:41
因みにもう一つ作ってあります。
これも同じような問題です。
a、bを実数として
f(x)=x^2+ax+b、g(x)=f(f(x))とおく。
@方程式g(x)-x=0が重解をもつとき、a、bの関係式を求めよ。
Aa、bをa>bをみたす整数とする。
方程式g(x)-x=0の重解も含めた4つの解のうち、
2つは整数で残りの2つが虚数となるとき、a、bの値を求むよ。
756132人目の素数さん:2009/04/26(日) 18:12:29
>>754
 >>742 の解答
 (左辺) = (x+1){(x^2 +1)(x-1)^2 +a・x^2},

 a=0 のとき 1(重根), -1, ±i,   (左辺) = (x+1)(x^2 +1)(x-1)^2,
 a=-8 のとき -1(3重根), 2±√3,  (左辺) = (x+1)^3 (x^2 -4x+1),
かな?
757132人目の素数さん:2009/04/26(日) 22:22:04
>>741

Σ[n=1,∞) 1/(n^3) = 1 + (1/8) + (1/27) + (1/64) + (1/125) + (1/216) + (1/343) + Σ[n=8,∞) 1/(n^3)
  > 1 + (1/8) + (1/27) + (1/64) + (1/125) + (1/216) + (1/343) + Σ[n=8,∞) 1/n(n+1)(n+2)
  = 1 + 3・4771443/(N^3) + Σ[n=8,∞) {1/[2n(n+1)] - 1/[2(n+1)(n+2)]}
  = 1 + 3・4771443/(N^3) + 1/(2・8・9)
  = 1 + 3・4942943/(N^3)
  = 1 + (1/5) + 3・3743/(N^3)
  > 1 + (1/5),
ここに N = 3・4・5・7 = 420 とおいた。
758132人目の素数さん:2009/04/27(月) 00:00:24
≫756
そこまでは正解です。
ここで、g(x)=(x^2+1)(x-1)^2+ax^2と置き
g(x)=0の4つの解のうち、少なくとも2つの解が一致する
場合を考えなければなりません。
759132人目の素数さん:2009/04/27(月) 01:00:31
相反方程式にするんだよ。
760132人目の素数さん:2009/04/27(月) 21:34:39
>>758
 g(x) = (x^2 +1)^2 -2x(x^2 +1) + ax^2 = (x^2 -x+1)^2 + (a-1)x^2,

 a=1 のとき (1±i√3)/2 (重根), -1. (左辺) = (x^2 -x+1)^2,

761132人目の素数さん:2009/04/27(月) 23:18:04
>>741
 y=1/(x^3) は 下に凸なので、台形公式
 (1/2){1/(k^3) + 1/(k+1)^3} > ∫[k,k+1] 1/(x^3) dx,

Σ[n=1,∞) 1/(n^3) > 1 + 1/(2^3) + 1/(3^3) + 0.5/(4^3) + ∫[4,∞) 1/(x^3) dx
 = 4043/(128*27) + [ - 1/(2x^2) ](x=4,∞)
 = 1.169849537037・・・・ + 1/32
 = 1.201099537037・・・・
762132人目の素数さん:2009/04/28(火) 02:21:50
f_1(x) g_1(x)をx>0に対して定義された関数とし、任意の自然数nに対してf_{n}(x),g_{n}(x)を
f_{n+1}(x)=xf_{n}(x)-e^{x}g_{x}, g_{n+1}(x)=f_{n}(x)-(logx)g_{n}(x)
によってx>0に対して定義する。
このとき、f_1(x) ,g_1(x)の共通解が存在しないとすると、任意の自然数nに対してf_{n}(x),g_{n}(x)の
共通解も存在しないことを示せ。
763762:2009/04/28(火) 02:29:07
訂正 f_1(x) ,g_1(x)の共通解→f_1(x)=0 ,g_1(x)=0の共通解
764132人目の素数さん:2009/04/28(火) 02:30:27
>>762
関数の共通解とは?

ま、共通零点、のことかな、とは思うが。
765132人目の素数さん:2009/04/28(火) 02:32:09
>>763
わざわざ訂正するなら、4行目の後半もしておかないと。
766762:2009/04/28(火) 02:40:42
訂正
f_{n+1}(x)=xf_{n}(x)-e^{x}g_{n}(x)→f_{n+1}(x)=xf_{n}(x)-g_{n}(x)
f_{n}(x),g_{n}(x)の共通解→f_{n}(x)=0,g_{n}(x)=0の共通解
>>762のままじゃ全然面白くないです。
767132人目の素数さん:2009/04/29(水) 10:11:40
≫760
正解です。
自分の用意した答えはまとめると
a=8、0、1 で各場合の一致する解はそれぞれ
x=-1、1、(1±√3i)/2ですやっぱり簡単過ぎた様です。
768だいすけ@31.com:2009/04/29(水) 14:59:35
東大文3出身の元学コンマンです。(わけあって中退したので文3か理1か慶応SEC環境情報学部に入りなおそうかと思案中)
それはともかく・・・なんとなく問題つくってみたけど、自分で解けてない...
直線Lと半径1の球Sがあり、直線Lと、球Sとの最短距離は1である。
直線Lを軸として球Sが1週したときの、球Sの軌跡をMとする。(いわゆる円環)
また、Mと、Mの内側の空白を合体させた領域をNとする。

(1)動点A、BがMに含まれるとき、線文ABの最大長を求めよ。
(2)動点A、BがNに含まれるとき、線文ABの最大長を求めよ。
(3)正四面体FがNに含まれるとき、正四面体Fの1辺の最大長を求めよ。
(数学久し振りにやったので、言葉を忘れてしまって文章が分かりづらくてすみませんSはいわゆる円環(ドーナツ)、Tは、ドーナツ+ドーナツの穴の領域です)
てか、高校数学(今の高校数学の履修範囲をわかってないです)で解けないかも
769132人目の素数さん:2009/04/29(水) 16:16:03
出題ってのはちゃんと意図をもってやるべきもので、
自分でどういう工夫をすべきかすら把握できてないのに出しても意味無いだろ

問題を(1)(2)に分けたのはどういう状態を想定したんだ?そしてこんなシンプルな問題がなぜ解けない?
それに自分が使ってる文字すら把握できてないぞ
>Sはいわゆる円環(ドーナツ)、Tは、ドーナツ+ドーナツの穴の領域です)
770132人目の素数さん:2009/04/29(水) 17:18:49
さすが文系
771132人目の素数さん:2009/04/29(水) 17:29:11
>>768
細かいことだけど、誤字脱字が多いな
772132人目の素数さん:2009/04/29(水) 17:55:36
>>768
下手したら中学数学でも解けるんじゃね?
773132人目の素数さん:2009/04/29(水) 23:03:49
こういうのは如何ですか?
平面上に、|x↑|=|y↑|=1の2つベクトルx↑、y↑があり
x↑、y↑のなす角θに対して、tanθ/2は無理数である。
整数m、nに対してp↑=mx↑+ny↑で表される点の集合をSとする。
Sから4点を選び4角形をつくるとき、正方形は作れない
ことを証明せよ。

どっちかと言うと京大好みかも。
774132人目の素数さん:2009/04/29(水) 23:20:47
>>773
どのように選んでも、とするのが京大流。

京大の問題文はくどい。
775132人目の素数さん:2009/04/30(木) 10:51:11
>>773
tanθ/2ではtan(θ/2)の意味か(tanθ)/2の意味か分かりません
776132人目の素数さん:2009/04/30(木) 12:03:54
≫775

tan(θ/2)です。
777132人目の素数さん:2009/04/30(木) 14:55:41
>>775
カス市ね
778132人目の素数さん:2009/04/30(木) 16:28:43
>>773の問題は下記の文章に似てる。

テラキモんだ?今週なんだ?今週のサザエさんはこんがるか?っつーの
すでに運知思想だろーがっつーの
今から思うと、1969年のアポロがターニングポイントだったね
神国日本の威厳を体現するのは私のお稲荷さんであるということです
クラフトワークがテクノの元祖ということですッチんチン
こんな糞スレで、森尾由美好きが多数派の勝ち組であり、
ロリコンは少数派の負け組という事実が判明するとはな。
ネットの力はおそろしい。
無駄な抵抗はよせ
貴様の包囲網はジワジワと、こんなスレッドにまで迫っているぞ。
779132人目の素数さん:2009/05/03(日) 22:54:07
質問ですが、東大の98年後期の大問3をその当時に解いた予備校教師はいたのでしょうか?
780132人目の素数さん:2009/05/04(月) 03:11:57
放物線y=x^2+5と円x^2+y^2=2と直線Lを考える。放物線とLは異なる2点で交わり、円とLも異なる2点で交わっている。
・Lのy切片が整数
・放物線とLで囲まれる部分の面積が整数
・Lが円から切り取られる弦の長さが整数
の3つが成り立つとき、直線Lの方程式を求めよ。

東大文系の標準〜やや難 を想定し作問。
最後まで解ければ文系数学2完半か3完レベルかなぁ
781132人目の素数さん:2009/05/04(月) 09:10:53
1辺の長さが1の立方体がある。

ただし以下3つの条件を前提とする。
(A)この立方体において、頂点を選択するとき、各頂点とも選ぶ確率は等しいものとする。
(B 選択する頂点は何点でも重複してもかまわないものとする。
(C)長さ(問1の場合)、面積(問2の場合)、体積(問3、4、5)の値が、
   正なる値を持たない場合、上記の長さ/面積/体積を0とみなす。

(1)頂点を2つ選ぶ。これらを端店とする線分の長さの期待値を求めよ。

(2)頂点を3つ選ぶ。これらを頂点とする3角形の面積の期待値をもとめよ。

(3)頂点を4つ選ぶ。これらを頂点とする立体の面積の期待値をもとめよ。

(4)頂点を5つ選ぶ。これらを頂点とする立体の体積の期待値をもとめよ。

(5)頂点を6つ選ぶ。これらを頂点とする立体の体積の期待値をもとめよ。
782781:2009/05/04(月) 09:27:14
ごめん、問題訂正。

(3)、(4)、(5)で「立体」と書いてあるところを、それぞれ、
四面体、五面体、六面体 に変えてください。

∵たとえば(4)で、2点だけ重複すると、
(問題の文章を読むと)立体が四面体と見なすことができるかもしれいから、
783132人目の素数さん:2009/05/04(月) 10:23:28
そういう、ただ労力をかければすむようなひねりのない問題を
しかも自分で吟味もせずに出題するのはやめようぜ。
センターレベル以下じゃん

>(3)、(4)、(5)で「立体」と書いてあるところを、それぞれ、
>四面体、五面体、六面体 に変えてください。
変える意図がわからん
どういう出題意図なのか知りたい(あるのなら)

しかも、
そこまで考えずに頂点重複を避けさせようとして
頂点の数と面の数は一致するわけじゃないことに気付かず
勘違いしてるっぽい浅はかさを感じる
784132人目の素数さん:2009/05/04(月) 12:04:47
>>782
よく分からんが……頂点6の六面体ってどんなの?
五角錐しか思い浮かばん。
785132人目の素数さん:2009/05/04(月) 12:29:48
>>784
>>782が検討してない証拠だろう

>(B 選択する頂点は何点でも重複してもかまわないものとする。
を踏まえれば、立方体のうちの頂点5つで6面体にはできるが
6面体限定にせよ,6頂点限定にせよ
>体積(問3、4、5)の値が、正なる値を持たない場合、上記の長さ/面積/体積を0とみなす。
わざわざこんな条件つけてまで期待値の問題にしてる意味がなくなるんだよね。

いろんな意味でお粗末
786781:2009/05/04(月) 12:35:18
あれ、問題文の日本語変だった?

>>783

>(3)、(4)、(5)で「立体」と書いてあるところを、それぞれ、
>四面体、五面体、六面体 に変えてください。
===
>そこまで考えずに頂点重複を避けさせようとして

そじゃなくて、逆。頂点重複もOKとする。
三角形の場合、3点が、A、A,Bなら、面積は0という次第。

出題意図は、簡単にいうと、「確率」と「面積/体積」が融合すると、
>ただ労力をかければすむようなひねりのない問題
でなく、労力かけずにスマートに解ける問題、ってこと。

>頂点の数と面の数は一致するわけじゃないことに気付かず
もちろん気づいてた。そこでひかっかるかなと思ったが簡単?

>>784
やっぱ、頂点が重複すると、六面体の体積は正なる値にならない。
(小学生レベルの言い方すると、「頂点が重複すると、六面体にはなりえない」)

だから、「頂点6の六面体ってどんなの?」って、題意の立方体そのもの鹿「あり得ない。

まあ、(1)と(5)は、ネタ的問題で、即効解ける。
787781:2009/05/04(月) 12:42:44
あ、そか、

>>782 勘違いしてた。
「頂点が重複しない条件のもとでの立体」
って意味のつもりだったんだけど、>>782 の書き方だと意味が異なる。ミスった。
788132人目の素数さん:2009/05/04(月) 12:45:19
立方体は頂点が6つですと言ってるようにしか見えないんだが。
789132人目の素数さん:2009/05/04(月) 12:49:16
>>786
やっぱいろんな意味で浅はかだな。

>そじゃなくて、逆。頂点重複もOKとする。
だったら
>(問題の文章を読むと)立体が四面体と見なすことができるかもしれいから、
こんなことを書く理由がないんだよね。

>やっぱ、頂点が重複すると、六面体の体積は正なる値にならない。
最低限確認はしようよ。>>785にも挙げてあるんだしさ。
そもそも「正なる値」ってのがまずおかしい。よほど数学になじみがない人ですかねえ。

で、堂々と間違いを晒しておいて
>もちろん気づいてた。そこでひかっかるかなと思ったが簡単?
この負け惜しみ開き直りはすごいね。
790132人目の素数さん:2009/05/04(月) 12:50:17
どつぼですな・・・
791132人目の素数さん:2009/05/04(月) 13:06:49
ごめんなさい。コーヒーブレイク。

===

ナベアツ君は、数字を言ったとき、その数字が、
3の倍数または3の付く数字のときだけアホになる。

1から初め、10^n まで数字を数えて言うとき、アホになる回数を求めよ。

※ただし、連続してアホになる場合、1つの数字ごとにアホ回数をカウントする
(30、31、32・・・39 のとき、ずっとアホだからw)

===

っていうか、既出?
でも解き方が数通りあると思われ。
792132人目の素数さん:2009/05/04(月) 13:07:29
逆に>>781向けの練習問題が必要だな
中学内容の空間図形や確率の初歩がわかってないようだから

(1)立方体の8つの頂点のうち異なる5つを選んでできる多面体は何通りあるか求めよ
(合同な立体は1通りと見なす)

(2)異なる5つの頂点をランダムに選ぶとき、できる多面体が六面体になる確率を求めよ
793132人目の素数さん:2009/05/04(月) 13:09:06
スレタイ読めない人が増えたかな?
794132人目の素数さん:2009/05/04(月) 13:10:11
>出題意図は、簡単にいうと、「確率」と「面積/体積」が融合すると、
融合(笑)

確率論は測度論であって、測度論は「面積/体積」(及びその一般化)を扱う理論。
つまり、確率論は最初から面積や体積も普通に相手にしている(融合でも何でもない)。
無知すぎる。
795781:2009/05/04(月) 13:12:49
>そもそも「正なる値」ってのがまずおかしい。よほど数学になじみがない人ですかねえ。

「頂点を4つ選んだとき、4点全てが平面上に存在する場合は体積を0とみなす」
と書こうとしたけど、難しくしようとして、「正なる値」って言ってみたorz

数学じゃなくて国語を勉強します・・・
796132人目の素数さん:2009/05/04(月) 13:16:10
>>780
ほとんど整数問題だな
797132人目の素数さん:2009/05/04(月) 13:28:15
>>786
あぁ……少しずつ分かりかけてきたかも知れん。
あれかな、凸包をイメージすればいいのかな?

確率バージョンにすると、こんな感じか? でも簡単すぎる気がするんだが……

----


空間上に立方体Sがあり、Sの8頂点のうち、いくつかの頂点をランダムで選ぶ。
ただし以下の条件を前提とする。
(A)この立方体において、頂点を選択するとき、各頂点とも選ぶ確率は等しいものとする。
(B)選択する頂点は何点でも重複してもかまわないものとする。

(1)頂点を2つ選ぶ。これらの凸包が線分になる確率を求めよ。
(2)頂点を3つ選ぶ。これらの凸包が三角形になる確率を求めよ。
(3)頂点を4つ選ぶ。これらの凸包が四面体になる確率を求めよ。
(4)頂点を5つ選ぶ。これらの凸包が5面体になる確率を求めよ。
(5)頂点を6つ選ぶ。これらの凸包が6面体になる確率を求めよ。

798132人目の素数さん:2009/05/04(月) 13:28:37
>>795
ガンガレ
数学の言葉使いは国語の力のせいとは限らないよ
数学の問題に親しんでないとどんなに国語力つけてもボロが出る
テクニカルタームみたいなもんだし

まずは>>792が目標か
799132人目の素数さん:2009/05/04(月) 13:43:48
>>797

これは…
>>781以上に数学の苦手な人か?
背伸びして用語だけ調べたのか?
800132人目の素数さん:2009/05/04(月) 13:45:47
>>799
自分でも言ってることがおかしいと思うんだが、>>781がよく分からなくてね
801781:2009/05/04(月) 13:45:56
>>797 そうそう。それを、期待値の問題に変えたかった。
802132人目の素数さん:2009/05/04(月) 13:47:13
>>786
>だから、「頂点6の六面体ってどんなの?」って、題意の立方体そのもの鹿「あり得ない。
ってのは、どういう意味なの?
立方体は頂点が8個あると思うんだけど……
803132人目の素数さん:2009/05/04(月) 13:50:26
>>799
ちなみに、お前は何がおかしいと思ってるの?
804797:2009/05/04(月) 14:14:11
>>799
一応、どこら辺が数学苦手だと感じたのか説明してほしい。
ちなみに、言うまでもなく>>797の目的は>>781の俺的意訳であって、出題じゃないんでね。
それを踏まえたうえで説明してほしい。一応>>781本人は>>801で正しいといっているので、
俺の解釈に誤りはなかったわけだが……以上2点繰り返すと

・ 出題や解答が目的ではなく、あくまで>>781の翻訳が目的
>>801にて、>>781本人が正解であると言っている。

これらを踏まえて、どこら辺がおかしいと感じたか説明してもらえると助かる。
一応、目的から逆算すれば、ある意味完全正解だったわけだけど……(>>801
805797:2009/05/04(月) 14:16:04
ちなみに、どう考えても確率は明らかにxxだろ、的な回答は期待してない。
それは分かった上で>>797を書いているので。
806132人目の素数さん:2009/05/04(月) 14:40:24
問題文に不備があるからって攻撃的になるのはやめろ
807781:2009/05/04(月) 15:04:11
あーだめだ・・・。10年数学やってなかったから、ボロボロだ。
英語も、中学レベルの単語のスペル忘れるし・・・脳細胞死にまくり・・・
808132人目の素数さん:2009/05/04(月) 15:18:33
>>804
期待値の問題だったのに
期待値のカケラもない

これこそまさに>>801が言ってるね
809132人目の素数さん:2009/05/04(月) 15:27:50
それに、「確率バージョンにする」と宣言して
期待値の問題を自分でシンプルにしすぎておいて
「簡単すぎる気がする」はさすがにないと思うな

翻訳が目的なら問題の勘所を曲げるのはおかしいからね
もっとも>>781自身が出題意図を明確に持ってないから仕方ないといえば仕方ないが
それにしても>>781と同じくらい数学に対する感覚が鈍くない限り
ちょっとありえないと思う
810132人目の素数さん:2009/05/04(月) 15:36:55
>>789とか>>794は性格悪すぎだろう。人を見下すのが楽しいか?
811132人目の素数さん:2009/05/04(月) 15:45:57
>>794は誰か知らんが>>789は確かに人格否定しすぎだったね。言葉もすぎるかもしれない

ただ、出題者が出題意図もろくに持ってない投げっぱなしの上
指摘されてもどこがおかしいか気付けないようじゃ
あきれられたり見下されるのは仕方ないんじゃないかねえ。
>>786がなければここまで呆れ果てることはなかったと思うんだけど。
812132人目の素数さん:2009/05/04(月) 15:49:33
まあいいじゃないですか
807だって失敗だったと認めてるんだし
他人がズルズルとスレ違いの話題をひきのばして傷口を広げることはないでしょ
813132人目の素数さん:2009/05/04(月) 15:51:15
どうでもいいから>>780やってくれよ
814132人目の素数さん:2009/05/04(月) 15:56:16
ある自然数 x について、

===========================================
x が偶数であれば、x/2 を求める。
x が奇数であれば、(3x+1)/2 を求める。
===========================================

上記動作において、求めた数をxに置き換える。
そして、上記動作を永遠に繰り返す。

最初のxがいかなる自然数であっても、
上記動作を繰り返すと必ず、いつかは、x が1となることを示せ。
815132人目の素数さん:2009/05/04(月) 15:57:58
>>814
ヒントをくださいw
816132人目の素数さん:2009/05/04(月) 15:58:50
>>809
まずは出題者の意図を理解することが目的なのに……
自分でズレたことを言ってる自覚はないのか?
817132人目の素数さん:2009/05/04(月) 16:01:23
>>780

>Lが円から切り取られる弦の長さが整数
「円から切り取られる弦」は2つあるんだけど、どっちの長さが整数?どっちでもいいの?
818132人目の素数さん:2009/05/04(月) 16:04:09
>>816
本人かな?
何がズレてるのかわからない人にとっては
何をズレてるといわれてるのか自覚できないってやつだね。
平行線かなぁコレ。

では出題者>>801の意図とはなんですか?
理解していますか?
819132人目の素数さん:2009/05/04(月) 16:05:29
>>817
気はたしかですか?
820781:2009/05/04(月) 16:07:46
>>816

ごめんもう許してw

実は、某所で見つけた問題(過去問ではない)を拡張させようとして、間違った・・・
821132人目の素数さん:2009/05/04(月) 16:08:43
上塗りしすぎっしょ
822817:2009/05/04(月) 16:08:53
あ・・・弦を弧と読んでしまったw
823797:2009/05/04(月) 16:10:11
>>818
理解していない。
俺が>>797を書いた時点や、今の段階では情報が少ないため、断定できないが正しい。
ゆえに>>797を書いて、理解の助けにしようと考えた。

お前さんがもし>>809なら、逆になぜ、勘所がどーこーと分かったのか聞きたいところだ。
極端な話をすれば、出題時点では、781本人が書き間違いなどのミスをしている恐れすらありえるんだぞ。
その状態で、何が勘所かを判断する十分なソースがそろっていると判断したことのほうが不思議で仕方ない。


と書いているうちに、本人から>>820で出てきたな。
お前こそ、理解してたか?
824797:2009/05/04(月) 16:11:11
と思ったら、>>820は微妙に違うね。
まーいっか。
825132人目の素数さん:2009/05/04(月) 16:16:53
x^2 - [y]^2 = 20.09
y^2 + [x]^2 = 2009
を満たす実数x,yを全て求めよ。
826132人目の素数さん:2009/05/04(月) 16:17:15
>>780
直線の式をy=ax+m(m∈Z)とおく。
放物線とLで囲まれる面積S∈Nはこれらの交点の二つx座標α,βを用いて
S=1/6*(β-α)^3=1/6*((α+β)^2-4αβ)^(3/2)
=1/6*(a^2+4m-20)^(3/2)
⇔6S=(a^2+4m-20)^(3/2)(*)
a^2+4m-20> 0が放物線とLが異なる2点で交わる条件であることに注意しておく。

円で切り取られる弦の長さをl∈Nとする。
原点とLの距離が|m|/√(a^2+1)だから
(l/2)^2=2-m^2/(a^2+1)⇔a^2=4m^2/(8-l^2)-1(**)

ここで0<l<円の直径=2√2<3なのでl=1,2なので二通りに場合分けして考える。

1)l=1のとき
(**)よりa^2=4m^2/7-1 これを(*)に代入すると
6S=(4m^2/7+4m-21)^(3/2)
⇒36S^2=(4m^2/7+4m-21)^3
左辺は整数なので右辺も整数で右辺の()内も整数(∵整数でない有理数のベキもまた整数でない)
よって4m^2/7は整数でGCD(4,7)=1なので整数Mを用いてm=7Mとかけ、
36S^2=(28M^2+28M-21)^3 となるが左辺は偶数、右辺は奇数で不合理である。

2)l=2のとき
(**)からa^2=m^2-1 これを(*)に代入し
6S=(m^2+4m-21)√(m^2+4m-21)
ゆえに√(m^2+4m-21)は有理数、つまりm^2+4m-21は有理数の二乗である整数であるから、これは平方数である。
正の整数kを用いてm^2+4m-21=k^2⇔(m+2)^2-25=k^2⇔(m+2-k)(m+2+k)=25
これより容易に(m+2-k,m+2+k)=(1,25)⇔(m,k)=(11,12)だけ適することがわかる。
このときa^2=m^2-1=120からa=±2√30

以上からy=±(2√30)x+11が求める直線の式である。
827797:2009/05/04(月) 16:18:10
もうそろそろウザイんで俺もやめにするが、念のために一つ
>>797で簡単すぎると書いたのは、期待値バージョンでも簡単だと書いたのよ
念のためね

ま、かきまちがえたのはすまんかった
828132人目の素数さん:2009/05/04(月) 16:20:17
>>814
コラッツ予想だっけ?数論の超難問だよ。
829132人目の素数さん:2009/05/04(月) 16:25:00
コラッツ予想は解けるわけないが、少しもじるだけで高校生でも解ける問題に変わるのでたまに出題されてる。
けど、灯台レベルじゃ出てるのを見たことがない。
830791:2009/05/04(月) 16:26:29
だれか、>>791 解かない?
簡単すぎてスルーされてる?
831132人目の素数さん:2009/05/04(月) 16:27:05
専用スレがあった気がする。
832132人目の素数さん:2009/05/04(月) 16:27:15
>>823
何を理解しろというの?

結局わかったのは>>823が出題失敗してたってことだけでしょ。
最初から指摘してたけど。

>逆になぜ、勘所がどーこーと分かったのか聞きたいところだ。
ここってどういうスレでしたっけ?
数学の問題って、どういうコトをさせたくてそういう問題にしたかという
背景やストーリーや意図がちゃんとあるもんだと思うんだけどね。
そういうこと気にする意識すらないの?
(なさそうだから>>799のように感じたわけだけど)

あれば出題者が失敗してるのくらい誰でもわかるところじゃないかなあ。
よほど数学に疎いか、まだその単元になじんでなくて出題意図まで考えるところまで気が回らない人でない限り。
ミスでどうこうなる類の失敗じゃないでしょ。

でも失敗かどうか判断もできないから意訳などという無意味な>>797を書き込んでしまったんだろうし
いまだに>>823みたいなこと言ってると。


つーか>>820がいやがってるのにまだスレ違いの内容続けたいの?
ここは>>797を啓蒙するスレじゃないんだけど。
833814:2009/05/04(月) 16:28:42
>>828

あれ、そうだっけ。
だとしたら、

>>829
で言ってる<少しもじるだけで高校生でも解ける問題に変わるのでたまに出題されてる。>
ってのを前に見たかもしれない。
834132人目の素数さん:2009/05/04(月) 16:40:12
2008人の男子と2009人の女子が円周上に等間隔に座っている。
このとき間にちょうど1000人の人(男女どちらでもよい)が座っているような2人の女子の組が存在することを示せ。
835132人目の素数さん:2009/05/04(月) 16:42:59
>>826
正解。やるね〜。難易度どうだた?
ちなみに作るとき、円の半径をいくつにするかとか放物線をどうするかとかかなり手探りして、
それなりに解ける問題としてまとまったときは嬉しかった
「図形と方程式」「整数問題」の融合でした。
836132人目の素数さん:2009/05/04(月) 16:50:11
>>835
放物線や円の方程式がシンプル(大きな数字を使っていない)なわりに
答えが本質的に一通りに定まったから個人的にはかなり良問だと思った。
難易度的にも東大入試としてちょうどいいと思う。

作問のアイディアはいくつか出てきても解ける問題にしようと思ったら見た目が汚くなったりして
なかなかキレイな問題を作るのは難しいよね。
837132人目の素数さん:2009/05/04(月) 16:50:27
>>834
4017と1000が互いに素で
あとは鳩の巣だな
838132人目の素数さん:2009/05/04(月) 16:51:15
>>835
手探りw
839780:2009/05/04(月) 17:09:51
>>836
良問と言ってくれると嬉しいぜw

まあ俺東大落ちwww和田政経(数学受験)wwwだから大したことないけどwwww

またなんか思いついたら来るノシ
840132人目の素数さん:2009/05/04(月) 17:11:03
>>837
1000?互いに素?

>>834
2009年の問題にしたいのか知らんが
女子を何人まで減らせるかとかいう問題くらいにしないとつまらんでしょ
841132人目の素数さん:2009/05/04(月) 17:37:56
x,y,zはどの2つをとっても互いに異なる整数であり、x,y,zの最大公約数は1である。
yx^2 + zy^2 + xz^2 がxyzで割り切れるとき、ある整数a,b,cが存在しx=ab^2、y=bc^2、z=ca^2とあらわせることを示せ。
842132人目の素数さん:2009/05/04(月) 17:52:35
>>841
x,y,zのいずれかが0だとxyzで割るが定義されないのでどれも0でないとする。
xyz|yx^2+zy^2+xz^2
より
x|yx^2+zy^2+xz^2⇒x|zy^2
GCD(x,zy^2)=1よりx=±1でなければならない
同様にしてy=±1,z=±1
逆にこのとき条件は満たされる。
いずれの場合も
a=x
b=y
c=z
とa,b,cを定めればx=ab^2、y=bc^2、z=ca^2とあらわされるので示された。
843132人目の素数さん:2009/05/04(月) 18:15:39
>>842



GCD
っってどういう意味?
844132人目の素数さん:2009/05/04(月) 18:18:04
>>843
普通、整数a,bに対して
a|bでaがbを割り切ること
GCD(a,b)でaとbの最大公約数(Greatest Common Divisor)を表す。
845132人目の素数さん:2009/05/04(月) 19:05:37
>>781
立方体の頂点をABCD-EFGHとする。(1)(2)(3)では点Aを選ぶ、(4)(5)では点Aを選ばないとして一般性を失わない。
(1) AB,AD,AEの長さ1が3通り、AC,AF,AHの長さ\sqrt{2}が3通り、AGの長さ\sqrt{3}が1通りなので期待値は
(3+3\sqrt{2}+\sqrt{3})/7
846132人目の素数さん:2009/05/04(月) 19:09:19
>>842
x=1,y=4,z=2の時
xyz = 8
yx^2 + zy^2 + xz^2 = 4*1^2 + 2*4^2 + 1*2^2 = 4 + 32 + 4 = 40
847132人目の素数さん:2009/05/04(月) 19:12:54
>>846
「どの2つをとっても互いに異なる」

「どの2つをとっても互いに素」
って読み間違えてた。もっかいやってみます。
848845:2009/05/04(月) 19:14:32
続き
(2) ABCタイプは9通りで面積1/2、ACEタイプは面積\sqrt{2}/2で9通り、ACHタイプは面積\sqrt{3}/2で3通り。
(一辺aの正三角形の面積は\sqrt{3}a^2/4なので)
期待値は (3+3\sqrt{2}+\sqrt{3})/14
(3) 4点が同一平面上にあるとき、ABCDタイプは3通り、ABGHタイプは3通りで何れも体積0。
ACFHタイプは体積1/3で1通り、ABCEタイプは体積1/6で27通り、ACHGタイプは体積1/6で1通り。
期待値は(1/6*27+1/3*1+1/6*1)/35=1/7
(4) 選ばない点がABCタイプのとき、底面がAFGH、高さ1の錐体なので体積は1/3。場合の数は9通り。
ACEタイプのとき。FGH-BDは錐体を成さないので体積0。これは9通り。
ACHタイプも同様に錐体を成さず、3通りである。
期待値は(9*(1/3))/21=1/7
(5) 選ばない2点が立方体の辺AB,AD,AEのとき、体積は元の立方体の半分なので1/2である。
それ以外は錐体を成さないので体積0。従って期待値は(3*(1/2))/7=3/14

(3)の出典はは益田塾の阪大模試ですね。
http://88.xmbs.jp/ch.php?ID=checkmath3&c_num=96513
849132人目の素数さん:2009/05/04(月) 19:17:36
益田さん匿名になったんですね
850132人目の素数さん:2009/05/04(月) 19:42:07
有限項の正の整数列Aは次の条件を満たす。
条件:A(n+2) は A(n)をA(n+1)で割ったあまりである。(0<A(n+2)<A(n+1))
ただし、A(n)がA(n+1)で割り切れる場合、A(n+2)は定義されないものとし、
このとき、「有限項の数列Aは長さがn+1である」と言うものとする。

0<A(1)<100、かつ、0<A(2)<100を満たすとき、数列Aの長さの最大値と、そのときのA(1)、A(2)の値を求めよ。

例)
A(1)=60、 A(2)=36としたとき、
A(3)=24、A(4)=12となり、数列Aの長さは4である。
851132人目の素数さん:2009/05/04(月) 21:00:00
>>845
間違い。
同じ点を選んでもいいと設問にある。以下もだが。

>>848
(2)も0を無視していて期待値になっていない
(3)は0を無視せずカウントしている
一貫性がなくない?

(4)(5)錐体を成してない立体をカウントしないなら0もカウントしてはいけない
これは問題の不備だが。
852132人目の素数さん:2009/05/04(月) 21:06:20
x-y-z空間上に次の立方体がある。
-R≦x、y、z≦R (ただし、R>0)
この立方体の中に含まれていて、かつ、(x + y + z)/3 ≧ (xyz)^(1/3)を満たす領域の体積をV(R)とおく。

このとき、lim[R->∞]V(R)/(8*R^3) を求めよ。
853132人目の素数さん:2009/05/04(月) 22:07:28
これ極限って必要ですか
854132人目の素数さん:2009/05/05(火) 00:19:31
>>852
 x^(1/3) + y^(1/3) + z^(1/3) ≧ 0,
だが・・・

855132人目の素数さん:2009/05/05(火) 03:06:36
>>854
856132人目の素数さん:2009/05/05(火) 14:31:45
>>841回答着本
857132人目の素数さん:2009/05/05(火) 20:04:11
記号ΠはΣ記号を掛け算に置き換えたものである。
たとえば、Π[p≦x、pは素数]pはx以下の素数を全てかけたものであり、Π[p≦5、pは素数]=2*3*5=30となる。

このとき、x>0に対し

3^x > (Π[p≦x、pは素数]p)*(Π[p≦√x、pは素数]p)*(Π[p≦x^(1/3)、pは素数]p)*(Π[p≦x^(1/4)、pは素数]p)*……

を示せ。
858132人目の素数さん:2009/05/05(火) 21:21:02
P(N)をNの素因数のうち最大のものであるとする。
例) P(16)=2、 P(120)=5など。
このとき、2より大きい任意の自然数kに対し、ある互いに素な自然数a,d(d>1)が存在し、
P( a*(a + d)*(a + 2d)*……*(a + (k-1)d) ) ≦ k
を満たすことを示せ。

例) k=3の時、a=2、d=7が条件を満たす。
859132人目の素数さん:2009/05/05(火) 23:01:16
先日ふと、マンコの数が気になったので数えてみることにした。
1マンコ2マンコ3マンコと私は順調にマンコを数えていった。
そしてそれがある数に達したとき突然異変は起こった。
それは、9997マンコ…9998マンコ…9999マンコ…と数えた後である。
9999マンコのあと、次の数を数えようとしたところ、なんと1マンコに戻ってしまったではないか!

不思議に思い、また最初から数えなおしたのだがまたしても9999マンコの次で最初に戻ってしまった。
その後数回繰り返し実験してみたが、結果は同様であった。
試しにチンポを1から数えてみたところ、そのような現象は起こらなかった。

この発見を次の学会で発表するつもりである。
860132人目の素数さん:2009/05/06(水) 00:23:49
コラッツ予想が解けたと聞いてとんできますた
861132人目の素数さん:2009/05/06(水) 03:32:22
このスレだけなのか昔からなのかわからないが
理系らしさがあまりかんじられないな、ここ。
862132人目の素数さん:2009/05/06(水) 04:30:31
>>341
問題に不備が多い
863132人目の素数さん:2009/05/06(水) 12:06:10
>>852
 (x,y,z) が 与式を満たす。 ⇔ (-x,-y,-z) は与式を満たさない。
なので、対称性より
 V(R)/(8R^3) = 1/2.

857
素数pを1つ固定する。
 {1,2,3,・・・・,x} の中に、 p^kで割り切れるものはあるが p^(k+1) で割り切れるものはない
とすると、
 与式の右辺は pをk回含む。
∴ 与式の右辺はLCM(1,2,3,・・・・,x)
864132人目の素数さん:2009/05/06(水) 18:16:39
>>863

略すならもっと略す
書くならもうすこし書く
略し方がありえない


論外
865132人目の素数さん:2009/05/06(水) 19:45:51
>>864
>略し方がありえない
そうでもない
866132人目の素数さん:2009/05/06(水) 19:53:45
ま、わかってない証拠だろ
867132人目の素数さん:2009/05/06(水) 22:35:08
>>857
同じことだが・・・・
素数p に対して整数 k_p を
 p^k ≦ x < p^(k+1)
で定めると
 (右辺) = Π[p≦x, pは素数] e^k = LCM(1,2,3,・・・・, x)
868132人目の素数さん:2009/05/07(木) 00:26:54
なにがしたいのやら
869132人目の素数さん:2009/05/07(木) 01:30:37
>>841に誘導付けてみた

0でない整数x,y,zはどの2つをとっても互いに異なり、x,y,zの最大公約数は1である。
yx^2 + zy^2 + xz^2 がxyzで割り切れるとき、以下の問いに答えよ。

なお、以下では記号 | を 整数mが整数nを割り切るとき、m|nという意味で用いるものとする。

(1)
ある素数pが存在し、p|xであると仮定するとき、p|yzを示せ。

(2)
ある素数pが存在し、p|xかつp|yを満たす。
このとき、自然数q,rが存在して、 p^q|x かつ p^r|y かつ p^(q+1) はxを割り切らず、 p^(r+1)はyを割り切らないとするとき、
q,rの関係式を求めよ。

(3)
ある整数a,b,cが存在しx=ab^2、y=bc^2、z=ca^2とあらわせることを示せ。
870132人目の素数さん:2009/05/07(木) 01:37:08
>0でない整数x,y,zはどの2つをとっても互いに異なり

普通こんな言い方はしない。
0でない相異なる整数x,y,zとかく。
871132人目の素数さん:2009/05/07(木) 01:41:12
>>870
すまん

0でない相異なる整数x,y,zの最大公約数は1である。
yx^2 + zy^2 + xz^2 がxyzで割り切れるとき、以下の問いに答えよ。

なお、以下では整数mが整数nを割り切るとき、記号 | を用いて、m|nと記す。

(1)
ある素数pが存在し、p|xであると仮定するとき、p|yzを示せ。

(2)
ある素数pが存在し、p|xかつp|yを満たす。
このとき、自然数q,rが存在して、 p^q|x かつ p^r|y かつ p^(q+1) はxを割り切らず、 p^(r+1)はyを割り切らないとするとき、
q,rの関係式を求めよ。

(3)
ある整数a,b,cが存在しx=ab^2、y=bc^2、z=ca^2とあらわせることを示せ。

872132人目の素数さん:2009/05/07(木) 01:43:53
もっと言われそうなので、適当に直しとくか


>>870
すまん

0でない相異なる整数x,y,zの最大公約数は1である。
yx^2 + zy^2 + xz^2 がxyzで割り切れるとき、以下の問いに答えよ。

なお、以下では整数mが整数nを割り切るとき、記号 | を用いて、m|nと記す。

(1)
ある素数pが存在し、p|xであると仮定するとき、p|yzを示せ。

(2)
ある素数pが存在し、p|xかつp|yを満たす。
qを、p^n|xを満たす自然数nの最大値、rをp^n|yを満たす自然数nの最大値とする時、
q,rの関係式を求めよ。

(3)
ある整数a,b,cが存在しx=ab^2、y=bc^2、z=ca^2とあらわせることを示せ。
873132人目の素数さん:2009/05/07(木) 02:12:49
文系スレと名乗った方がいいのかもしれない
874132人目の素数さん:2009/05/07(木) 14:24:01
誘導つけた途端に簡単になった。特に(2)はクリティカル
875132人目の素数さん:2009/05/08(金) 02:47:21
lim[θ→0](sinθ)/θ=1となることを示せ
876132人目の素数さん:2009/05/08(金) 02:52:30
sinθ=θ+O(θ^2)より明らか
877132人目の素数さん:2009/05/08(金) 21:29:45
>>791, >>830

(1) 「3」のつかない数は  9^n 個、

a が3の倍数 ⇔ aの各桁の数字の和が3の倍数。ところで、
 0,6,9 ≡ 0 (mod 3),
 1,4,7 ≡ 1 (mod 3),
 2,5,8 ≡ 2 (mod 3),
う〜む、和が3の倍数となる頻度は、どう考えても1/3 だろうなぁ。という訳で、
 (1/3)(9^n)

(2) 「3」の付く数は 10^n - 9^n 個、

合計すると、
 10^n - (2/3)(9^n)  ・・・・ 答。
878132人目の素数さん:2009/05/08(金) 23:31:18
>>861
特に最近ここに投下される問題は文章に洗練さが感じられんな
てか問題文がきもい
しょうがないから俺が>>872の文を改善してやる

0でない相異なる整数x,y,zは最大公約数が1で、
xyz|yx^2 + zy^2 + xz^2である。
ただし、整数m,nに対し、mがnを割り切るとき、m|nと書く。
次の各問に答えよ。

(1) p|xなる素数pに対して、p|yzであることを示せ。
(2) p|x,p|yなる素数をpとする。
  p^n|x, p^m|yを満たす自然数n,mのうち最大のものをそれぞれq,rとするとき、
  q,rの満たす関係式を求めよ。
(3) x=ab^2、y=bc^2、z=ca^2を満たす整数a,b,cが存在することを示せ。

これでだいぶましになっただろ
879132人目の素数さん:2009/05/08(金) 23:37:47
>>878
肝心の答えは?
880132人目の素数さん:2009/05/08(金) 23:38:33
>>857
 x→∞ のとき、π(x)log(x)/x → 1,      (素数定理)
∴ x>N ⇒ log(3) > π(x)log(x)/x,
 3^x = exp(log(3)x) > exp(π(x)log(x)) = x^π(x) = Π[p≦x, pは素数] x > Π[p≦x, pは素数] p^k = LCM(1,2,3,・・・・,x).

x≦N のときは確認の必要が・・・・orz
881132人目の素数さん:2009/05/08(金) 23:57:03
>>879
お前が答えれば?
882132人目の素数さん:2009/05/09(土) 00:02:43
883132人目の素数さん:2009/05/09(土) 00:37:08
884132人目の素数さん:2009/05/09(土) 03:24:10
>>878
文だけの問題でもないけどな
そんなもん何を言おうとしてるかが分かればいいわけだから
よほど違和感のある使い方でない限りどうでもいい
というかそこまで要求するのは贅沢だろう、ここでは

それより、そういう言葉使いとか、記号の使い方とかの目先にばっかとらわれてて
内容がほとんどないところが理系らしくない
885132人目の素数さん:2009/05/09(土) 10:29:59
>>884
どういう意味? 内容がないってのは、何に対して、どういう理由で言ってるんだ?
886132人目の素数さん:2009/05/09(土) 11:10:35
>>878
誘導なしなら良かったと思うんだが……少なくとも>>847は解けなかったみたいだし
887132人目の素数さん:2009/05/10(日) 00:24:09
>>885
理系ならわかるんじゃないか?
スレタイ見て期待した理系が解いた甲斐あったと思うような問題ってここにはほとんど無いよ。

「こういう発想を問う」「こういう工夫を求める」という部分が明確じゃない、
というか存在してないのが多い

数学センスのない人間が、とにかく自分にとっての難易度だけ上げてみたとか、
どっかにある良問を、その問題のポイントを踏まえずにただ数値などをいじってみた
(結果、ポイントがわかってないのに変えてしまって良問である所以が失われた)
解けるかどうかも分からず、でたらめに作ってただ投げっぱなしてみた
というような感じのが多い
888132人目の素数さん:2009/05/10(日) 00:40:18
889132人目の素数さん:2009/05/10(日) 00:47:12
>>888

なるほど文系スレだわ
890132人目の素数さん:2009/05/10(日) 00:47:55
>>889
自分の感覚だけで語るところが特にね
891132人目の素数さん:2009/05/10(日) 00:50:07
多分お前の人間性は2億年経っても否定しかできないと思う
892132人目の素数さん:2009/05/10(日) 01:25:48
正数xを与えて、
      2 * a(1) = x、 2 * a(2) = {a(1)}^2 + 1、…、2 * a(n+1) = {a(n)}^2 + 1、 …
のように数列 a(n) を定めるとき
(1)  x≠2ならば、a(1)<a(2)<…<a(n)<…となることを証明せよ。
(2)  x<2ならば、a(n)<1となることを証明せよ。このとき、正数εを1-(x/2)より小となるようにとって、
  a(1)、a(2)、…、a(n)までが1-ε以下になったとすれば、個数nについて次の不等式が成り立つことを
  証明せよ
          2 - x > nε^2
893132人目の素数さん:2009/05/10(日) 01:40:23
与えられた実数係数の整式f(x)について
       ∫[0→1] f(x)dx = 2、 ∫[0→1] xf(x)dx = 3
となるとする。そのとき、
       ∫[0→1] { f(x) - ax - b }^2dx
の値を最小にする実数aおよびbの値を求めよ。
894132人目の素数さん:2009/05/10(日) 07:43:37
定数でない整数係数の多項式 f(x) であってすべての実数 x に対して
f (f '(x))=f '(f(x))
をみたすものをすべて求めよ。

(見にくいから一応補足するとg(x)=df/dxとおいたときf(g(x))=g(f(x))ということ。)
895132人目の素数さん:2009/05/10(日) 07:52:29
a+b+c=3をみたす正の実数a,b,cに対して不等式

a/(1+2b)+b/(1+2c)+c/(1+2a)≧1

が成立することを示せ。
896132人目の素数さん:2009/05/10(日) 11:08:53
>>895
1+2a=x,1+2b=y,1+2c=zと置くと、
a/(1+2b)+b/(1+2c)+c/(1+2a)
=(x-1)/(2y)+(y-1)/(2z)+(z-1)/(2x)
=(1/2)*(x/y+y/z+z/x)-(1/2)(1/x+1/y+1/z)
≧(3/2)*((x/y)*(y/z)*(z/x))^(1/3)-(1/2)*9/(x+y+z)=1 (∵相加相乗調和平均の関係、x,y,z>0、x+y+z=9)
等号はx=y=z、つまりa=b=cのとき成立

(*゚∀゚)=3 ハァハァ
897132人目の素数さん:2009/05/10(日) 12:19:55
だからなんでこんなに問題文がきもいの?
もっと良い言い回しはないかとかもうちょっと短くできないかとか
より良い表現を目指して考えないの?
意味が伝わればいいと上の方で誰か言ってたが
このスレはただの問題投稿スレではなく
入試作問者になったつもりのスレなんだから
そこまで意識してもらいたい

>>894-895はよし
898132人目の素数さん:2009/05/10(日) 12:49:30
>>897
確認するが、>>892,>>893はダメか?
899892=893:2009/05/10(日) 12:51:45
一応、実際の入試問題をかなり意識して書き込んだつもりなので、
問題文と内容、難易度などについて評価してもらえればうれしい。
そんなわけでよろしく頼む
900132人目の素数さん:2009/05/10(日) 13:12:09
>>897
とりあえず理由は良いから、問題文、難易度の2つについて5段階評価ぐらいで評価してくれ。頼む892と893ね
901132人目の素数さん:2009/05/10(日) 14:12:39
>>893 は去年の夏の東大実戦の理系第1問の数値替えだね。
902132人目の素数さん:2009/05/10(日) 17:53:05
>>892は難度は大数でいうところの(C寄りの)Bと見た
>>892は自分は一番最後の部分だけ解くのに手間取ってしまったけど、頻出問題だと思うし
大数のB評価の問題でもこれより難しい問題はあると思う
>>893は自分は難度A評価で、解いてみたあとの感じが難度Aの問題と変わらなかった
903132人目の素数さん:2009/05/10(日) 17:59:27
>>892
71年東大の第二問そのまま。
904132人目の素数さん:2009/05/10(日) 18:03:09
>>893
これも71年東大の第三問そのままだね。
東大の過去問以外で出題してくれると嬉しいんだけど。
905132人目の素数さん:2009/05/10(日) 18:46:15
>>896
> =(1/2)*(x/y+y/z+z/x)-(1/2)(1/x+1/y+1/z)
> ≧(3/2)*((x/y)*(y/z)*(z/x))^(1/3)-(1/2)*9/(x+y+z)

最初の項にAMGM使ってるのはわかるけど後の項の
-(1/x+1/y+1/z)≧-9/(x+y+z)
はどうやってるの?
906132人目の素数さん:2009/05/10(日) 18:52:55
>>897の気持ちはわかる。
問題文に違和感があると少しだが解く気を滅入らせられる。
まぁ解きたい人が解けばいいし、問題文の表現なんかより問題の本質のが大事だから
俺は別に「変えてくれ」、なんて言わないけどね。
907132人目の素数さん:2009/05/10(日) 19:06:08
>>894
最高次数の項を a[0]x^n とおいて,両辺の次数を比較してn=1.
f(x)=ax+(a-a^2) (aは実数)となる,でいいのかな.
908132人目の素数さん:2009/05/10(日) 19:17:03
>>907
>両辺の次数を比較してn=1

fが何次でも両辺の次数は等しくなると思うが・・・(degf=n>1ならdegff=degf'f=n(n-1))
909132人目の素数さん:2009/05/10(日) 19:54:36
>>894
解いてみた。あってるかわからんが。
http://www1.axfc.net/uploader/He/so/225293
910132人目の素数さん:2009/05/10(日) 22:42:46
n^{2011}+n^{2009}+1が素数になるような正の整数nをすべて求めよ。
911132人目の素数さん:2009/05/10(日) 22:57:33
>>910
誘導無しで入試に出すのはきついと思う。

n^{2011}+n^{2009}+1 は n^2+n+1 で割り切れるので、
n^{2011}+n^{2009}+1 が素数ならば、 n^2+n+1 = 1 または n^2+n+1 = n^{2011}+n^{2009}+1 である。
n^2+n+1 = 1 は正整数解を持たず、n^2+n+1 = n^{2011}+n^{2009}+1 の正整数解は n = 1 だけ。
逆に n = 1 のとき n^{2011}+n^{2009}+1 = 3 は素数である。
以上より、条件を満たす正整数は n = 1 だけである。
912132人目の素数さん:2009/05/10(日) 23:04:55
>>894
f(x) = (1/2)x^2 +b, (bは実数)
913132人目の素数さん:2009/05/10(日) 23:12:26
>>911
お見事

>>912
整数係数。
でも実数係数だったら解は求まるのかな?
914132人目の素数さん:2009/05/10(日) 23:43:45
>>911
何で割り切れるの?
915132人目の素数さん:2009/05/10(日) 23:57:22
>>914
n^2009(n^2+n+1)- (n^3)^670+1
916132人目の素数さん:2009/05/11(月) 00:00:15
>>897が核心ついたな

「入試作問者になったつもりのスレ」だから体裁のほうを優先的に気にして
中身は換骨奪胎や失敗作でも気にしない出題が多いと。
で、そこが気にならない人種だけが淘汰を免れて残ってると。
917132人目の素数さん:2009/05/11(月) 00:33:35
ハジケる幸せペニシリン・ロック
918132人目の素数さん:2009/05/11(月) 00:34:59
>>911
高1なら誘導なしは難しいかもな
919132人目の素数さん:2009/05/11(月) 00:36:37
京大ならありうるな
920132人目の素数さん:2009/05/11(月) 02:32:37
しかし、どんな誘導をつける?
921132人目の素数さん:2009/05/11(月) 02:41:17
(1)整式 x^3 - 1 を因数分解せよ
(2)自然数n、mについて n^(3m+1) + n^(3m-1) + 1 は n^2 + n + 1  でわりきれることをしめせ。

うーむ、イマイチ。こういうのも難しいな。
922132人目の素数さん:2009/05/11(月) 02:47:56
数学に熟練したやつなら誘導いらんだろ。
ベキ数の大きさからも因数分解できるのだろうと見当をつけられるはず・・
923132人目の素数さん:2009/05/11(月) 03:04:51
(1)n^{2011}+n^{2009}+1を割り切る定数でない整数係数多項式のうち最も次数の小さいものを一つ求めよ。

(2)n^{2011}+n^{2009}+1が素数になるような正の整数nをすべて求めよ。

こういうのはどう?
924132人目の素数さん:2009/05/11(月) 12:55:48
k(≧2)個の連続する自然数の集合Sには、次の条件を満たすxが必ず含まれることを示せ。
条件:xはSの他の要素全てと互いに素である。
925132人目の素数さん:2009/05/11(月) 13:34:08
>>924
大嘘
926924:2009/05/11(月) 14:38:30
マジか? すまん勘違いしてた。
927132人目の素数さん:2009/05/11(月) 15:04:25
>>924
>>925

じゃあ、

===
k(≧2)個の連続する自然数の集合Sには、次の条件を満たすxは存在しないことを示せ。
条件:xはSの他の要素すべてと互いに素である。
===
だとどうとくの?
928132人目の素数さん:2009/05/11(月) 15:15:03
>>921

(1)これおれ計算間違いか勘違いしてんのかなあ。。。

f(x)=x^3 - 1とおくと、f(1)=0
よって、
f(x) = (x-1)(x-α)(x-β)とおける。
計算すると、x^3 + { -(α+β)-1} * x^2 + {αβ+(α+β))} *x + αβ
ところでx^3 - 1 なので、{ -(α+β)-1} = {αβ+(α+β))} =0、αβ=1

これを満たす実数α、βは存在しない。よって因数分解できない。
==============---
てか、グラフからあきらか???
929132人目の素数さん:2009/05/11(月) 15:17:55
>>927
それも誤り。
930132人目の素数さん:2009/05/11(月) 15:19:27
>>927
否定文の作り方がおかしい。924の否定は

=========================
あるk≧2について、
次の条件を満たすxが存在しないような、
k(≧2)個の連続する自然数の集合Sが存在する。
条件:xはSの他の要素すべてと互いに素である。
=========================

だぞ。
931132人目の素数さん:2009/05/11(月) 15:23:49
存在すんの?
932132人目の素数さん:2009/05/11(月) 15:29:29
>>924
某所の問題に似ているなk=16で固定なら、そこと同じだ。
933132人目の素数さん:2009/05/11(月) 15:44:36
>>930
>>931
具体的にはk=17で題意のxが存在しないようなSを構成できる。


S={a_1,a_2,...,a_17}とおいたとき
a_1≡0 (mod 2, mod 5 mod 11)
a_2≡0 (mod 3)⇔a_1≡-1(mod 3)
a_3≡0 (mod 7)⇔a_1≡-2 (mod 7)
a_4≡0 (mod 13)⇔a_1≡-3 (mod 13)

となるようにa_1を設定できれば十分だが、中国剰余定理からこれを満たすa_1は存在する。
934132人目の素数さん:2009/05/11(月) 16:06:06
27830+30030k
935132人目の素数さん:2009/05/11(月) 16:18:04
GJ
936132人目の素数さん:2009/05/11(月) 16:21:00
ここの人たちって何してる人なの?大学生?予備校講師?
937132人目の素数さん:2009/05/11(月) 16:23:12
予備校教師がこの時間からry
938132人目の素数さん:2009/05/11(月) 16:25:09
東大って群環体論的な問題出したことある?
939132人目の素数さん:2009/05/11(月) 16:32:13
時期によるんじゃね?
昔は群論って高校でやってたって聞いたことあるし。その時代は普通に出てただろ
940132人目の素数さん:2009/05/11(月) 19:58:36
>>921
(1) x^3 -1 = (x-1)(x^2 +x+1),
(2) x^m -1 = (x-1){x^(m-1) + x^(m-2) + ・・・・ + x +1},
  x=n^3 とおくと、
  n^(3m) -1 = (n^3 -1){n^(3m-3) + (n^(3m-6) + ・・・・ + n^3 + 1}
       = (n^2 +n+1)(n-1){n^(3m-3) + n^(3m-6) + ・・・・ + n^3 + 1},
 与式 = (n^2 +n+1)n^(3m-1) - {n^(3m) -1}
    = (n^2 +n+1){n^(3m-1) - (n-1)[n^(3m-3) + ・・・・ + n^3 +1]},

うーむ、どこが難しい??
941132人目の素数さん:2009/05/11(月) 21:32:58
>>940
いや、難しいといっているのは、>>910へよい誘導をつけること。
(1)文字nの整式n^3-1を因数分解せよ。
(2)n^{2011}+n^{2009}+1が素数になるような正の整数nをすべて求めよ。

>>922さんの言うとおりいらんのかもね。
942132人目の素数さん:2009/05/11(月) 21:51:05
>>939
共通1次が始まるまえ、東大が自前の1次試験をしていた頃の一次試験の数学に
群論の「ぐ」の字もでてこないが、こんなような問題がでたことはある。
この通りの問題がでた、ということではない。もう少し凝っていたような気がする。

6個のxの関数 f_0(x)=x, f_1(x)=1/x, f_2(x)=1-x, f_3(x)=x/(x-1), f_4(x)(x-1)/x, f_5(x)=1/(1-x)
がある。
□の中に適当な数字(0,1,2,・・・,5)を入れよ
f_1(f_1(x))=f_i(x) のとき、 i は □ である。
f_2(f_3(x))=f_j(x) のとき、 j  は □ である。
f_4(f_k(x))=f_5(x) のとき、 k は □ である。
f_m(f_5(x))=f_3(x) のとき、 m は □ である。

 
943132人目の素数さん:2009/05/11(月) 22:52:33
(1)x^2+x+1=0の2つの解をa,bとおくとき、x=a,bに対してx^2011+x^2009+1=0になることを示せ。
(2)n^{2011}+n^{2009}+1が素数になるような自然数nをすべて求めよ。
944132人目の素数さん:2009/05/11(月) 23:01:45
>>928
(x-1)(x^2+x+1)と因数分解できる
そこまで因数分解するつもりなら複素数まで含めて因数分解してもいいと思うがなw
945928:2009/05/12(火) 02:16:24
>>944

おいら、あほだったw

高校数学課程に複素数が存在したころのニンゲンなもんで、
反射神経的に、(x-1)(x-α)(x-β)としてしまったw
946132人目の素数さん:2009/05/12(火) 02:27:05
こういう因数分解の問題って普通
実係数の範囲か複素係数の範囲のどっちで因数分解するか指定されてないか?
947132人目の素数さん:2009/05/12(火) 02:30:59
されない。
948132人目の素数さん:2009/05/12(火) 02:37:14
>>895

 f(x) = 1/(1+2x),
は単調減少かつ下に凸。よって Jensenより
 (左辺) = a・f(b) + b・f(c) + c・f(a) ≧ (a+b+c)f((ab+bc+ca)/(a+b+c)) ≧ 3・f(1) = 1,
∵ (ab+bc+ca)/(a+b+c) ≦ (a+b+c)/3 = 1,

http://science6.2ch.net/test/read.cgi/math/1179000000/891-893
不等式への招待3
949132人目の素数さん:2009/05/12(火) 03:11:06
そういえば数検一級の問題で実数係数の範囲で因数分解させる問題が出題されたが、ネットで公開された模範解答が整数係数の分解まででストップ。ここの板の住民が文句を言っても直らなかったため、秋山仁に直訴して模範解答を訂正させたなんてこともあった。
950132人目の素数さん:2009/05/12(火) 03:19:21
>ここの板の住民が文句を言っても
ちゃんとした手続きは踏んだのか?
ここでぼやいてるだけで訂正されるほど世の中おめでたくないぞ

951132人目の素数さん:2009/05/12(火) 04:58:52
当然ここで言ってるだけで訂正されるわけないでしょw
メールで協会に指摘したり、それに対し向こうが開き直ったりの経過報告もされていたよ。
で、結局下にいくら言っても話にならんからと仁さんに直訴になった。
それで直さなきゃマスコミ使うかなんて話も出てた。
952132人目の素数さん:2009/05/12(火) 05:11:37
>>921 の(2)を、
(1)を無視して、以下のように解こうとしたけど、解けそうで解けなかった・・・。
無謀でしょうか? by もと文系。

f(n,m) = n^(3m+1) + n^(3m-1) + 1 , g(n,m) = n^2 + n + 1 とする。
(1)n=1,m=1のとき、n^(3m+1) + n^(3m-1) + 1 = 3、n^2 + n + 1 = 3 よって題意は成り立つ。
(2)n=p,m=qのとき、題意が成立すると仮定する。(p.qは自然数)

この仮定のもとで、下記(2−A),(2−B)がともに成り立つ

(2−A)f(p,q+1) は g(p,q+1)で割り切れる。
(2−B)f(p+1,q) は g(p+1,q)で割り切れる。

よって、数学的帰納法により題意は示された。

===
高校生のとき、このやり方で何回か模擬試験を解いた。
953132人目の素数さん:2009/05/12(火) 07:49:44
任意の自然数kに対し、a^k - b^k が自然数になるとき、a,bは自然数であることを示せ。



プラスにした場合は明らかに……
954132人目の素数さん:2009/05/12(火) 07:56:50
>>953
a=2,b=-1
955132人目の素数さん:2009/05/12(火) 09:40:34
すまん整数だった
956132人目の素数さん:2009/05/12(火) 13:51:47
任意の自然数kに対し、a^k - b^k が整数になるとき、a,bは整数であることを示せ?
957132人目の素数さん:2009/05/12(火) 15:19:54
ume
958132人目の素数さん:2009/05/12(火) 19:05:46
>>956
a^k - b^k は正の整数とかにしなきゃ問題にならんだろ
959132人目の素数さん:2009/05/12(火) 19:39:05
>>952
>>943 (1)のようにすれば一発です。
960132人目の素数さん:2009/05/12(火) 20:48:53
>>953
a,bは何
961961:2009/05/12(火) 22:20:13
√(961) = 31 のアイスクリームっ
962132人目の素数さん:2009/05/13(水) 00:52:29
すまん寝ぼけまくってた

a,bを異なる実数とする。
任意の自然数kに対し、a^k - b^k が整数になるとき、a,bも整数であることを示せ。
963132人目の素数さん:2009/05/13(水) 07:17:50
k=1 a=3/2 b=1/2
964132人目の素数さん:2009/05/13(水) 07:30:38
しかもそれk=2でも成り立つな。
965132人目の素数さん:2009/05/13(水) 07:43:26
>>946
普通は指定されてない場合は実数の範囲で行う
複素数は極形式表示を考えると一意に定まらないからじゃないかな.a+biのように表示すると一意だが.

>>963-964
kは「任意」じゃないの?w
966132人目の素数さん:2009/05/13(水) 08:35:28
簡単な考察を行うことにより、
a=(px+r)/p,b=(py+r)/p (p,x,y∈Z,gcd(p,r)=1,p>0,0≦r<p)
という表示を得る。

任意の自然数mについてa^m−b^m∈Zだから、
a^m−b^m={(px+r)^m−(py+r)^m}/p^m∈Zとなり、
よってp^m|{(px+r)^m−(py+r)^m} を得る。
|の右側=Σ[k=1〜m]mCkp^kr^(m−k)(x^k−y^k)
=(x−y){p^m(x^m−y^m)/(x−y)+Σ[k=1〜m−1]mCkp^kr^(m−k)(x^k−y^k)/(x−y)}
であるから、結局 p^m|(x−y)Σ[k=1〜m−1]mCkp^kr^(m−k)(x^k−y^k)/(x−y)
となる。少し変形して
p^(m−1)|(x−y)Σ[k=1〜m−1]mCkp^(k−1)r^(m−k)(x^k−y^k)/(x−y) …(*)
となる。ここで、s|pなる素数sを任意に取るとき、
g :=gcd(s,Σ[k=1〜m−1]mCkp^(k−1)r^(m−k)(x^k−y^k)/(x−y))=gcd(s,mr^(m−1))=gcd(s,m)
となる(∵gcd(p,r)=1よりgcd(s,r)=1)。よって、gcd(p,m)=1なる自然数mを取れば、
そのときgcd(s,m)=1となるので、g=1となる。これはs|pなる任意の素数sで成り立つ。
よって、gcd(p,Σ[k=1〜m−1]mCkp^(k−1)r^(m−k)(x^k−y^k)/(x−y))=1
となり、これと(*)より、p^(m−1)|(x−y) となる。これがgcd(p,m)=1なる任意の自然数mに
ついて成り立つから、x−y=0あるいはp=1でなければならない。x−y=0のときはa≠bに
矛盾するので、p=1となり、このときa=x+r,b=y+rとなって、a,bともに整数である。
967963:2009/05/13(水) 08:56:44
>>965
すまん、あほな勘違いしてた
968132人目の素数さん:2009/05/13(水) 15:48:11
ume
969132人目の素数さん:2009/05/13(水) 18:57:15
>>911
何で割り切れる という事がすぐ分かるの?
970132人目の素数さん:2009/05/13(水) 19:24:37
971132人目の素数さん:2009/05/13(水) 19:37:23
>>969
すぐに分かるというわけではない
数字が大きいのと,素数という条件から因数分解出来ると考えるのが自然で試しに
(n±1)(nの2010次式)と因数分解することを試みる.(以下整数係数で考える)
数定理よりn=±1で与式は0となるはずだがならないので却下.
次に次数の低い2次式を考えると
n^{2011}+n^{2009}+1=(n^2+an±1)(nの2009次式)
という形となる.これがどのnでも成り立つ整数aを見つければよい.
試しに+側を取り,n=±1を代入すると
3=(2+a)(整数),-1=(2-a)(整数)
となることからa=1と取れ,結局与式は(n^2+n+1)(nの2009次式)と因数分解出来る.

>>943は(x-a)と(x-b)で割り切れる→(x-a)(x-b)=x^2+x+1 で割り切れるって方針だね
>>910のままだと旧帝向けだな(なんか九州大学とか名古屋大学が好きそう)
972132人目の素数さん:2009/05/13(水) 19:38:09
>>971
>数定理よりn=±1で与式は0となるはずだがならないので却下.
因数定理より〜ね
973132人目の素数さん:2009/05/13(水) 19:59:32
x^2+x+1=0が成り立つxについて、x^3=1が成り立つから、そのとき
x^2011=x^(1+670*3)=x
x^2009=x^(2+669*3)=x^2
となってx^2011+x^2009+1=x+x^2+1=0となる。
974132人目の素数さん:2009/05/13(水) 21:42:23
勘が鋭ければ1の3乗根ωについて知ってれば
ω^3=1,ω^2+ω+1=0から
ω^{2011}+ω^{2009}+1
=(ω^3)^{670}*ω+(ω)^{669}*ω^2+1
=ω+ω^2+1=0となって

n^{2011}+n^{2009}+1が(n-ω)(n-~ω)=n^2+n+1で割り切れると見抜けるでしょう。
975132人目の素数さん:2009/05/13(水) 21:46:19
ボケッとしてて勘が鋭くなくてもも>>971で解けるから誘導無しでおkだな
まとりあえず x^2009(x^2+1)+1 と括って何か(裏が)あるなと思えるようになって欲しいが
976132人目の素数さん:2009/05/13(水) 22:40:43
>>962

中学生or高1or2的な考え方で、方向性を変えないまま突っ走って解こうとしてみた・・・けど、あとちょっと?で解けない。。。

与条件において、「a^k - b^k が整数」⇔ 「b^k - a^k が整数」なので、
a,b の対称性と、与条件の a≠bにより、a > b のときについて、題意を示せば十分・・・(A)

また、一般的に、相異なる整数同士の「和」は整数であり、相異なる整数同士の「差」は整数である。

そこで、x > y なる任意の整数x,yがあるとき、以下の事実がなりたつ。

x+y, x-y はともに整数である・・・(1)

∴ (x + y) + (x - y) = 2*x , (x + y) - (x - y) = 2*y はともに整数である・・・(1)
∴ 2*x + 2*y = 2*(x + y) , 2*x - 2*y = 2*(x - y)はともに整数である・・・(2)
∴ 2*(x + y) + 2*(x - y) = 4*x = 2*{2*x}, 2*(x + y) - 2*(x - y)=4*y=2*{2*y}はともに整数である・・・(3)
∴ 4*x + 4*y = 4*(x + y) = 2*{2*(x + y)}, 4*x - 4*y = 4*(x - y) = 2*{2*(x - y)}はともに整数である・・・(4)

以下同様に、帰納的に、
--------------------
任意の自然数 t について、
(1)、(3)により、2^t * x, 2^t * yはともに整数であり、
(2)、(4)により、2^t * (x + y), 2^t * (x - y)も、ともに整数である。
--------------------
・・・(B)

・・・つづく・・・
977976:2009/05/13(水) 22:42:49
>>976 の続き。
さて、f(k) = a^k - b^k (kは整数)とおくと、予条件により、任意の自然数において、f(k) は整数。
すると、k>=2なる任意の自然数において
----------------------------------------------------------------------
f(k + 1) + f(k - 1) = a^(k - 1) * (k^2 + 1) - b^(k - 1) * (b^2 + 1) は整数である・・・(4)
f(k + 1) - f(k - 1) = a^(k - 1) * (k^2 - 1) - b^(k - 1) * (b^2 - 1) は整数である・・・(5)
----------------------------------------------------------------------
が成立する。
再度、整数同士の和は整数であり、整数同士の差も整数なので、(4)、(5)より、
----------------------------------------------------
2 * { a^(k + 1) - b^(k + 1) } は整数・・・(6)
2 * { a^(k - 1) - b^(k - 1) } は整数・・・(7)
----------------------------------------------------

以下同様に、加算、減算を繰り返し行うと、(B)により・・・
k>=2なる任意の自然数において
----------------------------------------------------
2^t * { a^(k + 1) - b^(k + 1) } は整数
2^t * { a^(k - 1) - b^(k - 1) } は整数
2^t * {( a^2 + 1) - ( b^2 + 1)} は整数
2^t * {( a^2 - 1) - ( b^2 - 1)} は整数 (tは任意の自然数)
----------------------------------------------------
が成立する。・・・(C)
★★★★★★★★★★★★★★★★★★★★★★★★★★★
ここから先、解けそうでとけない・・・。
(C)において、
   t = 1 のときを考え、
   2 * { a^(k + 1) - b^(k + 1) }は整数(kは2以上の任意の整数)
   2 * { a^(k - 1) - b^(k - 1) }は整数(kは2以上の任意の整数)
   2 * {(a^2 + 1) - ( b^2 + 1)}は整数
   2 * {(a^2 - 1) - ( b^2 - 1)}は整数
このあと、t やk に適当な数字を入れて、そいつを因数分解してみたり、剰余を考えたりしたけど、解けそうで解けない・・・。
978132人目の素数さん:2009/05/14(木) 00:04:06
979132人目の素数さん:2009/05/14(木) 00:05:39
>>974
>>973に全く同じことが書いてある。
980132人目の素数さん:2009/05/14(木) 02:06:33
二百五十六日。
981132人目の素数さん:2009/05/14(木) 05:08:52
レベル的には中学向けか

さしずめ「高校への数学」スレってとこだな
982132人目の素数さん:2009/05/14(木) 14:25:37
私は東大卒ですが、文科一類から理科三類までの並べ方にはちゃんと意味があります。
本当は偏差値もこの順になるように分けてます。
文科の場合は実際に1,2,3と大幅に易しくなっていくしとてもわかりやすいですよね・・・
理科の場合、論理的な思考が出来る知能の高い人が物理系研究者養成のための理一、そうでない人が生物系研究者養成のための理二・三,
その中でも末端のサービス業者養成のための理科三類が一番易しくなるように並べてあります。
また、三権に関わる文系を理系より先に置いて優越性を表示しています。
裁判官になれる人なら3日も勉強すれば医師国家試験で合格点取れます。
簡単なマルチョイで論述試験がある司法試験と比べたら屁のようなもの。
あの程度の試験に受かったくらいで他人を素人呼ばわりするのは傲慢ではないですか。
駒場の学生証番号は文一が1で始まり、文二は2,文三は3・・・理三は6で始まり、学生証番号もこの順になっています。私は1でしたけど。
何故か、偏差値が理三>他科類なのでみんな勘違いしているけど、東大の創設者は社会的地位の順番に科類を並べました。
ただ、最近は医師の質も落ちていて本当に理三が一番易しくなると思います。昨年度の足切りは400点で全科類中最低。
本当は価値の低い医学部医学科を騙されて高い値段(偏差値)で買ったことにはご愁傷様としか言いようがないです。
ネットオークションでとんでもない落札価格になってることがありますが、あれと同じですね。でも、自分でお手付きしてしまったのだから文句言う資格無し。
今後の人生では貧乏くじを引かないように気を付けて下さい。そして自分の職能をしっかり果たして下さい。
983132人目の素数さん:2009/05/14(木) 14:53:35
koko ha nani sure desu ka ?
984132人目の素数さん:2009/05/14(木) 15:23:23
>>982
ググッたらいっぱいでてきた。
有名なコピペなのか?
985132人目の素数さん:2009/05/14(木) 15:37:28
>>981
なんか感じ悪い奴増えたなぁ……
986132人目の素数さん:2009/05/14(木) 21:40:38
>>982
東大の医学部は東大よりも歴史が古いのにwww
とコピペにレスっとく

>>985
ほっとけ
987132人目の素数さん:2009/05/14(木) 21:47:07
こぴぺにマジレスするのもアレだが……
入学偏差値と大学の能力が釣り合わない所は確かに存在する。
しかし、東大理三は間違っても、そんな所ではない。
でも、確かにそんな所はある。でも、どこかは言わない……って、普通に調べてりゃ分かるわな
988132人目の素数さん:2009/05/14(木) 23:56:44
>>985
「高校への数学」甘く見ちゃダメだろ。
ああいうのを楽しんでる数学マニアの中学生と
非マニアの理系受験生だと

基礎知識は後者の方が安定してるかもしれないが
こういう場所では
>>981がけなし言葉になるほど前者が劣ってはいないだろ
989132人目の素数さん:2009/05/14(木) 23:58:18
あ、でも
「レベル的に中学向け」は明かにけなし言葉だな
990132人目の素数さん:2009/05/15(金) 02:06:28
二百五十七日。
991132人目の素数さん:2009/05/15(金) 08:11:37
nを自然数とする。
n*(n+1)*(n+2)が7以上の素因数を持たないとき、nの値を全て求めよ
992132人目の素数さん:2009/05/15(金) 11:06:45
ume
993132人目の素数さん:2009/05/15(金) 11:07:59
ume
994132人目の素数さん:2009/05/15(金) 15:57:32
n(n+1)(n+2)は連続した3つの整数の積より、6を因数に含む。
n=2kのとき。
1≦k∧n+2=2(k+1)≦6より1≦k≦2
したがってn=2,4
n=2k-1のとき
1≦k∧n+2=2k+1≦6より1≦k≦2
したがってn=1,3
よってn=1,2,3,4
995132人目の素数さん:2009/05/15(金) 16:19:18
>>994
正気ですか
996132人目の素数さん:2009/05/15(金) 16:58:34
ume
997132人目の素数さん:2009/05/15(金) 17:00:06
ume
998132人目の素数さん:2009/05/15(金) 17:01:17
ume
999132人目の素数さん:2009/05/15(金) 17:03:26
ume
1000132人目の素数さん:2009/05/15(金) 17:04:14
ume
10011001
このスレッドは1000を超えました。
もう書けないので、新しいスレッドを立ててくださいです。。。